You are on page 1of 103

Obz Naukowy

Olimpiady Matematycznej
Zwardo, 29 maja - 12 czerwca 2008
(wydanie pierwsze)
Obz Naukowy Olimpiady Matematycznej
Zwardo, 29 maja - 12 czerwca 2008
Dom wczasowy Zgoda, Zwardo 45A
34-373 Zwardo
tel. 0-33-864-63-28
Kadra:
Joanna Bogdanowicz
Wojciech Czerwiski
Maciej Gawron
Tomasz Kobos
Micha Krych
Przemysaw Mazur
Jakub Onufry Wojtaszczyk
Olimpiada Matematyczna w Internecie:
www.om.edu.pl
Wstp
Obz Naukowy Olimpiady Matematycznej odby si w dniach 29 maja - 12 cz-
erwca 2008 r. w Zwardoniu, w pensjonacie Zgoda. Kadr obozu stanowili: Joanna
Bogdanowicz, Wojciech Czerwiski, Maciej Gawron, Tomasz Kobos, Micha Krych,
Przemysaw Mazur oraz Jakub Onufry Wojtaszczyk.
W dniach 30 i 31 maja oraz 1, 2, 3, 6, 7, 9 i 10 czerwca uczestnicy obozu rozwizy-
wali zadania indywidualne (przy czym 1 czerwca miay miejsce wyjtkowe zawody
indywidualne, z okazji Dnia Dziecka), dnia 8 czerwca odbyy si zawody druynowe, a
4 i 11 czerwca rozegrane zostay mecze matematyczne (regulamin meczu znajduje
si na kocu tego zeszytu).
Kadego dnia zawodw indywidualnych uczestnicy otrzymywali 4 zadania (jedynie
w Dzie Dziecka 8) i mieli na ich rozwizanie 4,5 godziny. Zawody druynowe, jak i
mecze matematyczne trway cay dzie. Dla uczestnikw, ktrzy rozwizali ju wszys-
tkie zadania przygotowane zostay nieobowizkowe zadania nieco trudniejsze ni po-
zostae, za ktre nie mona byo otrzyma punktw, lecz nagrod.
W ramach zawodw indywidualnych mona byo uzyska 216 punktw. Trzy na-
jlepsze wyniki to: 185 punktw, 172 punkty i 165 punktw. Punkty uzyskane za
poszczeglne zadania przedstawia tabela na nastpnej stronie.
Dla uczestnikw zorganizowane zostay dwie wycieczki: 4 czerwca piesza wycieczka
na Wielk Racz, a 8 czerwca wycieczka pocigiem do

Zyliny na Sowacji.
Niniejszy zeszyt zawiera wszystkie zadania z obozu oraz szkice ich rozwiza.
Zeszyty z poprzednich Obozw Naukowych Olimpiady Matematycznej znajduj
si na stronie internetowej Olimpiady Matematycznej: www.om.edu.pl.
Kadra obozu Zwardo 2008
3
Zestawienie ocen z zawodw indywidualnych
Zad. l. prac
na 6p.
l. prac
na 5p.
l. prac
na 2p.
l. prac
na 0p.
1. 15 1 2 3
2. 5 - - 16
3. 12 - - 9
4. 5 4 - 12
5. 14 1 - 6
6. 14 2 - 5
7. - 1 - 20
8. 4 1 1 15
DD1. 20 - - 1
DD2. 2 - - 19
DD3. 6 - - 15
DD4. 12 2 - 7
DD5. 7 - - 14
DD6. 10 2 1 8
DD7. 16 1 - 4
DD8. 5 - - 16
9. 17 - 1 3
10. 16 - - 5
11. 5 - - 16
12. 1 - - 20
Zad. l. prac
na 6p.
l. prac
na 5p.
l. prac
na 2p.
l. prac
na 0p.
13. 18 - - 3
14. 12 2 - 7
15. 3 - - 18
16. 4 - 3 14
17. 14 1 - 6
18. 12 - - 9
19. 6 3 - 12
20. 2 - - 19
21. 8 - 1 12
22. 5 - - 16
23. 2 - - 19
24. 2 - 2 17
25. 4 11 - 6
26. 11 4 - 6
27. 3 - - 18
28. 2 - - 19
29. 20 - - 1
30. 7 1 - 13
31. 9 - - 12
32. 9 1 2 9
Uwaga: Kada praca bya oceniana w skali 0, 2, 5, 6 punktw, przy czym zadania
z Dnia Dziecka liczone byy z wag
1
2
.
4
Spis treci
Wstp 3
Zestawienie ocen 4
Spis treci 5
Treci zada 7
Zawody indywidualne 7
Dzie Dziecka 11
Zawody druynowe 12
Pierwszy mecz matematyczny 13
Drugi mecz matematyczny 15
Zadania nieco trudniejsze 16
Rozwizania 19
Zawody indywidualne 19
Dzie Dziecka 46
Zawody druynowe 50
Pierwszy mecz matematyczny 56
Drugi mecz matematyczny 66
5
Zadania nieco trudniejsze 80
Regulamin Meczu Matematycznego 101
6
Treci zada
Zawody indywidualne:
1. Kad liczb naturaln pomalowano na jeden z dwch kolorw.
Dowie, e dla kadej liczby naturalnej n istniej rne liczby naturalne
a, b > n takie, e liczby a, b i a +b s jednego koloru.
2. Niech O i I oznaczaj odpowiednio rodek okrgu opisanego i
wpisanego w nierwnoboczny trjkt ABC. Udowodni, e AIO 90

wtedy i tylko wtedy, gdy 2BC AB +AC.


3. Liczb naturaln n nazywamy wypasion, jeeli dla kadej liczby
pierwszej p dzielcej n liczba p
2
rwnie dzieli n. Rozstrzygn, czy ist-
nieje nieskoczenie wiele liczb n takich, e n oraz n + 1 s wypasione.
4. Wyznaczy wszystkie funkcje f : Z
+
Z
+
speniajce dla dowol-
nych liczb cakowitych dodatnich x i y warunek
f(x
2
+f(y)) = xf(x) +y.
5. Liczby rzeczywiste a
1
, a
2
, . . . , a
n
(gdzie n , 4) speniaj warunki
a
1
+a
2
+. . . +a
n
, n
oraz
a
2
1
+a
2
2
+. . . +a
2
n
, n
2
.
Dowie, e przynajmniej jedna z liczb a
1
, a
2
, . . . , a
n
jest nie mniejsza ni
2.
6. Znale wszystkie liczby naturalne n , 2 takie, e wszystkie liczby
naturalne mniejsze od n i wzgldnie pierwsze z n tworz cig arytmety-
czny.
7. Zabezpieczenie sejfu skada si z trzech k, z ktrych kade moe
by ustawione w jednej z omiu pozycji. Z powodu uszkodzenia mecha-
nizmu blokujcego sejf drzwiczki do sejfu mona otworzy, gdy dowolne
dwa koa znajduj si w prawidowej pozycji. Rozstrzygnij jaka jest na-
jmniejsza liczba prb, ktra gwarantuje otworzenie sejfu.
7
8. Punkt O jest rodkiem okrgu opisanego na trjkcie ABC. Punkt
K ley na boku AB, punkt L na boku AC oraz punkty K, O i L s
wspliniowe. Punkt R jest rodkiem odcinka BL, za S jest rodkiem
odcinka CK. Udowodnij, e BAC = SOR.
9.Dany jest ostroktny trjkt ABC, w ktrymAB ,= AC. Dwusieczna
kta Aprzecina bok BC w punkcie V , natomiast D jest spodkiem wysokoci
poprowadzonej z A. Okrg opisany na trjkcie AV D przecina boki AB
i CA odpowiednio w punktach F i E. Dowie, e proste AD, BE i CF
przecinaj si w jednym punkcie.
10.Szachownic 2009
2008
2009
2008
wypeniono liczbami rzeczywistymi
o module niewikszym ni 1. Co wicej suma liczb w kadym kwadracie
2 2 jest zerem. Udowodni, e suma wszystkich liczb na szachownicy
nie przekracza 2009
2008
.
11.Niech C bdzie liczb dodatni, za a
1
, a
2
, . . . nieskoczonym cigiem
liczb dodatnich speniajcym dla kadego i = 1, 2, . . . warunek 0 a
i

C oraz dla kadych liczb 1 i < j nierwno
1
i+j
[a
i
a
j
[. Udowodni,
e C , 1.
12.Niech P bdzie unormowanym wielomianem dziesitego stopnia
o wspczynnikach cakowitych. Rozstrzygn, czy liczby P(0), P(1),
. . . , P(100) mog dawa parami rne reszty przy dzieleniu przez 101.
13.Wielomian P spenia dla kadej liczby rzeczywistej x rwno
P(x)P(2x
2
) = P(2x
3
+x).
Udowodni, e jeli P ma pierwiastek rzeczywisty, to jest tosamociowo
rwny zero.
14.Niech okrgi o
1
i o
2
przecinaj si w punktach A i B. Pewna prosta
przechodzca przez B przecina o
1
i o
2
odpowiednio w punktach C i D.
Proste styczne do o
1
w C i o
2
w D przecinaj si w punkcie M. Proste
AM i CD przecinaj si w punkcie X. Punkt K jest takim punktem na
prostej AC, e proste XK i MC s rwnolege. Udowodni, e prosta
KB jest styczna do okrgu o
2
.
15.Dane s trzy liczby cakowite a, b, c i liczba pierwsza p , 5. Udowod-
ni, e jeeli an
2
+ bn + c jest kwadratem liczby cakowitej dla 2p 1
8
kolejnych wartoci n, to p[b
2
4ac.
16.Dana jest liczba naturalna n , 2. Tablica n n jest wypeniona
liczbami 0 i 1 w ten sposb, e kady podzbir n pl, z ktrych adne
2 nie le w jednej kolumnie ani w jednym wierszu zawiera co najmniej
jedno pole z liczb 1. Dowie, e istnieje i wierszy i j kolumn, gdzie
i +j , n + 1, takich, e na przeciciu kadego wiersza i kadej kolumny
jest liczba 1.
17.Udowodni, e istnieje nieskoczenie wiele liczb naturalnych n ta-
kich, e suma cyfr liczby 3
n
jest nie mniejsza ni suma cyfr liczby 3
n+1
.
18.Dany jest wielomian stopnia n speniajcy zaleno P(i) = 2
i
dla
i = 0, 1, 2, . . . , n. Wyznaczy P(n + 1).
19.Na paszczynie dany jest wielokt W o polu wikszym od n.
Udowodni, e mona tak przesun rwnolegle wielokt W, e w jego
wntrzu znajdzie si co najmniej n + 1 punktw kratowych.
20.Niech KL i KN bd stycznymi do okrgu k w punktach L i N.
M jest takim punktem, e K, N, M le na jednej prostej w tej wanie
kolejnoci. Okrg opisany na trjkcie KLM przecina okrg k w punkcie
P. Punkt Q jest rzutem prostopadym N na prost ML. Udowodni, e
MPQ = 2KML.
21.Ahmed i Fredek graj w gr na szachownicy n n, gdzie n jest
liczb nieparzyst. Ahmed stawia kka, a Fredek krzyyki. Na pocztku
wszystkie pola s puste, tylko w lewym dolnym rogu jest kko, a w
prawym grnym jest krzyyk. Zaczyna Ahmed. Ruch gracza polega na
postawieniu swojego znaczka na wolnym polu ssiadujcym przez krawd
z polem, na ktrym jest ju postawiony jego znaczek. Gdy gracz nie moe
wykona ruchu, to traci go. Gra koczy si, gdy aden z graczy nie moe
wykona ruchu. Gr wygrywa ten gracz, ktry wykona wicej ruchw.
Rozstrzygnij, ktry z graczy posiada strategi wygrywajc.
22.Dany jest wielomian P stopnia n , 2 o wspczynnikach cakow-
itych dodatnich a
n
, a
n1
, . . . , a
1
, a
0
speniajcych warunki: a
nk
= a
k
dla kadego k = 1, 2, . . . , n1 oraz a
n
= a
0
= 1. Udowodni, e istnieje
nieskoczenie wiele par liczb cakowitych a, b takich, e a[P(b) i b[P(a).
9
23.Punkt I jest rodkiem okrgu wpisanego w trjkt ABC, a D
punktem stycznoci tego okrgu z bokiem BC. Okrg jest styczny do
prostej BC w punkcie D a do okrgu opisanego na trjkcie ABC w
punkcie T, przy czym A i T le po jednej stronie prostej BC. Dowie,
e ATI = 90

.
24.Dane s dodatnie liczby rzeczywiste x
1
, x
2
, . . . , x
n
takie, e
n

i=1
x
i
= 1. Udowodni nierwno:
n

i=1
x
i

1 +x
1
+ +x
i1

x
i
+x
i+1
+ +x
n
<
1 +

5
2
.
25.W trjkcie ABC punkt D jest spodkiem wysokoci poprowad-
zonej z punktu A. Na pewnej prostej przechodzcej przez D wybrano
takie punkty E i F, rne od D, e AEB = AFC = 90

. Punkty M i
N s odpowiednio rodkami odcinkw BC i EF. Dowie, e ANM =
90

.
26.Dane jest 2n parami rnych liczb rzeczywistych a
1
, a
2
, . . . , a
n
,
b
1
, b
2
, . . . , b
n
oraz tablica n n. W pole lece w i-tym wierszu i w j-
tej kolumnie tablicy wpisano liczb a
i
+ b
j
. Udowodni, e jeli iloczyny
liczb we wszystkich kolumnach s rwne, to rwnie iloczyny liczb we
wszystkich wierszach s rwne.
27.W pola tablicy nn wpisano wszystkie liczby naturalne od 1 do n
2
.
Udowodni, e istniej dwa pola ssiadujce krawdzi takie, e wpisane
w nie liczby rni si o co najmniej n.
28.Dana jest liczba naturalna n , 2. Udowodni, e liczba 2
2
n
+ 1
posiada dzielnik pierwszy, ktry jest wikszy ni 2
n+2
(n + 1).
29.Liczby dodatnie a, b, x, y speniaj rwnoci a
2
+x = b
2
+y oraz
a +x
2
= b +y
2
, a take nierwno a +b +x +y < 2. Dowie, e a = b
oraz x = y.
30.Rozstrzygn czy istniej parami wzgldnie pierwsze liczby nat-
uralne a, b, c > 1, dla ktrych zachodz warunki: a[2
b
+ 1, b[2
c
+ 1,
c[2
a
+ 1.
10
31.Sfery opisana oraz wpisana w czworocian ABCD maj wsplny
rodek, AB = CD oraz wszystkie ciany tego czworocianu s trjktami
ostroktnymi. Udowodni, e rodki krawdzi czworocianu ABCD le
na jednej sferze wtedy i tylko wtedy, gdy jest on foremny.
32.Udowodni, e istnieje dokadnie jeden podzia zbioru liczb nat-
uralnych na rozczne zbiory A i B speniajcy warunek: dla dowolnej
liczby naturalnej n liczba sposobw zapisania n w postaci a
i
+ a
j
, gdzie
a
i
, a
j
A i a
i
,= a
j
jest rwna liczbie sposobw zapisania n w postaci
b
i
+b
j
, gdzie b
i
, b
j
B i b
i
,= b
j
.
Dzie Dziecka:
1. W zaczniku do zada masz aktualn kopi punktacji poszczegl-
nych uczestnikw obozu. Dla cigu rzeczywistych liczb a
1
, a
2
, . . . a
8
zmody-
kowanym wynikiem zawodnika nazywamy liczb

8
i=1
a
i
P
i
, gdzie P
i
to
liczba punktw, ktre ten zawodnik zdoby z itego zadania. Twoim
zadaniem jest tak dobra wagi a
i
, aby by liderem rankingu zmody-
kowanych wynikw ex aequo z jak najmniejsz liczb innych zawod-
nikw.
2. Znajd wszystkie pary liczb cakowitych a, b takie, e a
3
= 6b
2
+2.
3. Niech ABC bdzie trjktem rwnobocznym o polu 1, za P dowol-
nym punktem w jego wntrzu. Przez D, E, F oznaczamy rzuty pros-
toktne P odpowiednio na BC, CA i AB. Znajd najmniejsz moliw
sum pl trjktw BDP, CEP i FAP.
4. Majc dan kartk papieru A5 skonstruowa (bez uycia cyrkla
tudzie linijki) trjkt rwnoboczny. Mona zaoy, e boki kartki A5
dziel si w stosunku 1 :

2.
Uwaga: Do opisu konstrukcji naley doczy skonstruowany trjkt.
5. Czy z kwadratowej kartki papieru o wymiarach 7, 997, 99 potrasz
wyci 50 kwadratw jednostkowych?
6. Rozstrzygnij, czy istnieje 2008 rnych trjek parami wzgldnie
pierwszych liczb naturalnych a, b, c takich, e a
2
, b
2
i c
2
tworz cig aryt-
metyczny.
11
7. Joszua rzuca monet n razy, za Ahmed n + 1 razy. Oblicz praw-
dopodobiestwo, e Ahmed wyrzuci wicej orw ni Joszua.
8. Udowodnij, e dla kadego x (0, 1) i kadych liczb naturalnych
m, n zachodzi nierwno
(1 x
n
)
m
+ (1 (1 x)
m
)
n
, 1.
Zawody druynowe:
1. Znale wszystkie funkcje f : R R speniajce dla dowolnych
x, y R tosamo
f(x f(y)) = f(x) +xf(y) +f(f(y)).
2. Rozstrzygn czy istnieje zbir liczb naturalnych S mocy 2008 taki,
e suma elementw dowolnego podzbioru S jest potg liczby naturalnej
o wykadniku wikszym od 1.
3. Dana jest rodzina T podzbiorw k elementowych zbioru n elemen-
towego S, przy czym n > 2k. Kady k+1 elementowy podzbir S zawiera
dokadnie m , 1 zbiorw z rodziny T. Wykaza, e T zawiera wszystkie
k elementowe podzbiory S.
4. Dany jest okrg i rozczne, styczne do niego wewntrznie, okrgi

1
,
2
o rodkach O
1
, O
2
. Niech A i B bd punktami stycznoci wsplnej
stycznej zewntrznej
1
,
2
odpowiednio z
1
i
2
. Wsplne styczne
wewntrzne
1
,
2
przecinaj w punktach C, D, przy czym punkty A,
B, C i D le po tej samej stronie prostej O
1
O
2
. Udowodni, e proste
AB i CD s rwnolege.
5. Znale gur F R
2
o jak najmniejszym polu, w ktrej da si
obrci odcinek, tzn. istnieje funkcja ciga f : [0, 1] [0, 1] F taka,
e f(0, 0) = f(1, 1) oraz f(0, 1) = f(1, 0) oraz dla kadych p, q, t [0, 1]
zachodzi d(f(p, t), f(q, t)) = [p q[, gdzie d to metryka euklidesowa w
R
2
.
12
Pierwszy mecz matematyczny:
1. Kulk bdziemy nazywa kul o promieniu 1. Ukad n parami
rozcznych kulek zawartych w kuli K o promieniu R nazywamy dobrym,
gdy nie da si dooy do niego kolejnej kulki (rozcznej i zawartej w K),
za megadobrym, jeli jest dobry oraz nie istnieje dobry ukad o wikszej
liczbie kulek. Dla dwch megadobrych ukadw X i Y udowodni, e da
si tak ustawi rodki kulek z ukadu X w cig A
1
, A
2
, . . . , A
n
, za rodki
kulek z ukadu Y w cig B
1
, B
2
, . . . , B
n
, e dla kadego i 1, 2, . . . , n
dugo odcinka A
i
B
i
nie przekracza 2.
2. Znale wszystkie funkcje f : R
+
R
+
speniajce dla dowolnych
x, y R
+
tosamo
f(x +f(y)) = f(x +y) +f(y).
Uwaga: R
+
oznacza zbir liczb rzeczywistych dodatnich.
3. Niech a, b, c bd liczbami rzeczywistymi dodatnimi speniajcymi
nierwno
21ab + 2bc + 8ca 12.
Znale najmniejsz moliw warto wyraenia
1
a
+
2
b
+
3
c
.
4. Dana jest nieparzysta liczba pierwsza p. Udowodni, e zachodzi
kongruencja
p1

i=1
2
i
i
p2

p1
2

i=1
i
p2
(mod p.)
5. W czworocianie ABCD na krawdziach AB, AC, AD, BC, BD,
CD wybrano odpowiednio punkty K, L, M, N, O, P tak, e s one
rne od wierzchokw. Udowodni, e sfery opisane na czworocianach
AKLM, BKNO, CLNP i DMOP przecinaj si w jednym punkcie.
6. Ahmed i Fredek zdecydowali si zagra w gr. Tym razem maj
nieskoczon szachownic, pocztkowo pust. Ruch polega na postawie-
niu swojego znaku (Ahmed gra kkami, Fredek za krzyykami, Ahmed
13
rusza si pierwszy) na dowolnym pustym polu. Zwycia gracz, ktremu
uda si ustawi n swoich znakw w ssiadujcych polach jednego wiersza,
kolumny lub skosu. Rozstrzygn, czy istnieje takie n, dla ktrego przy
dobrej grze Fredka Ahmed nie zdoa wygra w skoczonej liczbie ruchw.
7. Znale wszystkie wielomiany P takie, e dla kadego x rzeczywis-
tego zachodzi rwno
P(x
2
+ 1) = (P(x))
2
+ 1.
8. Dany jest cig wektorw jednostkowych v
1
, v
2
, . . . , v
n
. Rozstrzygn,
czy moemy dobra taki cig znakw, e dla kadego k wektor
k

i=1
v
i
ley
w kole o promieniu 3.
9. Znale wszystkie rozwizania rwnania x
3
+2x+1 = 2
n
w liczbach
naturalnych.
10. Okrg o rodku O jest styczny wewntrznie do dwch okrgw w
jego wntrzu w punktach S i T. Okrgi te przecinaj si w punktach M i
N, przy czym punkt N ley bliej prostej ST. Udowodni, e proste OM
i MN s prostopade wtedy i tylko wtedy, gdy punkty S, N, T le na
jednej prostej.
11. Niech a bdzie liczb naturaln wiksz od 1. Cig a
n
deniujemy
wzorem
a
n
= a
n+1
+a
n
1.
Wykaza, e istnieje podcig cigu a
n
, ktrego dowolne dwa wyrazy s
wzgldnie pierwsze.
12. Symetralne bokw AB i BC nierwnobocznego trjkta ABC
przecinaj boki BC i AB odpowiednio w punktach A
1
i C
1
. Dwusieczne
ktw A
1
AC i C
1
CA przecinaj si w punkcie B

, a punkty A

oraz
C

deniujemy analogicznie. Dowie, e punkty A

, B

, C

le na jed-
nej prostej, ktra przechodzi przez rodek okrgu opisanego na trjkcie
ABC.
14
Drugi mecz matematyczny:
1. Dla danej liczby naturalnej n , 1 niech A oznacza liczb sposobw
na jak mona zapisa n w postaci sumy liczb cakowitych dodatnich
nieparzystych, a B niech oznacza liczb sposobw na jak mona zapisa
n w postaci sumy rnych liczb cakowitych dodatnich (w obu zapisach
nie zwracamy uwagi na kolejno wystpowania skadnikw). Udowodni,
e A = B.
2. Niech k, t > 1 bd wzgldnie pierwszymi liczbami naturalnymi.
Majc dan permutacj (a
1
, a
2
, . . . , a
n
) zbioru 1, 2, . . . , n moemy za-
mieni w niej dwie liczby miejscami jeli rni si o k lub t. Wykaza,
e zaczynajc od permutacji (1, 2, ..., n) moemy otrzyma kad permu-
tacj zbioru 1, 2, . . . , n wtedy i tylko wtedy, gdy n , k +t 1.
3. Zbiory A
1
, A
2
, . . . , A
n
s podzbiorami zbioru n-elementowego
A, przy czym kady z nich ma co najmniej 2 elementy. Dla kadego
dwuelementowego podzbioru A

zbioru A istnieje dokadnie jeden taki


A
i
, e A

A
i
. Udowodni, e jeli 1 i, j n to A
i
A
j
,= .
4. Rozwiza w liczbach rzeczywistych ukad rwna
_

_
a
2
2b
2
= 1
2b
2
3c
2
= 1
ab +bc +ca = 1
5. Niech a
1
, a
2
, ..., a
n
bd rnymi liczbami naturalnymi. Dowie, e
zachodzi nierwno
a
7
1
+a
7
2
+... +a
7
n
+a
5
1
+a
5
2
+... +a
5
n
, 2(a
3
1
+a
3
2
+... +a
3
n
)
2
6. Niech n bdzie liczb naturaln. Udowodni, e wielomian (x
2
+
x)
2
n
+1 jest nierozkadalny na iloczyn niestaych wielomianw o wspczyn-
nikach cakowitych.
7. ABCD jest czworoktem wypukym, w ktrym prosta AC jest
dwusieczn kta BAD. Punkt E ley na odcinku CD, a F jest przeci-
ciem BE i AC. Odcinek DF przeduamy do przecicia z bokiem BC w
punkcie G. Wykaza, e GAC = EAC.
15
8. Dany jest nierozwartoktny trjkt ABC. Punkt D jest spodkiem
wysokoci z A, natomiast I
1
oraz I
2
s odpowiednio rodkami okrgw
wpisanych w trjkty ABD i ACD. Prosta I
1
I
2
przecina AB i AC
odpowiednio w punktach P i Q. Udowodni, e AP = AQ wtedy i tylko
wtedy, gdy AB = AC lub A = 90

.
9. Dany jest trjkt ABC. Okrg o jest styczny do odcinkw AB i
AC odpowiednio w punktach D i E, rnych od B i C. Ten sam okrg
przecina bok BC w punktach K i L. Odcinki AL i DE przecinaj si w
punkcie P, a przektne czworokta BCED przecinaj si w punkcie Q.
Dowie, e punkty P, Q, K s wspliniowe.
10. Wyznaczy najmniejsz liczb naturaln n > 1, dla ktrej rednia
kwadratowa liczb 1, 2, ..., n jest liczb cakowit.
11. Dla danego h = 2
r
, gdzie r , 0, wyznaczy wszystkie liczby
naturalne k, dla ktrych istnieje m > 1 nieparzyste oraz liczba naturalna
n taka, e k[m
h
1 oraz m[n
m
h
1
k
+ 1.
12. Rozstrzygn dla jakich liczb naturalnych a istnieje nieskoczenie
wiele liczb bezkwadratowych n takich, e n[a
n
1.
Zadania nieco trudniejsze:
1. Wrd n osb niektre trjki byy razem na imprezie. Dla kadych
dwch rnych osb A i B istnieje dokadnie jedna osoba C taka, e
A, B i C byli razem na imprezie. Co wicej, jeli dla szeciu rnych
osb A, B, C, X, Y, Z trjki A, B, X, B, C, Y oraz C, A, Z byy razem na
imprezie, to rwnie X, Y, Z byli razem na imprezie. Znale wszystkie
n, dla ktrych taka sytuacja jest moliwa.
2. Niech punkty D, B, C, E le na jednej prostej w tej wanie
kolejnoci i niech punkt A spenia rwnoci AB = DB oraz AC = EC.
Poprowadmy dwusieczne ktw ABC oraz ACB i ich przecicia z
okrgiem opisanym na trjkcie ABC oznaczmy odpowiednio przez K i
L, za ich przecicia z przeciwlegymi bokami trjkta ABC odpowiednio
przez P i Q. Niech O
1
bdzie rodkiem okrgu opisanego na trjkcie
DBL, za O
2
rodkiem okrgu opisanego na trjkcie ECK. Przez S
oznaczmy punkt przecicia CO
1
i BO
2
. Udowodni, e AS PQ.
16
3. Dane jest n liczb rzeczywistych dodatnich x
1
, x
2
, . . . , x
n
o iloczynie
rwnym 1. Wykaza, e

1i<jn
(x
i
x
j
)
2
,
n

i=1
x
2
i
n.
4. W kadym punkcie kratowym paszczyzny, ktry ma niedodatni
wsprzdn x pooono jeden pionek. Dozwolone s ruchy polegajce na
wybraniu pewnej pary pionkw stojcych na ssiadujcych w pionie lub
poziomie punktach A i B i zbicie jednym z nich drugiego, tj. zdjcie
pionka z pola A i przestawienie pionka z pola B na taki punkt C, e A
jest rodkiem odcinka BC (w punkcie C, na ktry przestawiamy pionek z
punktu B, nie mg dotychczas sta aden pionek). Znale najwiksze x,
dla ktrego istnieje skoczona sekwencja dozwolonych ruchw, po ktrej
pewien pionek znajduje si w punkcie (x, y) (dla pewnego y).
5. Niech P bdzie wielomianem o wspczynnikach cakowitych, a n
liczb naturaln. Dla kadej liczby cakowitej dodatniej m liczba P(2
m
)
jest n-t potg pewnej liczby naturalnej. Udowodni, e wwczas P(x) =
(Q(x))
n
dla pewnego wielomianu Q o wspczynnikach cakowitych.
6. Niech S
1
, S
2
bd okrgami przecinajcymi si w dwch rnych
punktach A i B. Prosta przechodzca przez punkt A przecina okrg S
1
w punkcie C, a okrg S
2
w punkcie D. Punkty M, N, K le odpowied-
nio na odcinkach CD, BC, BD oraz prosta MN jest rwnolega do
BD, a prosta MK jest rwnolega do BC. uki BC okrgu S
1
oraz BD
okrgu S
2
zawieraj odpowiednio punkty E i F, przy czym prosta EN
jest prostopada do BC, a prosta FK jest prostopada do BD. Dowie,
e kt EMF jest prosty.
7. Udowodni, e istnieje liczba postaci 333333
333333
n
, gdzie n jest
liczb naturaln, zakoczona 333333
333333
trjkami w zapisie dziesitnym.
8. Wielomiany W i V nazywamy wzgldnie pierwszymi jeli nie istnieje
wielomian U stopnia dodatniego taki, e U[W i U[V . Niech P, Q i R bd
wzgldnie pierwszymi wielomianami stopnia dodatniego. Udowodni, e
jeeli zachodzi
_
P(x)
_
n
+
_
Q(x)
_
n
=
_
R(x)
_
n
,
to n 2.
17
9. S
1
, S
2
, ..., S
2008
s podzbiorami zbioru 1, 2, . . . , 2008, z ktrych
kady posiada parzyst liczb elementw. Dowie, e dla pewnych 1
i < j 2008 zbir S
i
S
j
rwnie posiada parzyst liczb elementw.
10. W nierwnoramiennym trjkcie ABC punkty M
a
, M
b
, M
c
oz-
naczaj odpowiednio rodki bokw BC, CA, AB. Punkt I jest rodkiem
okrgu wpisanego w ten trjkt, a punkty A

, B

, C

s punktami sty-
cznoci tego okrgu odpowiednio z bokami BC, CA, AB. Prosta k jest
prost symetryczn do prostej BC wzgldem prostej AI, a prosta l jest
prost prostopada do prostej IM
a
i przechodzc przez punkt A

. X
a
jest
punktem przecicia prostych k i l, a punkty X
b
i X
c
deniujemy analog-
icznie. Wykaza, e punkty X
a
, X
b
, X
c
le na jednej prostej, ktra jest
styczna do okrgu wpisanego w trjkt ABC.
11. Udowodni, e dla dodatnich liczb a, b, c zachodzi nierwno

2a
a +b
+

2b
b +c
+

2c
c +a
3
12. Na paszczynie dany jest n-kt wypuky P. Trjkt utworzony
z trzech rnych wierzchokw P nazywamy dobrym jeli wszystkie jego
boki maj dugo 1. Dowie, e jest nie wicej ni
2
3
n dobrych trjktw.
13. Cig (e
n
) deniujemy nastpujco. e
1
= 1, e
2
= 2, a e
n+1
jest na-
jmniejsz liczb, ktra dotychczas nie wystpia w cigu i NWD(e
n
, e
n+1
) >
1. Udowodni, e dla kadej odpowiednio duej liczby pierwszej p pier-
wszym wyrazem cigu (e
n
) podzielnym przez p jest wyraz 2p.
18
Rozwizania
Zawody indywidualne:
1. Kad liczb naturaln pomalowano na jeden z dwch kolorw.
Dowie, e dla kadej liczby naturalnej n istniej rne liczby naturalne
a, b > n takie, e liczby a, b i a +b s jednego koloru.
Rozwizanie
Sposb I
Pokaemy najpierw, e istniej a, b , 1 takie, e a, b i a + b s jednego
koloru. Przypumy przeciwnie.
Nazwijmy kolory czerwony i niebieski. Bez straty oglnoci zamy,
e 1 jest niebieskie. Jeli nie istnieje ju adna inna liczba niebieska, to
w oczywisty sposb dostajemy sprzeczno, wic istnieje pewna liczba
niebieska - n. Wwczas n 1 oraz n + 1 s czerwone. Gdyby 2 byo
czerwone, to dostalibymy w ten sposb czerwon trjk (2, n 1, n +1)
- zatem 2 jest niebieskie. Skoro 1 i 2 s niebieskie, to 3 jest czerwone.
Analogicznie jak poprzednio wnioskujemy, e istnieje pewna liczba
niebieska wiksza od 4 - niech bdzie to n. Wwczas n 1 oraz n + 2 s
czerwone. Jednak wwczas trjka 3, n 1, n + 2 jest czerwona i spenia
zaoenia zadania. Sprzeczno.
Analogicznie moemy wykaza, e istniej a, b , n + 1 speniajce
zaoenia zadania, rozpatrujc zamiast zbioru N zbir (n + 1)N, gdzie
kN = n : n = km, m N.
Sposb II
Przypumy nie wprost, e nie istniej takie liczby a, b > n, e a, b i
a+b s jednego koloru. Nazwijmy kolory czerwony i niebieski. Bez starty
oglnoci niech n+1 bdzie czerwone. Istniej co najmniej 2 liczby czer-
wone wiksze od 2n + 2, w przeciwnym wypadku od pewnego momentu
wszystkie liczby byyby niebieskie i teza byaby speniona. Niech wic
a > b > 2n+2 czerwone. Zatem b+a, b (n+1), (n+1) +a s niebieskie
i wszystkie rne. Jednak (b (n + 1)) + ((n + 1) + a) = b + a, czyli
istnieje niebieska trjka wiksza od n. Sprzeczno.
2. Niech O i I oznaczaj odpowiednio rodek okrgu opisanego i
wpisanego w nierwnoboczny trjkt ABC. Udowodni, e AIO 90

wtedy i tylko wtedy, gdy 2BC AB +AC.


19
Rozwizanie
Oznaczmy przez T przecicie dwusiecznej kta BAC z okrgiem opisanym
na trjkcie ABC. Punkt I ley wwczas na boku AT trjkta AOT i
nietrudno zauway, e nierwno AIO 90

jest rwnowana temu,


e TI AI. Powszechnie wiadomo (lub jest to proste wiczenie), e
TB = TI = TC. Z Twierdzenia Ptolemeusza wnioskujemy jednoczenie,
e
TC AB +TB AC = AT BC
czyli te
TI (AB +AC) = (AI +TI) BC.
Dlatego warunek TI AI rwnowany jest warunkowi
TI (AB +AC) , 2TI BC
lub po prostu
AB +AC , 2BC
i dowd jest zakoczony.
3. Liczb naturaln n nazywamy wypasion, jeeli dla kadej liczby
pierwszej p dzielcej n liczba p
2
rwnie dzieli n. Rozstrzygn, czy ist-
nieje nieskoczenie wiele liczb n takich, e n oraz n + 1 s wypasione.
Rozwizanie
Sposb I
Tak, istnieje nieskoczenie wiele n takich, e n i n +1 s wypasione. Za-
uwamy, e iloczyn dwch liczb wypasionych jest liczb wypasion oraz,
e kwadraty liczb naturalnych rwnie s wypasione. Zamy wic, e n
i n + 1 s wypasione. Wwczas wypasiona jest te liczba 4n(n + 1). Jed-
noczenie 4n(n+1) +1 = 4n
2
+4n+1 = (2n+1)
2
, a wic ta liczba take
jest wypasiona. Oznacza to, e majc dan par kolejnych liczb natural-
nych, ktre s wypasione moemy wygenerowa wiksz, gdy jasne jest,
e n < 4n(n + 1) . Aby zakoczy rozwizanie wystarczy zauway, i
liczby 8 oraz 9 s wypasione.
Sposb II
Rwnanie Pella a
2
8b
2
= 1 posiada nieskoczenie wiele rozwiza w
liczbach naturalnych. Dla kadego takiego rozwizania moemy przyj
n = a
2
, wtedy n + 1 = 8b
2
i oczywicie obie te liczby s wypasione, czyli
znalelimy nieskoczenie wiele liczb n speniajcych dany warunek.
20
4. Wyznaczy wszystkie funkcje f : Z
+
Z
+
speniajce dla dowol-
nych liczb cakowitych dodatnich x i y warunek
f(x
2
+f(y)) = xf(x) +y.
Rozwizanie
Sposb I
Oznaczmy C = f(1) + 1. Podstawiajc x = y = 1 otrzymujemy f(C) =
C. Wykaemy indukcyjnie, e zachodzi f(kC) = kC oraz f(kC + 1) =
kC +f(1).
Pocztek indukcji: istotnie f(C) = C; podstawiajc x = 1, y = C
otrzymujemy f(1 +C) = f(1) +C.
Krok indukcyjny: podstawiajc x = 1 oraz y = kC otrzymujemy f(1 +
kC) = f(1) +kC.
Podstawiajc x = 1, y = kC + 1 otrzymujemy
f((k+1)C) = f(1+kC+f(1)) = f(1+f(kC+1)) = f(1)+kC+1 = (k+1)C.
Zatem teza indukcji zostaa udowodniona.
Pokaemy teraz, e f(1) = 1. Mona to zrobi na rne sposoby. Za-
uwamy, e funkcja f jest rnowartociowa, gdy jeli f(a) = f(b), to
xf(x) +a = f(x
2
+f(a)) = f(x
2
+f(b)) = xf(x) +b.
Podstawiajc x = C, y = 1 mamy
f(C
2
+f(1)) = Cf(C) + 1 = C
2
+ 1.
Dla x = C
2
+f(1), y = 1 dostajemy
f((C
2
+f(1))
2
+f(1)) = (C
2
+f(1))(C
2
+ 1) + 1.
Podobnie podstawiajc x = C
2
+ 1, y = 1 mamy
f((C
2
+ 1)
2
+f(1)) = (C
2
+ 1)(C
2
+f(1)) + 1,
czyli z rnowartociowoci otrzymujemy
(C
2
+f(1))
2
+f(1) = (C
2
+ 1)
2
+f(1),
a co za tym idzie f(1) = 1.
Zatem C = 2 i przedstawiony wyej dowd indukcyjny pokazuje, e
musi zachodzi f(n) = n dla dowolnego n Z
+
.
21
Sprawdzamy, e faktycznie f(n) = n spenia warunki zadania, gdy
f(x
2
+f(y)) = x
2
+y = x x +y = xf(x) +y.
Sposb II
Podstawiajc do rwnania par x, z otrzymujemy f(x
2
+f(z)) = xf(x)+
z, a par y, z otrzymujemy f(y
2
+f(z)) = yf(y) +z.
Zauwamy, e znaczy to, e od pewnego miejsca dla funkcji f zachodzi
nastpujca wasno: gdy przesuniemy si o x
2
y
2
na osi, to wartoci
funkcji wzrosn o xf(x)yf(y). Zauwamy, e dla dowolnych par
_
(x, y)
_
oraz
_
(z, t)
_
stosunki
x
2
y
2
xf(x)yf(y)
oraz
z
2
t
2
zf(z)tf(t)
musz by rwne.
Przypumy, e nie jest to prawd, niech
x
2
y
2
xf(x)yf(y)
>
z
2
t
2
zf(z)tf(t)
.
Wartoci funkcji na przedziale [t
2
, z
2
] s ograniczone z gry przez pewne
M R. Zatem wartoci funkcji na przedziale [t
2
+ k(z
2
t
2
), z
2
+
k(z
2
t
2
)] s ograniczone z gry przez M + k(zf(z) tf(t)). Jednak
f(t
2
+m(x
2
y
2
)) = f(t
2
) + m(xf(x) yf(y)) dla dostatecznie duego
m wypadnie w przedziale [t
2
+ k(z
2
t
2
), z
2
+ k(z
2
t
2
)] takim, e
f(t
2
+m(x
2
y
2
)) > M +k(zf(z) tf(t)), gdy
x
2
y
2
xf(x)yf(y)
>
z
2
t
2
zf(z)tf(t)
.
Z ten sposb pokazalimy, e
xf(x)yf(y)
x
2
y
2
= k dla pewnego k R.
Zatem xf(x) kx
2
= yf(y) ky
2
, czyli xf(x) kx
2
= C dla pewnego
C. Innymi sowy f(x) = kx +
C
x
.
Aby obliczy wspczynniki k oraz C mona teraz oznaczy D = f(1)+1
i zauway, e podstawiajc par
_
(1, 1)
_
do pierwotnego rwnania mamy
f(D) = D, par
_
(1, D)
_
mamy f(D+1) = D+f(1) i par
_
(1, D+1)
_
mamy f(2D) = 2D. W ten sposb natychmiast dostajemy, e k = 1 i
C = 0. Sprawdzamy, e funkcja f(x) = x spenia pocztkowe rwnanie,
zatem jest to jedyne rozwizanie zadania.
5. Liczby rzeczywiste a
1
, a
2
, . . . , a
n
(gdzie n , 4) speniaj warunki
a
1
+a
2
+. . . +a
n
, n
oraz
a
2
1
+a
2
2
+. . . +a
2
n
, n
2
.
Dowie, e przynajmniej jedna z liczb a
1
, a
2
, . . . , a
n
jest nie mniejsza ni
2.
22
Rozwizanie
Sposb I
Dowd przeprowadzimy nie wprost. Przypumy, e a
i
< 2 dla i =
1, 2 . . . , n. Zauwamy najpierw, e jeeli rozsuwamy liczby tak, by ich
suma si nie zmieniaa, to suma kwadratw ronie. Formalnie, niech
a > b, x > 0. Wwczas
(a +x)
2
+ (b x)
2
= a
2
+ 2ax +x
2
+b
2
2bx +x
2
= a
2
+b
2
+ 2x
2
+ 2x(a b) > a
2
+b
2
.
Jeeli wic rozsuniemy liczby a
1
, a
2
, . . . , a
n
w ten sposb, e wszystkie
oprcz jednej bd rwne 2, a ta ostatnia wyniesie a
1
+ a
2
+ . . . + a
n

(n 1) 2, to suma kwadratw wzronie, czyli
a
2
1
+a
2
2
+. . . +a
2
n
< (n 1) 2
2
+ (a
1
+a
2
+. . . +a
n
(n 1) 2)
2
.
Jednak a
1
+a
2
+. . . +a
n
, n, czyli
a
2
1
+a
2
2
+. . . +a
2
n
< (n 1) 2
2
+ (a
1
+a
2
+. . . +a
n
(n 1) 2)
2
4(n 1) + (n 2)
2
= 4n 4 +n
2
4n + 4 = n
2
,
gdzie nierwno bierze si z tego, e a
1
+ a
2
+ . . . + a
n
2(n 1) ,
n2(n1) , (n2) oraz a
1
+a
2
+. . . +a
n
2(n1) < 2n2(n1) =
2 4 2 n 2.
Jednak z zaoe zadania wynika, e a
2
1
+ a
2
2
+ . . . + a
2
n
, n
2
, czyli
dostajemy sprzeczno z zaoeniem, e a
i
< 2 dla i = 1, 2, . . . , n.
Sposb II
Po pomnoeniu pierwszej nierwnoci przez n 4 (n , 4) i dodaniu
drugiej otrzymujemy
n

i=1
(a
2
i
+ (n 4)a
i
) , 2n
2
4n.
Przeksztacajc dostajemy
n

i=1
(a
i
2)(a
i
+ (n 2)) , 0.
Zatem dla pewnego i mamy (a
i
2)(a
i
+ (n 2)) , 0, czyli a
i
, 2 lub
a
i
2 n. Jeeli zachodzi pierwsza z nierwnoci, to otrzymujemy tez.
23
W drugim przypadku mamy 2 n , a
i
, bez straty oglnoci zamy
i = 1. Wwczas dodajc nierwno 2 n , a
1
do pierwszej nierwnoci
z treci zadania i skracajc otrzymujemy
n

i=2
(a
i
2) , 0,
co implikuje a
i
, 2 dla pewnego i, czyli tez.
6. Znale wszystkie liczby naturalne n , 2 takie, e wszystkie liczby
naturalne mniejsze od n i wzgldnie pierwsze z n tworz cig arytmety-
czny.
Rozwizanie
Wszystkie liczby n speniajce zadane warunki to liczby pierwsze, potgi
dwjki oraz liczba 6. Istotnie, zamy, e wszystkie liczby naturalne
mniejsze od n i wzgldnie pierwsze z n tworz cig arytmetyczny o rnicy
r. Jeli r = 1, to n nie posiada adnych dzielnikw wikszych od 1, a
zatem n jest liczb pierwsz. Jeli r = 2, to n nie posiada dzielnikw
pierwszych nieparzystych, a posiada parzyste czyli jest potga dwjki.
Przyjmijmy wic, e r , 3. Wtedy oczywicie n jest liczb parzyst,
a zatem moemy zapisa n = 2
a
b, gdzie b jest nieparzyste i wiksze
ni 1, gdy jeli n byoby potga liczby 2, to r = 2, a ten przypadek
rozpatrzylimy ju wczeniej. Nietrudno zauway, e liczby b 2, b + 2
s mniejsze ni n oraz wzgldnie pierwsze z n, czyli nale do cigu
arytmetycznego. Poniewa r > 2 mamy zatem r = 4. W szczeglnoci
3[n. Jeli n > 6, to n , 12. Wypisujc jednak trzy pierwsze wyrazy
naszego cigu: 1, 5, 9 otrzymujemy sprzeczno. Bez trudu sprawdzamy,
e n = 6 rwnie spenia warunki zadania.
7. Zabezpieczenie sejfu skada si z trzech k, z ktrych kade moe
by ustawione w jednej z omiu pozycji. Z powodu uszkodzenia mecha-
nizmu blokujcego sejf drzwiczki do sejfu mona otworzy, gdy dowolne
dwa koa znajduj si w prawidowej pozycji. Rozstrzygnij jaka jest na-
jmniejsza liczba prb, ktra gwarantuje otworzenie sejfu.
Rozwizanie
Odpowied: 32.
Oznaczmy pozycje numerami 0, 1, . . . , 7. Prawidowy kod skada
si z trzech cyfr; co najmniej dwie z nich s w zbiorze 0, 1, 2, 3 lub
24
4, 5, 6, 7. Sprawdzamy wobec tego trjki (a, b, c), gdzie 4[a +b +c oraz
wszystkie trzy liczby nale do jednego z dwch wspomnianych zbiorw.
Takich trjek jest 32: wybieramy jeden z dwch zbiorw, liczb a na
cztery sposoby i liczb b na cztery sposoby; kady taki ukad daje nam
jedn trjk (a, b, c). Takie postpowanie gwarantuje nam otworzenie se-
jfu; wybieramy pozycje, na ktrych liczby z otwierajcego ukadu s w
jednym zbiorze i ustalamy tam prawidowe cyfry - wtedy na pozostaej
da si tek dobra cyfr tak, by otrzymana trjka bya sprawdzana. Przy-
pumy teraz, e uda si otworzy sejf w co najwyej 31 sprawdzeni-
ach. Wtedy istnieje takie a, e sprawdzamy co najwyej 3 trjki postaci
(a, b, c). Bez straty oglnoci przyjmijmy, e sprawdzamy trjki (0,0,0),
(0,1,1) i (0,2,2). Gdyby sejf otwieraa trjka (0, b, c), gdzie 3 b, c, to
aby go otworzy, musimy sprawdzi trjk (a, b, c), gdzie a > 0. Daje to
25 trjek do sprawdzenia; razem z poprzednimi trzema 28 (zauwamy, e
gdybymy sprawdzili mniej ni 3 trjki postaci (0, b, c) lub zestaw trjek
istotnie inny ni (0,0,0), (0,1,1), (0,2,2) to na tym etapie mielibymy ju
5 6 +3, 6 6 +2, 7 7 +1 lub 88 sprawdze - sprzeczno). Do tej pory
nie otworzylimy sejfu, jeli otwiera go jedna z trjek: (1,0,1), (1,0,2),
(2,0,1), (2,0,2), (3,1,0), (3,2,0), (4,1,0), (4,2,0). Jednak do sprawdzenia
pierwszych czterech moliwoci potrzeba dwch trjek i do ostatnich
czterech te dwch, przy czym s to zbiory rozczne (bo trjka z pier-
wszych czterech i trjka z ostatnich czterech rni si na wszystkich
trzech miejscach), razem 32 - sprzeczno.
8. Punkt O jest rodkiem okrgu opisanego na trjkcie ABC. Punkt
K ley na boku AB, punkt L na boku AC oraz punkty K, O i L s
wspliniowe. Punkt R jest rodkiem odcinka BL, za S jest rodkiem
odcinka CK. Udowodnij, e BAC = SOR.
Rozwizanie
Niech B

, C

oznaczaj punkty symetryczne do punktw B, C wzgldem


punktu O. Z twierdzenia Talesa mamy SO[[KC

i OR[[B

L. Ponadto jeli
X jest punktem przecicia prostej KC

i okrgu przechodzcego przez


punkty A, B, C, to z twierdzenia Pascala dla szeciokta ABB

XC

C
wynika, ze punkty K, O oraz przecicie B

X z AC lea na jednej prostej.


Oznacza to, e proste KO, AC i B

X przecinaj si w jednym punkcie,


czyli L ley na prostej B

X. Wobec tego SOR = C

XB

. Aby za-
koczy dowd wystarczy zauway, e uki BC i B

s rwnej du-
goci, wic BAC = C

XB

25
B
C
A
O
K
L
S
R
B
C
X
9. Dany jest ostroktny trjkt ABC, w ktrymAB ,= AC. Dwusieczna
kta Aprzecina bok BC w punkcie V , natomiast D jest spodkiem wysokoci
poprowadzonej z A. Okrg opisany na trjkcie AV D przecina boki AB
i CA odpowiednio w punktach F i E. Dowie, e proste AD, BE i CF
przecinaj si w jednym punkcie.
Rozwizanie
Poniewa AD V D, wic AV jest rednic okrgu. Wobec tego AE
V E oraz AF V F. W poczeniu z rwnoci EAV = FAV daje
to AE = AF oraz V E = V F. Mamy w takim razie:
AF
FB

BD
DC

CE
EA
=
CE
FB

BD
DC
=
V E ctg C
V F ctg B

AD ctg B
AD ctg C
= 1,
co w poczeniu z twierdzeniem odwrotnym do twierdzenia Cevy daje
tez.
10. Szachownic 2009
2008
2009
2008
wypeniono liczbami rzeczywistymi
o module niewikszym ni 1. Co wicej suma liczb w kadym kwadracie
2 2 jest zerem. Udowodni, e suma wszystkich liczb na szachownicy
26
nie przekracza 2009
2008
.
Rozwizanie
Kolorujemy szachownic w odpowiedni sposb - schemat pokazany na
rysunku. Lewe dolne schody skadaj si z pewnej liczby kwadratw
2 2, czyli suma liczb na ich polach wynosi 0. Rwnie prawe grne
schody posiadaj t waciwo. Z pl caej planszy wszystkie pola
zostay pokolorowane dokadnie raz oprcz pl na przektnej. Oznaczmy
przez B sum liczb na biaych polach, przez C sum liczb na polach
pokolorowanych 2 razy, przez S sum liczb na caej szachownicy, przez L
sum liczb na lewych schodach, a przez P na prawych. Niech b to liczba
pl biaych, a c czarnych. Zauwamy, e wwczas S = L +P C +B =
0 +0 +BC = BC < b +c = 2009
2008
, czyli tym samym teza zostaa
wykazana.
11. Niech C bdzie liczb dodatni, za a
1
, a
2
, . . . nieskoczonym
cigiem liczb dodatnich speniajcym dla kadego i = 1, 2, . . . warunek
0 a
i
C oraz dla kadych liczb 1 i < j nierwno
1
i+j
[a
i
a
j
[.
Udowodni, e C , 1.
27
Rozwizanie
Niech n bdzie dowoln liczb naturaln, a (k
1
, k
2
, . . . , k
n
) tak permu-
tacj cigu (1, 2, . . . , n), e a
k
1
< a
k
2
< . . . < a
k
n
. Zachodz nierwnoci:
C , a
k
n
a
k
1
=
n1

i=1
[a
k
i+1
a
k
i
[ ,
n1

i=1
1
k
i+1
+k
i
,
,
(n 1)
2
n1

i=1
(k
i+1
+k
i
)
>
(n 1)
2
2
n

i=1
k
i
=
(n 1)
2
n(n + 1)
,
gdzie druga nierwno wynika z nierwnoci Cauchyego-Schwarza dla
liczb (k
1
+k
2
), . . . , (k
n1
+k
n
) oraz
1
k
1
+k
2
, . . . ,
1
k
n1
+k
n
.
Przechodzc do granicy przy n dostajemy C , 1.
12. Niech P bdzie unormowanym wielomianem dziesitego stopnia
o wspczynnikach cakowitych. Rozstrzygn, czy liczby P(0), P(1),
. . . , P(100) mog dawa parami rne reszty przy dzieleniu przez 101.
Rozwizanie
Odpowied jest negatywna. Zaczniemy od udowodnienia nastpujcego
lematu:
Lemat. Niech p ,= 2 bdzie liczb pierwsz, natomiast k liczb naturaln
niepodzieln przez p 1. Wwczas p[1
k
+ 2
k
+... + (p 1)
k
.
Wiadomo, e istnieje generator grupy multiplikatywnej modulo p, tzn.
taka reszta g modulo p, e liczby ze zbioru g
0
, g
1
, g
2
, . . . g
p2
daj wszys-
tkie moliwe niezerowe reszty modulo p (lub te inaczej, rzd g modulo
p wynosi p 1). Wwczas g
k
, 1 (mod p), gdy p 1 nie dzieli k. Za-
uwamy, e liczby g, 2g, ..., (p 1)g daj wszystkie moliwe niezerowe
reszty z dzielenia przez p i kada reszta pojawia si dokadnie raz. Zatem
g
k
+ (2g)
k
+...((p 1)g)
k
1
k
+ 2
k
+... + (p 1)
k
(mod p),
a std
(g
k
1)(1
k
+ 2
k
+... + (p 1)
k
) 0 (mod p).
Poniewa pierwszy czynnik jest niepodzielny przez p podzielny musi by
drugi i dowd lematu jest zakoczony.
Dla dowodu nie wprost zamy, e istnieje wielomian P(x) stopnia 10,
dla ktrego liczby P(0), P(1), . . . , P(100) daj rne reszty przy dzieleniu
28
przez 101. Nietrudno wwczas zauway, e P(P(x)) jest unormowanym
wielomianem stopnia 100 i liczby P(P(0)), P(P(1)), . . . , P(P(100)) te
daj rne reszty przy dzieleniu przez 101. Mamy wic
P(P(0))+P(P(1))+. . . +P(P(100)) 0+1+. . . +100 0 (mod 101).
Z drugiej jednak strony niech P(P(x)) = x
100
+ a
99
x
99
+ . . . + a
1
x + a
0
.
Wtedy
P(P(0)) +P(P(1)) +. . . +P(P(100))
(0
100
+ 1
100
+. . . 100
100
) +a
99
(0
99
+ 1
99
+. . . 100
99
)
+. . . +a
1
(0
1
+ 1
1
+. . . 100
1
) + 101a
0
100 (mod 101),
gdy na mocy lematu przy kadym wspczynniku a
i
jest liczba podzielna
przez 101, a reszt z dzielenia sumy 0
100
+ 1
100
+... + 100
100
obliczamy z
Maego Twierdzenia Fermata. Oczywicie 100 , 0 (mod 101) i uzyskana
sprzeczno koczy dowd.
13.Wielomian P spenia dla kadej liczby rzeczywistej x rwno
P(x)P(2x
2
) = P(2x
3
+x).
Udowodni, e jeli P ma pierwiastek rzeczywisty, to jest tosamociowo
rwny zero.
Rozwizanie
Przypumy przeciwnie i oznaczmy przez x
0
pierwiastek P o najwikszej
wartoci bezwzgldnej. Mamy wwczas
P(2x
3
0
+x
0
) = P(2x
2
0
)P(x
0
) = P(2x
2
0
) 0 = 0.
Zatem 2x
3
0
+x
0
jest rwnie pierwiastkiem P. Co wicej, jeeli x
0
,= 0, to
[2x
3
0
+x
0
[ > [x
0
[, gdy znak x
3
0
jest taki sam co x
0
. Wobec tego jedynym
pierwiastkiem P jest 0, czyli P(x) = x
n
Q(x) oraz Q(0) ,= 0. Zatem
x
n
2x
2n
Q(x)Q(2x
2
) = (2x
3n
+x
n
)Q(2x
3
+x).
Spjrzmy jednak na krotno pierwiastka w x = 0. Lewa strona rwnania
ma 0 w 3n potdze, prawa za w n potdze. Zatem n = 3n, czyli n = 0.
Sprzeczno z zaoeniem, e P / 0.
14. Niech okrgi o
1
i o
2
przecinaj si w punktach A i B. Pewna prosta
przechodzca przez B przecina o
1
i o
2
odpowiednio w punktach C i D.
29
Proste styczne do o
1
w C i o
2
w D przecinaj si w punkcie M. Proste
AM i CD przecinaj si w punkcie X. Punkt K jest takim punktem na
prostej AC, e proste XK i MC s rwnolege. Udowodni, e prosta
KB jest styczna do okrgu o
2
.
Rozwizanie
O
B
C
D
M
X
K
A
Poniewa BAC = BCM = KXC, wic czworokt ABXK jest
wpisany w okrg. Ponadto ACM = 180

ABC = ABD = 180

ADM, wic czworokt ACMD te jest wpisany w okrg. Oznaczmy


przez O rodek okrgu o
2
. Zachodz rwnoci:
ABK = AXK = AMC = ADC =
ADB =
1
2
AOB = 90

ABO,
czyli KBO = 90

, i mamy tez.
15. Dane s trzy liczby cakowite a, b, c i liczba pierwsza p , 5.
Udowodni, e jeeli an
2
+ bn + c jest kwadratem liczby cakowitej dla
2p 1 kolejnych wartoci n, to p[b
2
4ac.
30
Rozwizanie
Niech f(n) = an
2
+ bn + c oraz niech k, k + 1, . . ., k + 2p 2 bd
2p1 kolejnymi liczbami naturalnymi, dla ktrych f przyjmuje wartoci
bdce kwadratami liczb cakowitych.
Zacznijmy od zbadania dla jakich x, y Z zachodzi f(x) f(y)
(mod p). Pytamy wic kiedy speniona jest kongruencja
ax
2
+bx +c ay
2
+by +c (mod p),
lub rwnowanie
(x y)(ax +ay) +b(x y) 0 (mod p),
czyli
(x y)(a(x +y) +b) 0 (mod p).
Wynika std, e f(x) f(y) (mod p), wtedy i tylko wtedy, gdy x y
(mod p) lub
ay ax b (mod p). (1)
Nietrudno zauway, e jeli a , 0 (mod p), to przy danym x istnieje
dokadnie jedno rozwizanie kongruencji (1) mod p. W szczeglnoci nie
istniej liczby cakowite x, y, z dajce parami rne reszty z dzielenia
przez p, dla ktrych f(x) f(y) f(z) (mod p). Jeli natomiast a 0
(mod p), to owa kongruencja posiada rozwizanie wtedy i tylko wtedy,
gdy rwnie b 0 (mod p).
Jak wiadomo wszystkich niezerowych reszt kwadratowych modulo p
jest dokadnie
p1
2
. Z treci zadania wynika natychmiast, e reszty dziele-
nia liczb f(k), f(k+1), . . . , f(k+p1) s niezerowymi resztami kwadra-
towymi modulo p lub s podzielne przez p. Zauwamy jednak, e tych
liczb jest p, a moliwych reszt
p+1
2
, skd wynika, e pewna reszta jest
przyjmowana co najmniej 2 razy. W szczeglnoci jeli p[a, to p[b, gdy
kongruencja (1) posiada rozwizanie. Ale wwczas p[b
2
4ac, czyli teza
jest speniona. Zamy wic, e a , 0 (mod p). Jasne jest te, e ktra
z wartoci f(k), . . . , f(k + p 1) musi by podzielna przez p, gdy in-
aczej pewna reszta kwadratowa byaby przyjmowana 3 razy, co jak za-
uwaylimy wczeniej, jest niemoliwe.
Zamy najpierw, e istniej dwie liczby k t < s k + p 1, dla
ktrych p[f(t) oraz p[f(s). Z faktu t k+p2 wynika, e t+p k+2p2,
a wic f(t +p) rwnie jest kwadratem i jasne jest, e p[f(t +p). Liczby
31
podzielne przez liczb pierwsz p: f(t), f(t + p) s kwadratami liczb
naturalnych, a wic s rwnie podzielne przez p
2
. Mamy wic
p
2
[f(t+p)f(t) = a(t+p)
2
+b(t+p)+c(at
2
+bt+c) = 2atp+ap
2
+bp,
skd wynika, e p[2at +b. Czyli
p[4af(t) = (2at +b)
2
(b
2
4ac),
czyli te p[b
2
4ac i w tym przypadku zadanie jest rozwizane.
Zamy teraz, e wrd liczb k, k + 1, . . . , k + p 1 jest dokadnie
jedna liczba t, dla ktrej p[f(t). Jeli jednak wemiemy z tych liczb tak
liczb s, ktra jest rozwizaniem kongruencji (1) przy ustalonym x = t,
to wwczas p[f(s). Musi by zatem s t (mod p), czyli wstawiajc do
kongruencji (1) x = y = t dostajemy
p[2at +b,
co w poczeniu z p[f(t) daje, jak widzielimy wczeniej, p[b
2
4ac i
dowd jest zakoczony.
16. Dana jest liczba naturalna n , 2. Tablica n n jest wypeniona
liczbami 0 i 1 w ten sposb, e kady podzbir n pl, z ktrych adne
2 nie le w jednej kolumnie ani w jednym wierszu zawiera co najmniej
jedno pole z liczb 1. Dowie, e istnieje i wierszy i j kolumn, gdzie
i +j , n + 1, takich, e na przeciciu kadego wiersza i kadej kolumny
jest liczba 1.
Rozwizanie
Do rozwizania tego zadania zastosujemy twierdzenie Halla (tu zapisane
w terminologii chopcw i dziewczynek)
Twierdzenie Halla
Na tacach jest n chopcw i pewna liczba dziewczynek. Mwimy, e
speniony jest warunek Halla, jeeli dla kadej grupy k chopcw, gdzie
1 k n istnieje przynajmniej k dziewczynek takich, e ktry z
tych chopcw j zna. Jeeli speniony jest warunek Halla, to wwczas
kademu chopcu moemy przydzieli do pary dziewczynk, ktr zna.
Musimy teraz jedynie wykona prac typow dla zada na twierdze-
nie Halla, czyli zinterpretowa co jest chopcem, co jest dziewczynka i
32
co jest znajomoci. W tym przypadku niech chopcy to bd wiersze,
dziewczynki to kolumny (bd odwrotnie). Powiemy, e wiersz i kolumna
znaj si, jeeli pole na ich przeciciu zawiera 0.
Warunek, e dla dowolnego podzbioru n pl z rnych wierszy i rnych
kolumn ktre z nich posiada 1 jest to dokadnie powiedzenie, e dla
dowolnego sparowania wszystkich chopcw ze wszystkimi dziewczynkami
ktra para nie bdzie si znaa.
Teza zadania mwi, e istnieje pewne i wierszy (chopcw) takich, e
istnieje dla nich j , n + 1 i kolumn (dziewczynek) takich, e adna z
tych kolumn nie jest znana przez aden wiersz. Czyli innymi sowy teza
zadania mwi, e istnieje pewna i-elementowa grupa chopcw, ktra zna
jedynie co najwyej n (n + 1 i) = i 1 dziewczynek. Jest to wic
dokadnie zaprzeczenie warunku Halla.
Zatem zadanie jest jedynie przeformuowaniem twierdzenia Halla, mwi
ono, e z zaprzeczenia tezy tego twierdzenia wynika zaprzeczenia zaoe-
nia, czyli dysponujc twierdzeniem Halla wykazalimy tez zadania.
17. Udowodni, e istnieje nieskoczenie wiele liczb naturalnych n
takich, e suma cyfr liczby 3
n
jest nie mniejsza ni suma cyfr liczby 3
n+1
.
Rozwizanie
Udowodnimy tez zadania nie wprost. Oznaczmy przez S(n) sum cyfr
liczby n. Przypumy, e tylko dla skoczenie wielu liczby naturalnych
n zachodzi S(3
n
) , S(3
n+1
). Wwczas oprcz pewnych k liczby dla
wszystkich liczb naturalnych zachodzi S(3
n+1
) > S(3
n
). Zauwamy jed-
nak, e dla n , 2 zachodzi 9 [ 3
n
, a co za tym idzie 9 [ S(3
n
). Za-
tem skoro S(3
n+1
) > S(3
n
), to S(3
n+1
) , 9 + S(3
n
). Zatem S(3
n
) ,
9(nk2)+S(9) = 9(nk1). Jednak liczba 3
2n
ma co najwyej n cyfr,
a co za tym idzie S(3
2n
) 9n. A zatem 9n , S(3
2n
) , 9(2n k 1), co
dla dowolnego k przy dostatecznie duym n nie jest prawd. Sprzeczno.
18. Dany jest wielomian stopnia n speniajcy zaleno P(i) = 2
i
dla
i = 0, 1, 2, . . . , n. Wyznaczy P(n + 1).
Rozwizanie
Sposb I
Przez P
n
oznaczmy wielomian stopnia co najwyej n i dla i = 0, 1, . . . , n
speniajcy wasno z treci zadania. Niech Q
n
(x) = P
n
(x +1) P
n
(x).
Wwczas Q
n
(i) = P
n
(i + 1) P
n
(i) = 2
i+1
2
i
= 2
i
dla i = 0, 1, . . . , n
33
1. Co wicej, Q
n
jest wielomianem stopnia co najwyej n 1, gdy
wspczynniki przy najwyszej potdze P
n
redukuj si. Poniewa istnieje
tylko jeden wielomian stopnia co najwyej n1 o ustalonych wartociach
w n punktach, to Q
n
= P
n1
.
Udowodnimy teraz przez indukcj, e P
n
(n + 1) = 2
n+1
1. Dla n = 0
wielomian P
0
jest wielomianem stopnia 0 takim, e P
0
(0) = 1, czyli
P
0
(x) = 1, a co za tym idzie istotnie P
0
(1) = 1 = 2
1
1.
Zamy teraz, e P
n
(n+1) = 2
n+1
1. Przypomnijmy, e dla dowolnego
n zachodzi Q
n
= P
n1
, a zatem P
n
= Q
n+1
. Wynika z tego, e
2
n+1
1 = P
n
(n + 1) = Q
n+1
(n + 1)
= P
n+1
(n + 2) P
n+1
(n + 1) = P
n+1
(n + 2) 2
n+1
,
czyli P
n+1
(n + 2) = 2
n+2
1. Tym samym wykazany zosta krok induk-
cyjny.
Odpowied P(n + 1) = 2
n+1
1.
Sposb II
Mona skorzysta ze wzoru interpolacyjnego Lagrangea, ktry podaje w
sposb jawny wielomian stopnia co najwyej n o ustalonych wartociach
w n+1 punktach. Jeli wielomian W co najwyej n w punktach x
0
, . . . , x
n
posiada odpowiednio wartoci a
0
, . . . , a
n
, to jest on postaci
W(x) =
n

i=0
a
i

j=i
x x
j
x
i
x
j
.
Korzystajc ze wzoru Lagrangea otrzymujemy, e
P(n + 1) =
n

i=0
2
i

j=i
n + 1 j
i j
=
n

i=0
2
i

j=i
(n + 1 j)

j=i
i j
=
n

i=0
2
i
(n + 1)!
(n + 1 i)i!(n i)!(1)
ni
=
n

i=0
2
i
(1)
ni
_
n + 1
i
_
=
n+1

i=0
2
i
(1)
n+1i
_
n + 1
i
_
+ 2
n+1
_
n + 1
n + 1
_
= (2 1)
n+1
+ 2
n+1
= 2
n+1
1.
34
19. Na paszczynie dany jest wielokt W o polu wikszym od n.
Udowodni, e mona tak przesun rwnolegle wielokt W, e w jego
wntrzu znajdzie si co najmniej n + 1 punktw kratowych.
Rozwizanie
Niech S bdzie zbiorem wszystkich kwadratw jednostkowych o wierz-
chokach w punktach kratowych, przecinajcych wielokt W. Jeeli zsumu-
jemy dla kadego takiego kwadratu jednostkowego K pole czci KW,
to suma ta bdzie rwna polu W, czyli wiksza ni n. Zatem jeeli
naoymy wszystkie kwadraty na jedno miejsce, to z zasady szuad-
kowej Dirichleta w wersji polowej wynika, e istnieje punkt, ktry bdzie
pokryty przez gury KW co najmniej n+1 razy. Oznaczmy ten punkt
przez A = (a
1
, a
2
), gdzie 0 a
1
, a
2
1. Zatem wrd punktw postaci
(k + a
1
, l + a
2
), gdzie k, l Z przynajmniej n + 1 naley do wntrza
wielokta W. Zatem jeli W przesuniemy o wektor v = (a
1
, a
2
), to
przynajmniej n + 1 z punktw postaci (k, l) takich, e k, l Z, czyli
punktw kratowych bdzie naleao do (przesunitego) wielokta W.
20. Niech KL i KN bd stycznymi do okrgu k w punktach L i N.
M jest takim punktem, e K, N, M le na jednej prostej w tej wanie
kolejnoci. Okrg opisany na trjkcie KLM przecina okrg k w punkcie
P. Punkt Q jest rzutem prostopadym N na prost ML. Udowodni, e
MPQ = 2KML.
Rozwizanie
Oznaczmy przez R drugi punkt przecicia okrgu k z prost LM. Za-
chodz rwnoci:
MPK = MLK = RLK = RPL.
W takim razie MPR = KPL = KML. Wobec tej rwnoci okrg
przechodzcy przez punkty M, P, R jest styczny do prostej KM. Oz-
naczmy przez S rodek odcinka MN. Punkt ten ley na osi potgowej
okrgu k i okrgu opisanego na trjkcie MPR, czyli na prostej PR.
Trjkt MSQ jest rwnoramienny, czyli MQS = QMS = MPS i
punkty M, P, Q, S le na okrgu. W takim razie:
MPQ = MPS+SPQ = MQS+SMQ = 2SMQ = 2KML.
35
L
N
K
M
P
Q
R
S
21.Ahmed i Fredek graj w gr na szachownicy n n, gdzie n jest
liczb nieparzyst. Ahmed stawia kka, a Fredek krzyyki. Na pocztku
wszystkie pola s puste, tylko w lewym dolnym rogu jest kko, a w
prawym grnym jest krzyyk. Zaczyna Ahmed. Ruch gracza polega na
postawieniu swojego znaczka na wolnym polu ssiadujcym przez krawd
z polem, na ktrym jest ju postawiony jego znaczek. Gdy gracz nie moe
wykona ruchu, to traci go. Gra koczy si, gdy aden z graczy nie moe
wykona ruchu. Gr wygrywa ten gracz, ktry wykona wicej ruchw.
Rozstrzygnij, ktry z graczy posiada strategi wygrywajc.
Rozwizanie
Poniewa gra jest skoczona i kada rozgrywka koczy si wygran ktre-
go z graczy, to istnieje gracz posiadajcy strategi wygrywajc. Udowod-
nimy, e tak strategi posiada gracz zaczynajcy, Ahmed, bdzie to
dowd niekonstruktywny.
Przypumy, nie wprost, e strategi wygrywajc posiada Fredek. Wska-
emy teraz strategi dla Ahmeda, ktr grajc przeciw Fredkowi moe
z nim wygra. Zrbmy na pocztek jedno spostrzeenie - wasny znak
nigdy nie przeszkadza graczowi w prowadzeniu jakiejkolwiek strategii.
36
Ahmed stawia na pocztek kko gdziekolwiek. Nastpnie stawia si w
sytuacji drugiego gracza, zapomina o postawionym kku i gra wygrywa-
jc strategi drugiego gracza. Jeeli strategia ta kae postawi kko w
tym miejscu, gdzie stoi pierwsze kko, to, poniewa kko ju tam jest,
to Ahmed stawia kko w dowolnym innym miejscu (i zapomina o nim).
W ten sposb pokazalimy, e Ahmed, kopiujc strategi drugiego gracza
moe wygra, czyli doszlimy do sprzecznoci z zaoeniem, e Fredek ma
strategi wygrywajc.
Zatem strategi wygrywajc posiada Ahmed.
22.Dany jest wielomian P stopnia n , 2 o wspczynnikach cakow-
itych dodatnich a
n
, a
n1
, . . . , a
1
, a
0
speniajcych warunki: a
nk
= a
k
dla kadego k = 1, 2, . . . , n1 oraz a
n
= a
0
= 1. Udowodni, e istnieje
nieskoczenie wiele par liczb cakowitych a, b takich, e a[P(b) i b[P(a).
Rozwizanie
Zauwamy, e (a, b) = (P(1), 1) spenia warunki zadania oraz P(1) > 1.
Przypumy, e para (a, b), a > b spenia warunki zadania. Pokaemy,
e wwczas para (a,
P(a)
b
) rwnie spenia warunki zadania, a co wicej
P(a)
b
> a > b. Jeeli to udowodnimy, to teza zadania zostanie pokazana,
gdy z dowolnej pary da si stworzy inn speniajc warunki zadania i
to dodatkowo o wikszej sumie.
Oczywicie
P(a)
b
[ P(a).
Aby pokaza, e b [ P(
P(a)
b
) zauwamy najpierw, e skoro a
0
= 1, to
P(a) = ak + 1 dla pewnego k Z. Zatem
P(
P(a)
b
) = P(
ak + 1
b
) = a
n
(ak + 1)
n
b
n
+a
n1
(ak + 1)
n1
b
n1
+. . .
+a
1
ak + 1
b
+a
0
= C +a
n
_
1
b
_
n
+a
n1
_
1
b
_
n1
+. . . +a
1
1
b
+a
0
,
gdzie C jest liczb wymiern, ktra zapisana w postaci nieskracalnej
wynosi
as
t
, gdzie s, t Z oraz t [ b
n
. Wynika to std, e C jest sum
skadnikw postaci a
m
(
m
l
)
(ak)
l
b
m
dla l , 1 oraz z faktu, e a i b s wzgldnie
pierwsze, bowiem b [ P(a) = ak + 1 dla pewnego k Z.
Dla wygody bdziemy teraz operowa na uamkach modulo k, powiemy,
e
a
b

c
d
(mod k) dla b oraz d wzgldnie pierwszych z k jeeli ad bc
(mod k). Nietrudno sprawdzi, e kadej liczbie wymiernej o mianowniku
wzgldnie pierwszym z k odpowiada dokadnie jedna reszta modulo k.
37
Mamy wic C = a
s
t
0 (mod a), gdy t jest wzgldnie pierwsze z a
jako, e t [ b
n
. Zatem
P(
P(a)
b
) = C +a
n
_
1
b
_
n
+a
n1
_
1
b
_
n1
+. . . +a
1
1
b
a
n
_
1
b
_
n
+a
n1
_
1
b
_
n1
+. . . +a
1
1
b
= P(
1
b
) = P(b)
1
b
n
0 (mod a),
gdzie przedostatnia rwno wynika z tego, e wspczynniki P speniaj
a
nk
= a
k
. Jeli zachodzi
p
q
0 (mod k), to oczywicie rwnie p 0
(mod k), czyli para (a,
P(a)
b
) spenia zaoenia zadania.
Poniewa stopie P jest nie mniejszy ni 2 oraz wspczynniki s cakowite
dodatnie, to
P(a)
b
>
P(a)
a
> a > b, co pokazuje, e faktycznie suma pary
si zwikszya oraz, e wyrazy w parze s rne.
23. Punkt I jest rodkiem okrgu wpisanego w trjkt ABC, a D
punktem stycznoci tego okrgu z bokiem BC. Okrg jest styczny
do prostej BC w punkcie D a do okrgu opisanego na trjkcie ABC w
punkcie T, przy czym A i T le po jednej stronie prostej BC. Dowie,
e ATI = 90

.
Rozwizanie
Udowodnimy nastpujcy lemat:
Lemat. Okrg o znajduje si we wntrzu okrgu O i jest do niego styczny
wewntrznie w punkcie T. Punkty A i B, rne od punktu T, le na
okrgu O, a proste AK i BL s stycznymi do okrgu o, gdzie K i L le
na o. Wwczas
TA
TB
=
AK
BL
.
Oznaczmy przez A

, B

drugie punkty przecicia z okrgiemo odpowied-


nio prostych TA, TB. Punkt T jest rodkiem jednokadnoci przekszta-
cajcej okrg o na O, w ktrej punkt A

przechodzi na A, a punkt B

przechodzi na B. Zatem proste A

oraz AB s rwnolege, a wic na


mocy Tw. Talesa otrzymujemy
TA
TB
=
AA

BB

.
Natomiast z potgi punktu wzgldem okrgu o dostajemy rwnoci AA

AT = AK
2
, BB

BT = BL
2
. Po pomnoeniu wszystkich trzech rwnoci
stronami otrzymujemy tez lematu.
Niech E, F oznaczaj punkty stycznoci okrgu wpisanego odpowied-
nio z bokami AB, AC. Z udowodnionego powyej lematu oraz rwnoci
38
BE = BD i CF = CD dostajemy
BT
CT
=
BD
CD
=
BE
CF
.
B
C
A
T
B
A
F
E
B
A
I
D
E
F
T
Jednoczenie EBT = ABT = ACT = FCT, a zatem trjkty
EBT oraz FCT s podobne. Wynika std, e AET = AFT, czyli
punkty A, E, I, F, T le na jednym okrgu o rednicy AI i oczywicie
ATI = 90

.
24. Dane s dodatnie liczby rzeczywiste x
1
, x
2
, . . . , x
n
takie, e
n

i=1
x
i
= 1. Udowodni nierwno:
n

i=1
x
i

1 +x
1
+ +x
i1

x
i
+x
i+1
+ +x
n
<
1 +

5
2
Rozwizanie
Niech 0 =
0

1
. . .
n
=

2
bd takimi liczbami rzeczywistymi,
e sin
i
= x
1
+x
2
+. . .+x
i
. Wtedy x
i
= sin
i
sin
i1
dla i = 1, 2, . . . , n.
Nierwno przyjmuje posta:
n1

i=0
sin
i+1
sin
i

1 + sin
i

1 sin
i
<
1 +

5
2
.
39
Udowodnimy nierwno mocniejsz:
n1

i=0
sin
i+1
sin
i
cos
i
<

2
.
Mamy bowiem:
sin
i+1
sin
i
cos
i
=
2 sin

i+1

i
2
cos

i+1
+
i
2
cos
i
=
= (
i+1

i
)
sin

i+1

i
2

i+1

i
2

cos

i+1
+
i
2
cos
i
< (
i+1

i
),
bo dla dodatnich x jest sin x < x oraz funkcja x cos x jest malejca.
W takim razie
n1

i=0
sin
i+1
sin
i
cos
i
<
n

0
=

2
.
Ponadto (2, 2)
2
= 4, 84 < 5, czyli 1 +

5 > 3, 2 > , co koczy dowd.


25. W trjkcie ABC punkt D jest spodkiem wysokoci poprowad-
zonej z punktu A. Na pewnej prostej przechodzcej przez D wybrano
takie punkty E i F, rne od D, e AEB = AFC = 90

. Punkty M i
N s odpowiednio rodkami odcinkw BC i EF. Dowie, e ANM =
90

.
Rozwizanie
Skoro AEB = ADB = 90

, to punkty A, D, B, E le na jednym
okrgu o rednicy AB. Analogicznie punkty A, D, C, F le na jednym
okrgu o rednicy AC. Std AEF = ABC oraz AFE = ACB
jako kty oparte na tych samych ukach, czyli trjkty ABC i AEF s
podobne. W takim razie
AN
AM
=
AE
AB
oraz MAN = EAB, co daje
podobiestwo trjktw MAN i EAB i tym samym koczy dowd.
26.Dane jest 2n parami rnych liczb rzeczywistych a
1
, a
2
, . . . , a
n
,
b
1
, b
2
, . . . , b
n
oraz tablica n n. W pole lece w i-tym wierszu i w j-
tej kolumnie tablicy wpisano liczb a
i
+ b
j
. Udowodni, e jeli iloczyny
liczb we wszystkich kolumnach s rwne, to rwnie iloczyny liczb we
wszystkich wierszach s rwne.
40
Rozwizanie
Rwno iloczynw liczb w kolumnach znaczy dokadnie, e wyraenia
postaci
(b
j
+a
1
) (b
j
+a
2
) . . . (b
j
+a
n
)
s rwne dla dowolnego 1 j n. Zdeniujmy wielomian P(x) nastpu-
jco
P(x) = (x +a
1
)(x +a
2
) . . . (x +a
n
).
Wwczas P(b
1
) = P(b
2
) = . . . = P(b
n
) = C dla pewnego C R. Wielo-
mian P(x)C jest unormowany, n tego stopnia oraz zeruje si w liczbach
b
1
, b
2
, . . . , b
n
, a wic jest postaci
P(x) C = (x b
1
)(x b
2
) . . . (x b
n
).
Poniewa
(x +a
1
)(x +a
2
) . . . (x +a
n
) C = (x b
1
)(x b
2
) . . . (x b
n
),
to rwnie
(x a
1
)(x a
2
) . . . (x a
n
) (1)
n
C = (x +b
1
)(x +b
2
) . . . (x +b
n
),
gdy po prostu zmienilimy znaki przy wyrazach x
k
, gdzie 2 [ n k.
Podstawiajc do powyszej rwnoci a
j
dla 1 j n otrzymujemy, e
0 (1)
n
C = (a
j
+b
1
)(a
j
+b
2
) . . . (a
j
+b
n
),
czyli
(a
j
+b
1
)(a
j
+b
2
) . . . (a
j
+b
n
) = (1)
n+1
C.
Ostatnia rwno oznacza dokadnie, e iloczyny liczb w kadym wierszu
s rwne i wynosz (1)
n+1
C.
27.W pola tablicy nn wpisano wszystkie liczby naturalne od 1 do n
2
.
Udowodni, e istniej dwa pola ssiadujce krawdzi takie, e wpisane
w nie liczby rni si o co najmniej n.
Rozwizanie
Przeprowadzimy dowd nie wprost. Przypumy, e dla dowolnych dwch
ssiadujcych pl wpisane w nie liczby rni si o co najwyej n 1.
Zdeniujmy zbiory A
k
= 1, 2, . . . , k, B
k
= k + 1, . . . , k + n 1,
C
k
= k + n, . . . , n
2
dla k = 1, 2, . . . , n
2
n. Zauwamy, e dla dowol-
nego k dowolny element ze zbioru A
k
nie moe ssiadowa z dowolnym
41
elementem ze zbioru C
k
, zbir B
k
jest wic zbiorem granicznym dla
zbiorw A
k
i C
k
.
Poniewa [B
k
[ < n, to dla dowolnego k musz istnie wiersz i kolumna
obie w caoci nalece do A
k
lub do C
k
. Dla k = 1 nale one do C
k
, dla
k = n
2
n do A
k
. Niech m bdzie najmniejszym indeksem takim, e A
m
zawiera cay wiersz i ca kolumn. Wwczas C
m1
musi zawiera cay
wiersz i ca kolumn. Zatem A
m
oraz C
m1
przecinaj si w co najm-
niej dwch miejscach, co jest niemoliwe, gdy nie maj one elementw
wsplnych. Sprzeczno.
28. Dana jest liczba naturalna n , 3. Udowodni, e liczba 2
2
n
+ 1
posiada dzielnik pierwszy, ktry jest wikszy ni 2
n+2
(n + 1).
Rozwizanie
Niech p bdzie dowolnym dzielnikiem pierwszym liczby 2
2
n
+1, a k niech
bdzie rzdem liczby 2 modulo p, tzn. najmniejsz liczb cakowit do-
datni, dla ktrej 2
k
1 (mod p). Poniewa 2
2
n
1 , 1 (mod p) k
nie dzieli 2
n
, ale jednoczenie 2
2
n+1
1 (mod p), czyli k dzieli 2
n+1
. Std
k = 2
n+1
. Z Maego Twierdzenia Fermata wiadomo rwnie, e 2
p1
1
(mod p), a std wnioskujemy, i k[p 1, czyli 2
n+1
[p 1.
Niech 2
2
n
+1 = p
1
p
2
. . . p
m
bdzie rozkadem liczby 2
2
n
+1 na niekoniecznie
rne czynniki pierwsze. Na mocy powyszej obserwacji dla kadego i =
1, 2, . . . , m istnieje k
i
takie, e p
i
= 2
n+1
k
i
+1. Wystarczy, e wykaemy,
e dla pewnego i zachodzi k
i
, 2(n + 1).
Najpierw oszacujemy m od gry. Zauwamy, e
2
2
n
+1 = p
1
p
2
. . . p
m
= (2
n+1
k
1
+1)(2
n+1
k
2
+1) . . . (2
n+1
k
m
+1) , 2
(n+1)m
+1,
a std otrzymujemy nierwno m
2
n
n + 1
.
Znajdziemy teraz oszacowanie dolne na sum k
1
+k
2
+. . . +k
m
. Poniewa
n , 3 atwo pokaza, e 2
n
, 2n + 2. Co za tym idzie
1 2
2
n
+ 1 (2
n+1
k
1
+ 1)(2
n+1
k
2
+ 1) . . . (2
n+1
k
m
+ 1)
2
n+1
(k
1
+k
2
+. . . +k
m
) + 1 (mod 2
2n+2
),
gdzie trzecia kongruencja bierze si std, e po otworzeniu nawiasw
reszta wyrazw redukuje si modulo 2
2n+2
. Zatem zachodzi podzielno
2
n+1
[k
1
+k
2
+. . . k
m
, czyli w szczeglnoci k
1
+k
2
+. . . +k
m
, 2
n+1
.
42
Niech k
i
bdzie maksymaln spord liczb k
1
, k
2
, . . . , k
m
. Wwczas
2
n+1
k
1
+k
2
+. . . +k
m
mk
i

2
n
n + 1
k
i
skd wynika, e k
i
, 2(n + 1) i rozwizanie zadania jest zakoczone.
29. Liczby dodatnie a, b, x, y speniaj rwnoci a
2
+x = b
2
+y oraz
a +x
2
= b +y
2
, a take nierwno a +b +x +y < 2. Dowie, e a = b
oraz x = y.
Rozwizanie
Mamy:
ab = y
2
x
2
= (y x)(y +x) = (a
2
b
2
)(x+y) = (ab)(a+b)(x+y)
oraz
(a +b)(x +y)
_
a +b +x +y
2
_
2
< 1,
wic a b = 0. Analogicznie x y = 0.
30. Rozstrzygn czy istniej parami wzgldnie pierwsze liczby nat-
uralne a, b, c > 1, dla ktrych zachodz warunki: a[2
b
+ 1, b[2
c
+ 1,
c[2
a
+ 1.
Rozwizanie
Takie liczby nie istniej. Zamy przeciwnie i przyjmijmy przy tym,
e spord wszystkich trjek liczb, ktre speniaj dane warunki trjka
(a, b, c) posiada minimaln sum. Bez straty oglnoci moemy przyj,
e a jest najmniejsz spord liczb a, b, c. Niech k bdzie rzdem liczby 2
modulo a, tzn. najmniejsz liczb cakowit dodatni, dla ktrej 2
k
1
(mod a). Wiadomo, i 2
b
1 , 1 (mod a) oraz 2
2b
1 (mod a),
czyli k ,[b i k[2b. Oznacza to, e k = 2l gdzie l[b. Pokaemy teraz, e
trjka (a, l, c) rwnie spenia dane warunki. W tym celu zauwamy,
e a[2
l
+ 1. Rzeczywicie, poniewa k = 2l jest rzdem liczby 2 modulo
a zachodzi podzielno a[2
2l
1 = (2
l
1)(2
l
+ 1). Jeli liczby a i 2
l
1
posiadaaby jaki wsplny dzielnik pierwszy p, to wtedy te p[2
b
1, gdy
l[b, a jednoczenie p[2
b
+ 1 poniewa a[2
b
+ 1, co oznacza, e p = 2, a to
jest niemoliwe. Zatem liczby a i 2
l
1 s wzgldnie pierwsze i a[2
l
+ 1.
Jasne jest teraz, e l ,= 1, bowiem w przeciwnym razie a = 3 oraz c[9
co jest sprzeczne z warunkami zadania. Ponadto l, jako dzielnik liczby b,
43
jest wzgldnie pierwsze z a i c oraz l[2
c
+ 1. A wic rzeczywicie trjka
(a, l, c) spenia warunki dane w zadaniu. Jednak l < 2l (a) < a, a
std
a +l +c < a +a +c < a +b +c
co stoi w sprzecznoci z zaoeniem o minimalnoci sumy trjki (a, b, c).
31.Sfery opisana oraz wpisana w czworocian ABCD maj wsplny
rodek, AB = CD oraz wszystkie ciany tego czworocianu s trjktami
ostroktnymi. Udowodni, e rodki krawdzi czworocianu ABCD le
na jednej sferze wtedy i tylko wtedy, gdy jest on foremny.
Rozwizanie
Oznaczmy przez S wsplny rodek kuli opisanej i wpisanej w czworo-
cian ABCD, przez R i r odpowiednio promienie tych kul, za przez
S
A
, S
B
, S
C
, S
D
rzuty S odpowiednio na ciany BCD, ACD, ABD i ABC.
Z twierdzenia Pitagorasa otrzymujemy AS
2
D
= R
2
r
2
= BS
2
D
= CS
2
D
,
czyli punkt S
D
jest rodkiem okrgu opisanego na trjkcie ABC. Postpu-
jc analogicznie dla innych cian otrzymujemy, e S
A
, S
B
i S
C
s rwnie
rodkami okrgw odpowiednich cian i promienie wszystkich tych okrgw
s rwne i wynosz

R
2
r
2
.
Poniewa ciany s trjktami ostroktnymi, to rodki okrgw opisa-
nych le wewntrz tych cian. Zauwamy, e promie okrgu opisanego
na trjkcie oraz dugoci dwch kolejnych bokw jednoznacznie deter-
minuj trjkt. Pokaemy, e ciany ABC oraz BCD s przystajce.
Promie okrgu opisanego na obu cianach jest rwny, za bokom AB
i BC w trjkcie ABC odpowiadaj w trjkcie BCD boki CD i BC
o tych samych dugociach. Analogicznie wykazujemy przystawanie po-
zostaych cian otrzymujc, e wszystkie ciany czworocianu ABCD s
przystajce.
Dowolny czworocian mona wpisa w rwnolegocian tak, by wierz-
choki czworocianu pokryway si z pewnymi wierzchokami rwnolego-
cianu tak, e adne dwa wierzchoki rwnolegocianu bdce kocem jed-
nej krawdzi nie s wierzchokami czworocianu. Jeeli czworocian ten
ma przystajce ciany, to rwnolegocian jest prostopadocianem, gdy
za jest foremny, to prostopadocian ten jest szecianem. W tym momen-
cie teza zadania sprowadza si do faktu, e rodki cian prostopadocianu
le na jednej sferze wtedy i tylko wtedy, gdy prostopadocian ten jest
szecianem - co jest oczywiste.
44
32. Udowodni, e istnieje dokadnie jeden podzia zbioru liczb nat-
uralnych na rozczne zbiory A i B speniajcy warunek: dla dowolnej
liczby naturalnej n liczba sposobw zapisania n w postaci a
i
+ a
j
, gdzie
a
i
, a
j
A i a
i
,= a
j
jest rwna liczbie sposobw zapisania n w postaci
b
i
+b
j
, gdzie b
i
, b
j
B i b
i
,= b
j
.
Rozwizanie
Rozwamy funkcje tworzce:
A(x) =

nA
x
n
, B(x) =

nB
x
n
.
Z treci zadania wynikaj rwnoci
A(x) +B(x) =

n=0
x
n
=
1
1 x
,
(A(x))
2
A(x
2
) = (B(x))
2
B(x
2
).
Mamy w takim razie:
A(x) B(x) =
(A(x))
2
(B(x))
2
A(x) +B(x)
= (1 x)(A(x
2
) B(x
2
))
i przez indukcj:
A(x) B(x) =
n1

k=0
(1 x
2
k
) (A(x
2
n
) B(x
2
n
)).
Przypumy , e 0 A. Niech m bdzie dowoln liczb naturaln i wemy
takie n naturalne, eby 2
n
> m. Z powyszej rwnoci wida, e wsp-
czynnik przy x
m
w wyraeniu A(x) B(x) jest taki sam jak w wyraeniu

n1
k=0
(1 x
2
k
), a wic rwny 1, gdy m ma parzyst liczb jedynek w
zapisie dwjkowym lub 1 w przeciwnym wypadku. atwo zobaczy, e
dla
A(x) =
1
2
(
1
1 x
+

k=0
(1 x
2
k
)), B(x) =
1
2
(
1
1 x

k=0
(1 x
2
k
))
powysze rwnoci s spenione, a wic istotnie generuj one jedyny
podzia o zadanych wasnociach: A = a : a ma parzyst liczb je-
dynek w zapisie dwjkowym, B = N A.
45
Dzie Dziecka:
1. W zaczniku do zada masz aktualn kopi punktacji poszczegl-
nych uczestnikw obozu. Dla cigu rzeczywistych liczb a
1
, a
2
, . . . a
8
zmody-
kowanym wynikiem zawodnika nazywamy liczb

8
i=1
a
i
P
i
, gdzie P
i
to
liczba punktw, ktre ten zawodnik zdoby z itego zadania. Twoim
zadaniem jest tak dobra wagi a
i
, aby by liderem rankingu zmody-
kowanych wynikw ex aequo z jak najmniejsz liczb innych zawod-
nikw.
Rozwizanie
Zamiemy w treci zadania 8 na n, aby uoglni sytuacj. Punktacja
zawodnika jest punktem (p
1
, p
2
, . . . , p
n
) w przestrzeni R
n
. Chcemy po-
prowadzi paszczyzn o rwnaniu a
1
x
1
+ a
2
x
2
+ . . . + a
n
x
n
=

n
i=1
a
i
p
i
tak, aby wszystkie punkty leay po jednej jej stronie lub na niej, przy
czym na niej jak najmniej. Zauwamy, e jeeli punkt p jest postaci
p =

n
i=1

i
p
(i)
(), gdzie

k
i=1

i
= 1 i
i
> 0 dla i = 1, 2, . . . , k, to
punkty p
(1)
, p
(2)
, . . . , p
(k)
musz lee na tej paszczynie, bo wtedy mamy

k
i=1
(
i

n
j=1
a
j
p
(i)
j
) =

k
i=1
(
i

n
j=1
a
j
p
j
) i gdyby jeden ze skadnikw po
lewej stronie by mniejszy ni po prawej, to inny musiaby by wikszy
- sprzeczno. Zauwamy, e warunek (*) oznacza, e p ley wewntrz
otoczki wypukej zbioru p
(1)
, p
(2)
, . . . , p
(k)
. Przypumy teraz, e tak
nie jest i bez straty oglnoci przyjmijmy p = 0. Wtedy p ley na brzegu
otoczki wypukej p
(1)
, p
(2)
, . . . , p
(k)
- wielocianu n-wymiarowego. W
takim razie ley na jakiej cianie (n1)-wymiarowej. Wiadomo, e cay
wielocian ley po jednej stronie hiperpaszczyzny wyznaczonej przez t
cian. Ograniczylimy zbir rozwaanych punktw do tych lecych na
tej cianie. Zejdmy wymiar niej, na t hiperpaszczyzn o rwnaniu
a, x) = 0. Powtarzamy rozumowanie, jeeli jestemy w l wymiarach i 0
jest na brzegu otoczki wypukej, to jest na cianie (l 1)-wymiarowej;
wemy bardzo may wektor e lecy w tej paszczynie l-wymiarowej
i prostopady do paszczyzny (l 1)-wymiarowej i nie nalecy do tej
ciany i podstawmy a := a+e. To spowoduje, e hiperpaszczyzna (n1)-
wymiarowa lekko si obrci, wyleci z niej ta ciana l-wymiarowa, a po-
zostanie ta (l 1)-wymiarowa. e mona dobra tak may, eby to nie
popsuo leenia wielocianu po jednej stronie hiperpaszczyzny. W kocu
dojdziemy do momentu, gdy 0 bdzie wewntrz otoczki wypukej jakich
punktw w l wymiarach (by moe l = 0) i wtedy to postpowanie si
46
koczy, bo 0 jest postaci (*).
Uwaga
Zbiegiem okolicznoci kady uczestnik mg uczyniiebie samodzielnym
liderem, oprcz dwch, ktrzy prowadzili ex aequo, bo mieli tak sam
liczb punktw za kade zadanie.
2. Znajd wszystkie pary liczb cakowitych a, b takie, e a
3
= 6b
2
+2.
Rozwizanie
Mamy a
3
= 6b
2
+2 = (1+b)
3
+(1b)
3
. Z Wielkiego Twierdzenia Fermata
wynika, e a = 0, b +1 = 0 lub b 1 = 0. W takim razie pary speniajce
to rwnanie to (2, 1) i (2, 1).
3. Niech ABC bdzie trjktem rwnobocznym o polu 1, za P dowol-
nym punktem w jego wntrzu. Przez D, E, F oznaczamy rzuty pros-
toktne P odpowiednio na BC, CA i AB. Znajd najmniejsz moliw
sum pl trjktw BDP, CEP i FAP.
Rozwizanie
Zamalujmy trjkty BDP, CEP i FAP oraz dorysujmy proste prze-
chodzce przez P i rwnolege do prostych AB, AC, BC. Dziel one
trjkt na trzy rwnolegoboki i trzy trjkty rwnoboczne. Kada z tych
gur jest zamalowana w poowie, skd wida, e rwnie poowa trjkta
ABC zostaa zamalowana. Std najmniejsza (i najwiksza) moliwa warto
sumy pl trjktw BDP, CEP i FAP wynosi
1
2
.
4. Majc dan kartk papieru A5 skonstruowa (bez uycia cyrkla
tudzie linijki) trjkt rwnoboczny. Mona zaoy, e boki kartki A5
dziel si w stosunku 1 :

2.
Uwaga: Do opisu konstrukcji naley doczy skonstruowany trjkt.
Rozwizanie
Niech ABCD bdzie kartk A5, przy czym AB < BC.
1. Skadamy kartk wzdu symetralnej odcinka AB.
2. Zginamy kartk tak, aby punkt A wypad na tej symetralnej a
zgicie przechodzio przez punkt B.
3. Zginamy kartk wzdu prostej, na ktrej znajduj si teraz punkty
A i B.
4. Rozkadamy kartk.
47
Zauwamy, e dokonalimy trysekcji kta ABC. Analogicznie dokonu-
jemy trysekcji kta BAD i wtedy dwa z otrzymanych zgi tworz wraz
z odcinkiem AB brzeg trjkta rwnobocznego. Zaginajc niepotrzebne
fragmenty kartki do rodkadostajemy trjkt rwnoboczny.
5. Czy z kwadratowej kartki papieru o wymiarach 7, 997, 99 potrasz
wyci 50 kwadratw jednostkowych?
Rozwizanie
Tak. Bok duego kwadratu ma dugo
11

2
, czyli mniej ni 7,99.
6. Rozstrzygnij, czy istnieje 2008 rnych trjek parami wzgldnie
pierwszych liczb naturalnych a, b, c takich, e a
2
, b
2
i c
2
tworz cig aryt-
metyczny.
Rozwizanie
Tak, istnieje nieskoczenie wiele trjek o tej wasnoci. Rwnanie x
2

2y
2
= 1 jest rwnaniem anty-Pellowskim (tzn. rni si od zwykego
rwnania Pella tym, e po prawej stronie rwnoci jest 1, a nie 1). O
takim rwnaniu wiadomo, e jeli posiada jedno rozwizanie w liczbach
naturalnych, to posiada ich nieskoczenie wiele. Rozwizaniem naszego
48
rwnania w liczbach naturalnych jest np. x = y = 1, a wic rwnanie
to posiada takich rozwiza nieskoczenie wiele. Dla kadego takiego
rozwizania wystarczy wwczas przyj a = 1, b = y, c = x. Liczby te s
oczywicie parami wzgldnie pierwsze.
7. Joszua rzuca monet n razy, za Ahmed n + 1 razy. Oblicz praw-
dopodobiestwo, e Ahmed wyrzuci wicej orw ni Joszua.
Rozwizanie
Zauwamy, e skoro Ahmed rzuca monet n+1 razy, a Joszua n razy, to
Ahmed wyrzuci albo wicej orw albo wicej reszek ni Joszua. Jed-
noczenie - nie mg wyrzuci zarwno wicej reszek jak i orw ni
Joszua, gdy wwczas rzucaby przynajmniej o dwa rzuty wicej. Zatem
dokadnie jeden z dwch wynikw (ory lub reszki) zosta wyrzucony
przez Ahmeda wicej razy ni przez Joszue. Poniewa ory i reszki s
symetryczne, to prawdopodobiestwo, e Ahmed wyrzuci wicej orw
ni Joszua wynosi
1
2
.
8. Udowodnij, e dla kadego x (0, 1) i kadych liczb naturalnych
m, n zachodzi nierwno
(1 x
n
)
m
+ (1 (1 x)
m
)
n
, 1.
Rozwizanie
Wykaemy t nierwno poprzez interpretacj kombinatoryczn. Prze-
prowadmy nastpujcy eksperyment probabilistyczny. Niech w tablicy
liczb o n wierszach i m kolumnach dla kadego pola prawdopodobiestwo,
e liczba w tym polu to 0 wynosi x, a e liczba w tym polu to 1 wynosi
1 x. Poszczeglne losowania s niezalene.
Obliczmy prawdopodobiestwo, e w kadym wierszu wystpuje cho
jedno 0. W konkretnym wierszu prawdopodobiestwo tego, e wystpi
same jedynki wynosi (1x)
m
. Zatem chocia jedno zero wystpi z praw-
dopodobiestwem 1(1x)
m
, a co za tym idzie prawdopodobiestwo, e
w kadym wierszu bdzie cho jedno 0 wynosi dokadnie (1 (1 x)
m
)
n
.
Obliczmy teraz prawdopodobiestwo, e w kadej kolumnie jest chocia
jedna 1. Prawdopodobiestwo wystpienia samych zer w poszczeglnej
kolumnie wynosi x
n
, a wic cho jedna jedynka bdzie z prawdopodobiest-
wem 1 x
n
. Zatem bdzie tak w kadej kolumnie w prawdopodobiest-
wem (1 x
n
)
m
.
49
Zauwamy, e przynajmniej jedno z tych dwch zdarze wystpi. Bdzie
tak poniewa jeeli nie w kadym wierszu wystpi zero, to w pewnym
wierszu bd same jedynki, a co za tym idzie jedynka bdzie w kadej
kolumnie.
Zatem
(1 x
n
)
m
+ (1 (1 x)
m
)
n
, 1.
Zawody druynowe:
1. Znale wszystkie funkcje f : R R speniajce dla dowolnych
x, y R tosamo
f(x f(y)) = f(x) +xf(y) +f(f(y)).
Rozwizanie
Podstawmy najpierw x = f(z). Wwczas otrzymujemy
f(f(z) f(y)) = f(f(z)) +f(z)f(y) +f(f(y)).
Zauwamy, e prawa strona rwnoci jest symetryczna ze wzgldu na y i
z, czyli lewa rwnie. Na mocy tego mamy
f(f(z) f(y)) = f(f(y) f(z)).
Znaczy to, e dla kadego x takiego, e istniej a i b speniajce x = f(a)
f(b) zachodzi f(x) = f(x). Zauwamy teraz, e funkcja stale rwna 0
spenia warunki zadania. Od tego momentu bdziemy zakada, e f nie
jest stale rwna 0. Pokaemy, e przy tym zaoeniu dla dowolnego x
istniej a i b takie, e x = f(a) f(b). Niech y bdzie takie, e f(y) ,= 0.
Mamy
f(f(y)) +xf(y) = f(x f(y)) f(x).
Skoro f(y) ,= 0 to lewa strona jest funkcj liniow od x, czyli kadc
odpowiednio x otrzymujemy z lewej strony dowoln liczb rzeczywist.
Prawa strona jest postaci f(a) f(b). Zatem dla dowolnego x zachodzi
f(x) = f(x).
Podstawmy teraz do rwnoci z treci zadania x = 0. Otrzymujemy
f(f(y)) = f(0) +f(f(y)).
Poniewa f(f(y)) = f(f(y)), to mamy f(0) = 0.
50
Podstawiajc x = f(y) otrzymujemy
0 = 2f(f(y)) +f(y)
2
,
czyli
f(f(y)) =
f(y)
2
2
.
Podstawiajc jeszcze raz do gwnego rwnania x = f(z) i korzystajc
z otrzymanej rwnoci mamy
f(f(z) f(y)) = f(f(z)) +f(z)f(y) +f(f(y))
=
f(z)
2
2
+f(z)f(y)
f(y)
2
2
=
1
2
_
(f(z) f(y))
_
2
.
Poniewa jak wiemy dowolna liczba rzeczywista da si przedstawi w
postaci f(a) f(b), to dla dowolnego x R otrzymujemy f(x) =
x
2
2
.
Po sprawdzeniu, e f(x) =
x
2
2
w istocie spenia wyjciowe rwnanie
otrzymujemy 2 rozwizania, opisane wyej oraz f(x) = 0.
2. Rozstrzygn czy istnieje zbir liczb naturalnych S mocy 2008 taki,
e suma elementw dowolnego podzbioru S jest potg liczby naturalnej
o wykadniku wikszym od 1.
Rozwizanie
Tak, istnieje taki zbir. Udowodnimy indukcyjnie, e istnieje zbir S
k
o
wskazanej wasnoci oraz mocy k.
Dla uatwienia oznacze dobrym nazwiemy podzbir T pewnego zbioru
X o ile suma elementw podzbioru T jest potg liczby naturalnej o
wykadniku wikszym od 1.
Zbir S
1
oczywicie istnieje, moe by na przykad rwny 4.
Przypumy, e istnieje zbir S
k
o zadanej wasnoci, wykaemy, e
istnieje zbir S
k+1
. Dodajmy teraz do zbioru S
k
now, k + 1-sz liczb
o wartoci b, niech U
1
= S
k
b. Niektre z podzbiorw zbioru U
1
mog nie by dobre. Bdziemy je sukcesywnie naprawia tworzc zbiory
U
i
dla i = 1, 2, . . ., gdzie U
n+1
bdzie powstawa z U
n
poprzez pomnoe-
nie wszystkich elementw przez odpowiedni liczb. Zamy, e zbir U
n
ma t podzbiorw dobrych i ich sumy s liczb naturalnych o wykadnikach
odpowiednio m
1
, . . . , m
t
. Niech pewien podzbir W zbioru U
n
nie bdzie
dobry i suma jego elementw wynosi c. Wwczas mnoymy wszystkie
elementy U
n
przez c
NWW(m
1
,...,m
t
)
. Wszystkie dobre podzbiory U
n
nadal
51
pozostay dobre, a zbir o sumie c ma teraz sum c
NWW(m
1
,...,m
t
)+1
, czyli
jest dobry. Okrelamy wic zbir U
n+1
, ktry ma o jeden wicej dobry
podzbir nastpujco: U
n+1
= a c
NWW(m
1
,...,m
t
)
: a U
n
. Gdy wszys-
tkie podzbiory pewnego zbioru U
m
s dobre, to koczymy ten proces. W
ten sposb po skoczonej iloci krokw otrzymamy zbir S
k+1
= U
m
dla
pewnego m, ktrego wszystkie podzbiory s dobre.
Kadc S = S
2008
otrzymujemy poszukiwany zbir.
3. Dana jest rodzina T podzbiorw k elementowych zbioru n elemen-
towego S, przy czym n > 2k. Kady k+1 elementowy podzbir S zawiera
dokadnie m , 1 zbiorw z rodziny T. Wykaza, e T zawiera wszystkie
k elementowe podzbiory S.
Rozwizanie
Policzymy na dwa rne sposoby ilo par (U, V ) gdzie U T i V jest
k +1-elementowym podzbiorem S zawierajcym U. Z jednej strony, jeli
wybierzemy V na jeden z
_
n
k+1
_
sposobw, to z warunkw zadania wynika,
e zbir U moemy dobra na m sposobw, czyli par powyszej postaci
jest m
_
n
k+1
_
. A z drugiej strony, jeli wybierzemy najpierw zbir U, to
zbir V moemy wybra na n k sposobw, gdy musimy do niego
doczy dokadnie jeden element z tych, ktre nie nale do U. Dosta-
jemy wic (n k)[T[ par powyszej postaci. Otrzymujemy rwno
[T[ =
m
n k
_
n
k + 1
_
=
m
k + 1
_
n
k
_
.
Policzymy nastpnie, na dwa rne sposoby, ilo trjek (U, V, W)
gdzie U jest (k+1)-elementowym podzbiorem S, a V, W s rnymi zbio-
rami rodziny T, ktre s zawarte w U. Podobnie jak poprzednio zbir U
moemy wybra na
_
n
k+1
_
sposobw, zbir V na m, a zbir W na m1.
A zatem liczba takich trjek to
m(m1)
_
n
k + 1
_
=
m(m1)(n k)(n k + 1)
k(k + 1)
_
n
k 1
_
.
Zauwamy jednak, e dla dowolnej takiej trjki zbir V W ma dokad-
nie k 1 elementw, gdy gdyby mia ich mniej te zbiory nie mogyby
si jednoczenie zawiera w k + 1-elementowym zbiorze U. Ilo takich
trjek moemy wic zlicza zaczynajc od wybrania V W. Dla k 1-
elementowego podzbioru J zbioru S niech s
J
oznacza liczb zbiorw
52
w T, ktre go zawieraj. Wwczas istnieje dokadnie s
J
(s
J
1) trjek
powyszej postaci, dla ktrych J = V W (gdy wybr V i W wyznacza
U). Dostajemy zatem
m(m1)(n k)(n k + 1)
k(k + 1)
_
n
k 1
_
=

|J|=k1
s
J
(s
J
1).
Kady zbir z rodziny T zawiera w sobie dokadnie k rnych podzbiorw
k 1 elementowych. Mamy wic

|J|=k1
s
J
= k[T[ =
mk
k + 1
_
n
k
_
=
m(n k + 1)
k + 1
_
n
k 1
_
.
Z nierwnoci midzy redni arytmetyczn a kwadratow wynika, e

|J|=k1
s
2
J
,
(

|J|=k1
s
J
)
2
_
n
k1
_
.
czc t nierwno z poprzednimi zalenociami otrzymujemy
m(m1)(n k)(n k + 1)
k(k + 1)
_
n
k 1
_
=

|J|=k1
s
2
J
s
J
,
(

|J|=k1
s
J
)
2
_
n
k1
_

|J|=k1
s
J
=
m
2
(n k + 1)
2
(k + 1)
2
_
n
k 1
_

m(n k + 1)
k + 1
_
n
k 1
_
,
lub po prostu
(m1)(n k)
k
,
m(n k + 1)
k + 1
1.
Ta nierwno jest jednak rwnowana nierwnoci
m(n 2k) , (k + 1)(n 2k),
a poniewa n > 2k jest te m , k+1, czyli po prostu m = k+1. Oznacza
to, e dla dowolnego k + 1-elementowego podzbioru zbioru S kady jego
k-elementowy podzbir naley do T, a wic jasne jest, e do T naley
kady k-elementowy podzbir zbioru S i dowd jest zakoczony.
53
4. Dany jest okrg i rozczne, styczne do niego wewntrznie, okrgi

1
,
2
o rodkach O
1
, O
2
. Niech A i B bd punktami stycznoci wsplnej
stycznej zewntrznej
1
,
2
odpowiednio z
1
i
2
. Wsplne styczne
wewntrzne
1
,
2
przecinaj w punktach C, D, przy czym punkty A,
B, C i D le po tej samej stronie prostej O
1
O
2
. Udowodni, e proste
AB i CD s rwnolege.
Rozwizanie
Niech M i N bd odpowiednio punktami stycznoci
1
i
2
z , a przez
P, Q oznaczmy przecicie prostej AB z . rodek uku PQ zawierajcego
punkty C i D oznaczmy przez S. Aby wykaza rwnolego prostych AB
i CD wystarczy udowodni rwno SC = SD.
M
N
K
G
H
A
B
P
Q
C
D
S
L
X
Y
Zauwamy, e punkty M, A, S s wspliniowe, gdy jednokadno
przeksztacajca
1
w ma rodek w M oraz przenosi punkt A na
punkt S. Analogicznie, wspliniowe s punkty N, B, S. Zauwamy te,
e punkty M, A, B, N le na jednym okrgu. Rzeczywicie, styczna do
okrgu w punkcie S jest rwnolega do prostej AB, a zatem kt midzy
54
ow styczn, a prost MA ma t sam miar co kt SAB. Z drugiej jed-
nak strony, ten sam kt m t sam miar co kt MNS, a zatem punkty
M, A, B, N le na jednym okrgu.
Z punktu S poprowadmy styczne SX, SY odpowiednio do okrgw

1
,
2
. Zauwamy, e z Potgi Punktu otrzymujemy rwnoci
SX
2
= SA SM = SB SN = SY
2
,
a zatem SX = SY .
Niech G i H bd punktami stycznoci wsplnej stycznej okrgw

1
,
2
wyznaczajcej punkt C. Analogicznie niech L, K bd punktami
stycznoci wsplnej stycznej okrgw
1
,
2
wyznaczajcej punkt D.
Na mocy Lematu uytego w rozwizaniu zadania 23 z zawodw indy-
widualnych prawdziwe s zalenoci
MS
SX
=
MC
CG
=
MD
DL
.
Jednoczenie, z Twierdzenia Ptolemeusza otrzymujemy
MS CD = MD SC +MC SD.
Po podzieleniu obu stron tej rwnoci przez
MC
CG
dostajemy
SX CD = DL SC +CG SD.
Przeprowadzajc analogiczne rozumowanie wzgldem punktu N dosta-
jemy rwnie
SY CD = CH SD +DK SC.
A poniewa SX = SY mamy
DL SC +CG SD = CH SD +DK SC,
czyli
SC (DL DK) = SD (CH CG)
lub po prostu
SC KL = SD GH.
Nietrudno jednak zauway, e GH = KL, a zatem SC = SD i
rozwizanie zadania jest zakoczone.
5. Znale gur F R
2
o jak najmniejszym polu, w ktrej da si
obrci odcinek, tzn. istnieje funkcja ciga f : [0, 1] [0, 1] F taka,
55
e f(0, 0) = f(1, 1) oraz f(0, 1) = f(1, 0) oraz dla kadych p, q, t [0, 1]
zachodzi d(f(p, t), f(q, t)) = [p q[, gdzie d to metryka euklidesowa w
R
2
.
Rozwizanie
Dla dowolnie maej liczby > 0 istnieje gura F o polu nie wikszym od
i speniajca warunki zadania.
Z uwagi na dugo penego rozwizania nie piszemy go tutaj, mona
je przeczyta w ksice I.M.Jagoma i W.G.Botiaskiego Figury wy-
puke wydanej w Warszawie w 1955 roku przez PWN (tumaczenie
ksiki wydanej w 1951 roku w Moskwie i Leningradzie), bd te na
stronie http://www.jstor.org/pss/2317619 na ktrej to podany artyku
jest niestety dostpny nie zawsze bezpatnie.
Pierwszy mecz matematyczny:
1. Kulk bdziemy nazywa kul o promieniu 1. Ukad n parami
rozcznych kulek zawartych w kuli K o promieniu R nazywamy dobrym,
gdy nie da si dooy do niego kolejnej kulki (rozcznej i zawartej w K),
za megadobrym, jeli jest dobry oraz nie istnieje dobry ukad o wikszej
liczbie kulek. Dla dwch megadobrych ukadw X i Y udowodni, e da
si tak ustawi rodki kulek z ukadu X w cig A
1
, A
2
, . . . , A
n
, za rodki
kulek z ukadu Y w cig B
1
, B
2
, . . . , B
n
, e dla kadego i 1, 2, . . . , n
dugo odcinka A
i
B
i
nie przekracza 2.
Rozwizanie
Bdziemy chcieli skorzysta z twierdzenia Halla (patrz rozwizanie zada-
nia 16). Kulki ze zbioru X bd chopcami, a kulki ze zbioru Y - dziewcz-
tami. Dziewczynka i chopiec znaj si, jeeli si przecinaj, a wic jeli
odlego ich rodkw nie przekracza 2. Teza twierdzenia Halla jest rw-
nowana tezie zadania. Chcemy sprawdzi, czy speniony jest warunek
Halla. Wemy zatem k kulek ze zbioru X. Niech l bdzie liczb kulek ze
zbioru Y , ktre przecinaj ktr z nich. Rozwamy ukad Z tych k kulek
z X i pozostaych n l kulek z Y . Ten ukad jest dobry, bo dobre s
ukady X i Y oraz kulki z X nie przecinaj kulek z Y . Z denicji ukadu
megadobrego mamy n l + k = [Z[ [X[ = n, czyli k l. Warunek
Halla zachodzi, zachodzi wic zatem i teza zadania.
56
Uwaga
Przez pomyk na obozie zadanie zostao podane z bdn treci. Ukad
by megadobry, gdy by dobry i minimalny, a dugo kadego z od-
cinkw A
i
B
i
miaa nie przekracza 4. Wnikliwy Czytelnik z pewnoci
sprbuje przeprowadzi dowd analogiczny do powyszego, dostrzec w
nim bd, a take uzupeni go, czego ani autor zadania ani pozostali
czonkowie kadry nie byli w stanie zrobi.
2. Znale wszystkie funkcje f : R
+
R
+
speniajce dla dowolnych
x, y R
+
tosamo
f(x +f(y)) = f(x +y) +f(y).
Uwaga: R
+
oznacza zbir liczb rzeczywistych dodatnich.
Rozwizanie
Pokaemy najpierw, e dla kadego x > 0 zachodzi f(x) > x. Gdyby
dla pewnego a prawd byo f(a) < a, to podstawiajc w rwnaniu
x = a f(a), y = a dostalibymy, e f(a) < 0, co jest sprzeczne z
warunkami zadania. Podobnie zakadajc, e f(a) = a dla pewnego a
otrzymalibymy, e f(a) = 0.
Zauwamy, e rwnanie w treci mwi tyle, e jeli od punktu x + y
przesuniemy si w prawo o f(y)y, to wykres podniesie si o f(y). Znaczy
to, e w pewnym sensie wykres funkcji f jest nachylony ze wspczyn-
nikiem C
y
=
f(y)
f(y)y
. Pokaemy teraz, e dla dowolnych x, y R
+
zachodzi
C
x
= C
y
.
Dla kadego przedziau [a, b] funkcja f jest ograniczona z dou, gdy
f(x) > x. Przypumy, e dla pewnych x, y zachodzi C
x
> C
y
. Niech
na pewnym przedziale [a, a + f(x) x] funkcja f bdzie ograniczona
od dou przez m oraz f(b) = m dla pewnego b [a, a + f(x) x].
Intuicyjnie - pokaemy teraz, e poniewa C
x
> C
y
, to idc w prawo
krokami o dugoci f(x) x bdziemy szybciej zwiksza ograniczenie
dolne na wartoci f ni ronie warto f podczas poruszania si krokami
po f(y) y.
Zauwamy, e ograniczenie dolne f na przedziale [a +t(f(x) x), a +
(t + 1)(f(x) x)] wynosi m+tf(x) dla t Z
+
. Mamy wic
f(a +t(f(x) x)) , m+t|f(x) , m+ (t 1)f(x)
57
dla dowolnego t R. Czyli innymi sowy
f(a +t) , m+t
f(x)
f(x) x
f(x) = m+tC
x
f(x).
Jednoczenie mamy f(b+k(f(y)y)) = m+kf(y) dla dowolnego k Z.
Zatem dla dowolnego k Z zachodzi
m+kf(y) = f(b +k(f(y) y)) , m+(k(f(y) y) +(b a))C
x
f(y).
Wynika z tego, e
(k+1)f(y)
k(f(y)y)+(ba)
, C
x
, jednak poniewa dla duych
k liczba
(k+1)f(y)
k(f(y)y)+(ba)
jest dowolnie bliska C
y
, to zachodzi C
y
, C
x
.
Sprzeczno z zaoeniem, e C
x
> C
y
.
Zatem dla dowolnych x, y R
+
mamy C
x
= C
y
, czyli
f(x)
f(x)x
= C
dla pewnego C R. Zatem f(x) = Cf(x) Cx, czyli f(x) = Dx dla
pewnego D R.
Podstawiajc do rwnania wyjciowego dostajemy Dx +D
2
y = Dx +
Dy+Dy, z czego wynika, e D = 2 lub D = 0, jednak D = 0 wykluczamy,
gdy obraz f to R
+
.
Na sprawdzeniu, e funkcja f(x) = 2x istotnie spenia warunki zada-
nia koczymy rozwizanie.
3. Niech a, b, c bd liczbami rzeczywistymi dodatnimi speniajcymi
nierwno
21ab + 2bc + 8ca 12.
Znale najmniejsz moliw warto wyraenia
1
a
+
2
b
+
3
c
.
Rozwizanie
Niech x =
1
a
, y =
2
b
, z =
3
c
. Wwczas liczby x, y, z s rwnie do-
datnie oraz atwo sprawdzi, e nierwno dana w zaoenia zadaniach
przepisuj si jako 2xyz , 2x +4y +7z, chcemy za znale najmniejsz
warto wyraenia x +y +z.
Z nierwnoci z(2xy7) , 2x+4y wynika, e 2xy > 7 oraz z ,
2x+4y
2xy7
.
Z powyszego oszacowania a nastpnie z nierwnoci midzy redni
a geometryczn
x +y +z , x +y +
2x + 4y
2xy 7
=
58
= x +
11
2x
+
_
2xy 7
2x
_
+
2x +
14
x
2xy 7
, x +
11
2x
+ 2

1 +
7
x
2
.
Zachodzi te nierwno 2
_
1 +
7
x
2
,
3+
7
x
2
. Rzeczywicie, po podniesie-
niu do kwadratu i dokonaniu przeksztace widzimy, e nierwno jest
rwnowana nierwnoci 7(1
3
x
)
2
, 0. Otrzymujemy wic
x +y +z ,
3
2
+x +
9
x
,
15
2
.
Rwno zachodzi dla x = 3, y =
5
2
, z = 2. A wic szukana warto
wynosi
15
2
i jest osigana dla a =
1
3
, b =
4
5
oraz c =
3
2
.
4. Dana jest nieparzysta liczba pierwsza p. Udowodni, e zachodzi
kongruencja
p1

i=1
2
i
i
p2

p1
2

i=1
i
p2
(mod p.)
Rozwizanie
Dla wygody w rozwizaniu bdziemy operowa na uamkach modulo p,
tzn. mwimy, e jeli a, b, c, d Z oraz b, d s niepodzielne przez p, to
a
b

c
d
(mod p) wtedy i tylko wtedy, gdy ac bd (mod p). Nietrudno
sprawdzi, e wwczas kadej liczbie wymiernej o mianowniku niepodziel-
nym przez p odpowiada dokadnie jedna reszta modulo p. A wic uywa-
jc tej notacji moemy napisa i
p2

1
i
(mod p) dla 0 < i < p, gdy z
Maego Twierdzenia Fermata i
p1
1 (mod p).
Wykorzystamy dwa fakty. Po pierwsze, przy ustalonym j zachodzi
rwno
n

i=1
_
i
j
_
=
_
n + 1
j + 1
_
.
Mona j udowodni indukcyjnie wykorzystujc tosamo
_
n1
r
_
+
_
n1
r1
_
=
_
n
r
_
. Po drugie
p1

i=1
1
i
0 (mod p).
Istotnie, wystarczy zauway, e
1
i
+
1
pi
0 (mod p), a wic moemy
poparowa skadniki sumy tak, aby kada para dawaa w sumie 0 modulo
p.
59
Zauwamy, e
p1

i=1
2
i
i
p2

p1

i=1
(1 + 1)
i
i
=
p1

i=1
i

j=0
_
i
j
_
i
=
p1

j=0
p1

i=j
_
i
j
_
i
=
p1

j=1
p1

i=j
_
i
j
_
i
+
p1

i=1
1
i

p1

j=1
p1

i=j
i!
j!(ij)!
i
=
p1

j=1
1
j
p1

i=j
_
i 1
j 1
_
=
p1

j=1
1
j
_
p 1
j
_
=
=
p1

j=1
1
j
(p 1)(p 2) . . . (p j)
j!

p1

j=1
1
j
(1)
j
j!
j!
=
p1

j=1
(1)
j
j
=
=
p1
2
1

j=1
1
2j + 1
+
p1
2

j=1
1
2j
2
p1
2

j=1
1
2j
=
p1
2

i=1
1
i

p1
2

i=1
i
p2
(mod p),
co naleao wykaza.
5. W czworocianie ABCD na krawdziach AB, AC, AD, BC, BD,
CD wybrano odpowiednio punkty K, L, M, N, O, P tak, e s one
rne od wierzchokw. Udowodni, e sfery opisane na czworocianach
AKLM, BKNO, CLNP i DMOP przecinaj si w jednym punkcie.
Rozwizanie
Lemat
Na bokach Y Z, ZX, XY trjkta XY Z wybrano punkty U, V , W.
Wwczas okrgi opisane na trjktach XV W, Y UW, ZUV przecinaj
si w jednym punkcie.
Dowd
Niech S bdzie drugim punktem przecicia okrgw XV W i Y UW. Przy-
pumy, e S ley wewntrz trjkta XY Z (pozostae przypadki rozpa-
trujemy analogicznie). Zachodz rwnoci:
USV = 360

USWV SW = UY W +V XW = 180

UZV,
czyli punkty S, U, Z, V lea na jednym okrgu.

Rozwamy teraz inwersj wzgldem punktu A o dowolnym promieniu.
Sfera AKLM przejdzie na paszczyzn K

, a pozostae trzy sfery na


sfery. Mamy pokaza, e te cztery gury przecinaj si w jednym punkcie.
Z lematu wynika, e sfery AKLM, BKNO, CLNP i paszczyzna AKL
60
przecinaj si w jednym punkcie. W takim razie paszczyzny AK

,
K

oraz sfery B

, C

przecinaj si w jednym punkcie,


czyli sfery B

, C

i prosta K

przecinaj si w jednym
punkcie. Analogicznie pozostae pary sfer maj niepuste przecicia z
pozostaymi bokami trjkta K

. W takim razie na paszczynie


K

mamy tak konguracj jak w lemacie - std teza.


6. Ahmed i Fredek zdecydowali si zagra w gr. Tym razem maj
nieskoczon szachownic, pocztkowo pust. Ruch polega na postawie-
niu swojego znaku (Ahmed gra kkami, Fredek za krzyykami, Ahmed
rusza si pierwszy) na dowolnym pustym polu. Zwycia gracz, ktremu
uda si ustawi n swoich znakw w ssiadujcych polach jednego wiersza,
kolumny lub skosu. Rozstrzygn, czy istnieje takie n, dla ktrego przy
dobrej grze Fredka Ahmed nie zdoa wygra w skoczonej liczbie ruchw.
Rozwizanie
Istnieje takie n, e Fredek bdzie posiada strategi blokujc wygran
Fredka. Wskaemy t strategi. Pola na szachownicy bd poczone w
pary, gdy Ahmed postawi swj znak na jednym z tych pl, Fredek stawia
swj znak na drugim z nich.
Podzia na pary pl jest wskazany na rysunku. Pola umieszczone w
jednym dominie oraz ssiadujce ze sob rogiem i posiadajce ten sam
numer znajduj si w jednej parze.
Zauwamy, e dla n = 11 dowolna linia, pozioma, pionowa lub ukona
61
przechodzi przez oba pola pewnej pary, zatem wskazany podzia dostar-
cza Fredkowi strategii blokujcej Ahmeda dla n = 11.
7. Znale wszystkie wielomiany P takie, e dla kadego x rzeczywis-
tego zachodzi rwno
P(x
2
+ 1) = (P(x))
2
+ 1.
Rozwizanie
Przez Q
(n)
(x) bdziemy oznacza n-te zoenie wielomianu Q(x). Roz-
wizaniem zadania s wielomiany P speniajce
P(x) = Q(. . . Q(x) . . .) = Q
(n)
(x),
gdzie Q(x) = x
2
+ 1.
Najpierw wykaemy, e takie wielomiany speniaj rwnanie dane w
treci. Udowodnimy to przez indukcj. Dla n = 0 mamy P(x) = x,
ktry istotnie spenia rwnanie. Zamy, e R(x) = Q
(n)
(x) spenia
rwnanie R(x
2
+ 1) = (R(x))
2
+ 1, wykaemy, e P(x) = R(x)
2
+ 1 =
Q
(n+1)
(x) rwnie spenia rozwaane rwnanie. Istotnie jest to prawd,
mamy bowiem
P(x
2
+ 1) = R(x
2
+ 1)
2
+ 1 = ((R(x))
2
+ 1)
2
+ 1 = (P(x))
2
+ 1,
Udowodnimy teraz, e jedynie wielomiany postaci Q
(n)
(x) speniaj
warunki zadania. Jeli wykaemy, e z tego, e P(x) spenia rwnanie,
wynika P(x) = x, albo P(x) = R(x
2
+ 1), gdzie R(x) rwnie spe-
nia warunki zadania, to otrzymamy, e jedynie rozwaane wielomiany s
rozwizaniem zadania.
Niech zatem P taki, e P(x
2
+1) = (P(x))
2
+1. Aby P(x) = R(x
2
+1)
dla pewnego R wystarczy pokaza, e jedyne niezerowe wspczynniki
P to wspczynniki przy potgach parzystych. Zauwamy, e P(x
2
+
1) 1 ma wspczynniki niezerowe jedynie przy potgach parzystych,
wic (P(x))
2
rwnie musi mie niezerowe wspczynniki jedynie przy
potgach parzystych. Gdyby P(x) posiada niezerowy wspczynnik przy
pewnej potdze parzystej oraz przy pewnej potdze nieparzystej, to (P(x))
2
posiadaby niezerowy wspczynnik przy pewnej potdze nieparzystej.
Oznaczmy bowiem przez M
0
maksymaln potg parzyst, przy ktrej P
posiada niezerowy wspczynnik, a przez M
1
analogiczn potg nieparzyst.
Wwczas (P(x))
2
przy x
M
0
+M
1
posiada niezerowy wspczynnik.
62
Otrzymujemy zatem, e P posiada wspczynniki niezerowe jedynie
przy potgach parzystych lub jedynie przy potgach nieparzystych. Jeli
zachodzi pierwszy przypadek, to oczywicie istnieje R taki, e P(x) =
R(x
2
+ 1), pozostaje wic jedynie wykaza, e w drugim przypadku
P(x) = x.
Jeli P posiada wspczynniki niezerowe jedynie przy potgach niepa-
rzystych, to w szczeglnoci nie posiada wyrazu wolnego, czyli P(0) = 0.
Przypumy, e P(x) / x. Wwczas P(x) x posiada skoczenie wiele
pierwiastkw, czyli w szczeglnoci istnieje skoczenie wiele takich y, e
P(y) = y. Niech y
0
bdzie maksymaln liczb tak, e P(y) = y, w
szczeglnoci wic y
0
, 0. Jednak wwczas P(y
2
0
+ 1) = (P(y
0
))
2
+ 1 =
y
2
0
+ 1, czyli rwnie y
2
0
+ 1 spenia P(y) = y, jednak y
2
0
+ 1 > y
0
, gdy
rwnanie y
2
0
y
0
+ 1 nie posiada pierwiastkw w R. Otrzymujemy wic
sprzeczno z zaoeniem, e P(x) / x.
Tym samym wykazalimy, e jedyne wielomiany speniajce rwnanie
to P(x) = Q
(n)
(x).
8. Dany jest cig wektorw jednostkowych v
1
, v
2
, . . . , v
n
. Rozstrzygn,
czy moemy dobra taki cig znakw, e dla kadego k wektor
k

i=1
v
i
ley
w kole o promieniu 3.
Rozwizanie
Tak, mona dobra taki cig znakw. Oznaczmy w
i
=
2i1
v
2i1
+
2i
v
2i
,
gdzie
2i1
,
2i
1, 1. Zauwamy, e cztery moliwe wektory w
i
w za-
lenoci od wyborw
2i1
i
2i
tworz czwrk, ktr mona tak uporzd-
kowa, e kt pomidzy dwoma kolejnymi wektorami jest ktem prostym.
Zatem bdc w dowolnym punkcie A
k
=
2k

i=1

i
v
i
mona wybra takie
znaki
2k+1
oraz
2k+2
, aby wektor w
k+1
tworzy z wektorem

A
k
0 kt nie
mniejszy ni 135

. Wybieramy wic wszystkie znaki


i
, tak, by speniony
by ten wanie warunek dla dowolnego k.
Pokaemy, e wwczas dla dowolnego m suma
m

i=1
w
i
=
2m

i=1

i
v
i
ley w
kole o promieniu 2.
Wykaemy to przez indukcj. w
0
= 0 spenia tez. Niech
m1

i=1
w
i
ley
w kole o promieniu 2. Zauwamy, e dugo wektora w
m
jest nie wiksza
od 2 (jako suma dwch wektorw o dugoci 1), jednoczenie tworzy on
63
z
m1

i=1
w
i
kt nie wikszy ni 135

. Zatem suma
m

i=1
w
i
rwnie ley w kole
o promieniu 2.
Mamy
k

i=1

i
v
i
=

k
2

i=1
w
i
+
k
v
k
,
gdzie pierwszy skadnik sumy ley w kole o promieniu 2, a drugi ma
dugo nie wiksz ni 1, czyli dla dowolnego k suma
k

i=1

i
v
i
ley w kole
o promieniu 3.
9. Znale wszystkie rozwizania rwnania x
3
+2x+1 = 2
n
w liczbach
naturalnych.
Rozwizanie
W rozwizaniu wykorzystamy znane wasnoci symbolu Legendrea. Jeli
a Z, a p jest pierwsze, to symbol Legendrea
_
a
p
_
deniujemy jako: 0
jeli p[a; 1 jeli a jest niezerow reszt kwadratow modulo p oraz 1 w
przeciwnym wypadku.
Jasne jest, e dla n = 1 nie ma rozwiza, a dla n = 2 jedynym
rozwizaniem jest x = 1. Przyjmijmy wic, e n , 3. Badajc wszystkie
reszty z dzielenia przez 8, ktre moe dawa x nietrudno sprawdzi, e
x
3
+ 2x + 1 dzieli si przez 8 wtedy i tylko wtedy, gdy x 5 (mod 8).
Podobnie sprawdzamy, e lewa strona danego rwnania daje reszt 1 z
dzielenia przez 3, a zatem n = 2k dla pewnego k naturalnego. Dla n = 2
jedynym rozwizaniem jest x = 1, moemy wic zaoy, e n , 4. Po
dodaniu 2 do obu stron rwnania przybiera ono posta
(x + 1)(x
2
x + 3) = 2
n
+ 2 = (2
k
)
2
+ 2.
Poniewa x 5 (mod 8) wiadomo, e x
2
x + 3 7 (mod 8). Jeli
wszystkie dzielniki pierwsze liczby x
2
x + 3 byyby postaci 8k + 1 lub
8k + 3 to liczba x
2
x + 3 rwnie byaby tej postaci, a zatem istnieje
pewna liczba pierwsza p postaci 8k + 5 lub 8k + 7 taka, e p[x
2
x + 3.
Wwczas rwnie (2
k
)
2
2 (mod p) co oznacza, e liczba 2 jest reszt
kwadratow mod p, czyli (
2
p
) = 1. Ale jeli p jest postaci 8k + 5, to
_
2
p
_
=
_
1
p
__
2
p
_
= 1 (1) = 1,
64
a jeli jest postaci 8k + 7
_
2
p
_
=
_
1
p
__
2
p
_
= (1) 1 = 1
czyli sprzeczno. Jedynym rozwizaniem jest wic x = 1, n = 2.
10. Okrg o rodku O jest styczny wewntrznie do dwch okrgw w
jego wntrzu w punktach S i T. Okrgi te przecinaj si w punktach M i
N, przy czym punkt N ley bliej prostej ST. Udowodni, e proste OM
i MN s prostopade wtedy i tylko wtedy, gdy punkty S, N, T le na
jednej prostej.
Rozwizanie
Poprowadmy styczne do duego okrgu w punktach S i T i oznaczmy
przez P punkt ich przecicia. Zauwamy, e punkty P, S, O, T le na
okrgu o rednicy OP. Potgi punktu P wzgldem maych okrgw s
rwne, wic P ley na prostej MN. Prawdziwy jest cig rwnowanoci:
Proste OM i MN s prostopade punkty O, M, P, S, T lea na
jednym okrgu SMN + TMN + SPT = 180

NSP +
NTP +SPT = 180

punkty S, N, T le na jednej prostej.


11. Niech a bdzie liczb naturaln wiksz od 1. Cig a
n
deniujemy
wzorem
a
n
= a
n+1
+a
n
1.
Wykaza, e istnieje podcig cigu a
n
, ktrego dowolne dwa wyrazy s
wzgldnie pierwsze.
Rozwizanie
Skonstruujemy cig (b
n
), bdcy podcigiem cigu (a
n
), o wyrazach
parami wzgldnie pierwszych. Przyjmijmy b
1
= a
1
i zamy, e wybral-
imy ju k wyrazw cigu b
n
. Niech p
1
, p
2
, . . . , p
l
bd wszystkimi liczbami
pierwszymi, ktre dziel ktr z liczb b
1
, b
2
, . . . , b
k
.
Wemy b
k+1
= a
(p
1
1)(p
2
1)...(p
l
1)
i ustalmy i 1, 2, . . . , l. Naturalnie
liczba b
k+1
jest wiksza ni wszystkie wczeniej wybrane wyrazy cigu
b
n
. Jasne te jest, e skoro p
i
dzieli pewien wyraz cigu a
n
, to nie dzieli
liczby a. Z Maego Twierdzenia Fermata:
b
k+1
= a
(p
1
1)(p
2
1)...(p
l
1)
= a
(p
1
1)(p
2
1)...(p
l
1)+1
+a
(p
1
1)(p
2
1)...(p
l
1)
1
a + 1 1 a (mod p
i
).
65
Tak dobrany wyraz k +1-wszy jest wic niepodzielny przez adn z liczb
p
i
, czyli jest wzgldnie pierwszy z wszystkimi poprzednimi wyrazami
cigu b
n
.
12. Symetralne bokw AB i BC nierwnobocznego trjkta ABC
przecinaj boki BC i AB odpowiednio w punktach A
1
i C
1
. Dwusieczne
ktw A
1
AC i C
1
CA przecinaj si w punkcie B

, a punkty A

oraz
C

deniujemy analogicznie. Dowie, e punkty A

, B

, C

le na jed-
nej prostej, ktra przechodzi przez rodek okrgu opisanego na trjkcie
ABC.
Rozwizanie
Proste AA
1
i BA
1
s symetryczne wzgldem symetralnej odcinka AB.
Wobec tego prosta AA
1
przechodzi przez punkt C
2
symetryczny do punktu
C wzgldem tej symetralnej. Dwusieczna kta A
1
AC przechodzi zatem
przez rodek C
3
uku CC
2
, ktry jest jednoczenie rodkiem jednego z
ukw AB. Oznaczmy przez C
4
rodek drugiego uku AB, za punkty
A
3
, A
4
zdeniujmy analogicznie. Oczywicie proste AA
4
i CC
4
s dwu-
siecznymi ktw wewntrznych trjkta. Zastosujmy teraz twierdzenie
Pascala dla szeciokta AA
4
A
3
CC
4
C
3
. Dostajemy, e rodek I okrgu
wpisanego w trjkt ABC, rodek O okrgu opisanego na oraz B

le na
jednej prostej. Poniewa trjkt ABC jest nierwnoboczny, wic I ,= O
i punkty A

, B

, C

lea na prostej IO.


Drugi mecz matematyczny:
1. Dla danej liczby naturalnej n , 1 niech A oznacza liczb sposobw
na jak mona zapisa n w postaci sumy liczb cakowitych dodatnich
nieparzystych, a B niech oznacza liczb sposobw na jak mona zapisa
n w postaci sumy rnych liczb cakowitych dodatnich (w obu zapisach
nie zwracamy uwagi na kolejno wystpowania skadnikw). Udowodni,
e A = B.
Rozwizanie
Zauwamy, e A jest wspczynnikiem przy x
n
w wyraeniu:
A(x) =

k=0

j=0
x
(2k+1)j
.
66
Analogicznie okrelamy:
B(x) =

k=1
(1 +x
k
).
Teraz wystarczy pokaza, e A(x) = B(x). Ale:
A(x) =

k=0
1
1 x
(2k+1)
=

k=1
(1 x
2k
)

k=1
(1 x
k
)
=

k=1
1 x
2k
1 x
k
= B(x).
Uwaga
Aby uczyni powyszy dowd bardziej formalnym naleaoby przyj np.
A(x) =

n1
k=0

2
a(k)
1
j=0
x
(2k+1)j
, gdzie a(k) = log
2
n
2k+1
| + 1 i B(x) =

n
k=1
(1 +x
k
).
2. Niech k, t > 1 bd wzgldnie pierwszymi liczbami naturalnymi.
Majc dan permutacj (a
1
, a
2
, . . . , a
n
) zbioru 1, 2, . . . , n moemy za-
mieni w niej dwie liczby miejscami jeli rni si o k lub t. Wykaza,
e zaczynajc od permutacji (1, 2, ..., n) moemy otrzyma kad permu-
tacj zbioru 1, 2, . . . , n wtedy i tylko wtedy, gdy n , k +t 1.
Rozwizanie
Przypumy najpierw, e 1 < n < k + t 1. Rozwamy graf o n wierz-
chokach odpowiadajcym liczbom 1, . . . , n. Par wierzchokw czymy
krawdzi, jeli da si zamieni miejscami odpowiadajce im liczby. Za-
uwamy, e permutujc liczby nigdy nie wyrzucimy adnej poza jej spjn
skadow. Aby pokaza, e nie da si uzyska wszystkich permutacji
wystarczy dowie, e graf nie jest spjny. Policzmy jego krawdzie. Ot
takich odpowiadajcych rnicy k jest maxnk, 0, a tych odpowiada-
jcych rnicy t jest maxn t, 0. Jeeli co najmniej jedna z tych liczb
jest zerem, to cznie krawdzi jest mniej ni n 1, bo n, k, t > 1. Jeeli
obie nie s zerami, to mamy cznie krawdzi nk+nt < n1. W obu
przypadkach graf nie jest spjny (dooenie kolejnej krawdzi zmniejsza
liczb spjnych skadowych o co najwyej 1; graf pusty ma ich n, wic
aby dosta jedn potrzeba przynajmniej n1 krawdzi). Zamy teraz,
e n = k +t 1. Wykaemy, e analogicznie otrzymany graf jest ciek
dlugoci n 1. Rozwamy operacj polegajc na dodaniu do liczby k
i wzicia reszty z dzielenia przez n + 1. Poniewa k + t = n + 1, wic
dla liczb 0, 1, . . . , n moemy tylko doda k lub odj t. Ponadto startujc
od 0 i wykonujc n + 1 krokw wyczerpiemy po drodze wszystkie liczby
67
0, 1, . . . , n, bo k i t, a wic rwnie k i n + 1, s wzgldnie pierwsze.
Wobec tego gdybymy do grafu dodali wierzchoek 0 i krawdzie 0k i
t0, to mielibymy cykl dugoci n + 1. Wobec tego wyjciowy graf jest
ciek dugoci n1. Postpujemy teraz indukcyjnie: zamy, e liczby k
(mod n+1), 2k (mod n+1), . . . , mk (mod n+1) s ju na swoich miejs-
cach w wynikowej permutacji (na pocztku m moe by rwne 0). Wemy
liczb (m+1)k (mod n+1). Na jej miejscu jest liczba lk (mod n+1) dla
l > m. Zamieniamy teraz (i +1)k (mod n +1) z ik (mod n +1) kolejno
dla i = l 1, l 2, . . . , m + 1. W wyniku tych operacji liczba (m + 1)k
(mod n+1) jest na swoim miejscu w wynikowej permutacji. Po n takich
krokach dostajemy dowoln permutacj. Zamy teraz, e n > k +t 1
i skorzystajmy z indukcji matematycznej. Jeeli mamy otrzyma permu-
tacj z n na kocu, to jest to permutacja cigu (1, 2, . . . , n 1), ktr
otrzymamy na mocy zaoenia indukcyjnego. Jeeli nie, to najpierw per-
mutujemy cig (1, 2, . . . , n 1) tak, aby tam gdzie ma by n byo n k.
Nastpnie zamieniamy te dwie liczby i wtedy n jest ju na swoim miejscu,
wic pozostaje poprzestawia liczby 1, 2 . . . n 1, co umiemy zrobi.
3. Zbiory A
1
, A
2
, . . . , A
n
s podzbiorami zbioru n-elementowego
A, przy czym kady z nich ma co najmniej 2 elementy. Dla kadego
dwuelementowego podzbioru A

zbioru A istnieje dokadnie jeden taki


A
i
, e A

A
i
. Udowodni, e jeli 1 i, j n to A
i
A
j
,= .
Rozwizanie
W poniszym rozwizaniu przyjmujemy, e
_
x
2
_
=
x(x1)
2
, w szczeglnoci
_
1
2
_
= 0.
Niech A = x
1
, x
2
. . . , x
n
. Przyjmijmy, e n
i
= [A
i
[ oraz niech d
i
oznacza ilo podzbiorw A
j
(przy 1 j n), dla ktrych x
i
A
j
.
Zauwamy na pocztek, e z treci zadania wynika rwno
n

i=1
_
n
i
2
_
=
_
n
2
_
, (2)
gdy obie strony tej rwnoci odpowiadaj liczbie dwuelementowych pod-
zbiorw zbioru A. Jasna jest te rwno
n

i=1
d
i
=
n

i=1
n
i
. (3)
Poniewa kady dwuelementowy podzbir zbioru A naley do dokadnie
jednego zbioru A
i
wiadomo, e dla 1 i < j n zachodzi [A
i
A
j
[ 1.
68
Chcemy pokaza, e dla wszystkich i, j powysza nierwno jest rwno-
ci, czyli innymi sowy wystarczy wykaza, e

1i<jn
[A
i
A
j
[ =
_
n
2
_
.
Zwrmy jednak uwag, e skoro element x
k
naley do d
k
zbiorw A
i
, to
naley do dokadnie
_
d
k
2
_
przeci postaci A
i
A
j
. Zachodzi wic rwno
n

i=1
_
d
i
2
_
=

1i<jn
[A
i
A
j
[
i wystarczyoby dowie, e
n

i=1
_
d
i
2
_
=
_
n
2
_
.
Ze wzgldu na fakt, e
_
x
2
_
=
x
2
x
2
oraz na rwnoci (1) i (2) musimy
udowodni, i
n

i=1
d
2
i
=
n

i=1
n
2
i
. (4)
Dla ustalonego x
i
rozwamy dowolny zbir A
j
= y
1
, y
2
, . . . , y
s
taki,
e x
i
, A
j
. Wwczas wszystkie 2-elementowe zbiory x
i
, y
1
, x
i
, y
2
,
. . . , x
i
, y
s
zawieraj si w rnych zbiorach A
t
, bowiem elementy y
k
i
y
l
nie mog oba nalee do innego zbioru ni A
j
. Oznacza to, e x
i
naley
do co najmniej s rnych zbiorw A
t
, tzn. d
i
, n
j
, o ile tylko x
i
, A
j
.
Nietrudno sprawdzi, e z powyszej nierwnoci wynika te nierwno
d
i
n d
i
,
n
j
n n
j
.
Zauwamy, e jest dokadnie n d
i
zbiorw A
t
, do ktrych nie naley
x
i
oraz dokadnie n n
j
elementw x
i
, ktre nie nale do zbioru A
j
. A
zatem
n

i=1
d
i
=
n

i=1

j:x
i
A
j
d
i
n d
i
,
n

i=1

j:x
i
A
j
n
j
n n
j
=
=
n

j=1

i:x
i
A
j
n
j
n n
j
=
n

j=1
n
j
.
69
Ze wzgldu na rwno (2) we wszystkich powyszych nierwnociach
zachodzi rwno. Wynika std, e jeli x
i
, A
j
, to d
i
= n
j
. Mamy wic
n

i=1
(n d
i
)d
i
=
n

i=1

j:x
i
A
j
d
i
,
n

i=1

j:x
i
A
j
n
j
=
=
n

j=1

i:x
i
A
j
n
j
=
n

j=1
(n n
j
)n
j
,
a std oraz z rwnoci (2) wynika natychmiast rwno (3) i koczy to
rozwizanie zadania.
4. Rozwiza w liczbach rzeczywistych ukad rwna
_

_
a
2
2b
2
= 1
2b
2
3c
2
= 1
ab +bc +ca = 1
Rozwizanie
Z dwch pierwszych rwna wynika natychmiast, e a, b ,= 0. Ponadto,
jeli c = 0, to [a[ =

2 oraz [b[ =
1

2
. By trzecie rwnanie byo spenione
potrzeba i wystarcza aby a i b byy tego samego znaku. Udowodnimy,
e trjki (

2,
1

2
, 0), (

2,
1

2
, 0) s jedynymi rozwizaniami danego
ukadu.
Zamy wic, e istnieje trjka (a, b, c) speniajca dany ukad, dla
ktrej abc ,= 0. Moemy przyj, e co najmniej dwie z liczb a, b, c s
dodatnie, gdy jeli trjka (a, b, c) jest rozwizaniem danego ukadu to
jest nim rwnie (a, b, c).
Dokonajmy podstawienia
a = ctg , b = ctg , c = ctg ,
gdzie 0 < , , < . Patrzc na trzecie rwnanie widzimy, e a +b ,= 0,
gdy w przeciwnym razie ab = 0 co wykluczylimy na pocztku. A zatem
moemy wyliczy c =
1ab
a+b
. Korzystajc ze wzoru na cotangens sumy
dostajemy
ctg = c =
1 ab
a +b
=
1 ctg ctg
ctg + ctg
=
ctg ctg 1
ctg + ctg
= ctg ( +) = ctg ( ( +)).
70
A zatem mamy = ( + ) lub = 2 ( + ). Ale co najmniej
dwie spord liczb a, b, c s dodatnie co oznacza, e co najmniej dwa kty
z , , s mniejsze ni

2
. Czyli
+ + <

2
+

2
+ < 2.
Czyli zachodzi pierwsza z rwnoci i + + = .
Zauwamy, e z danych rwna wynikaj rwnoci
a
2
+ 1 = 2(b
2
+ 1) = 3(c
2
+ 1)
co, po wykorzystaniu tego, e ctg
2
x + 1 =
cos
2
x
sin
2
x
+ 1 =
1
sin
2
x
, sprowadza
si do
1
sin
2

=
2
sin
2

=
3
sin
2

,
ale wszystkie sinusy wystpujce w powyszym wyraeniu s dodatnie,
a wic po prostu
1
sin
=

2
sin
=

3
sin
.
Poniewa 0 < , , < oraz + + = istnieje trjkt ABC o
ktach , , . Z Twierdzenia Sinusw zastosowanego do trjkta ABC i
poczonego z powysz rwnoci otrzymujemy, e jego boki na przeci-
wko ktw , , s odpowiednio dugoci k,

2k,

3k, gdzie k > 0 jest


pewn liczb rzeczywist. Ale wwczas trjkt ABC jest prostoktny i
=

2
, a wic c = 0. To jest jednak sprzeczno z przyjtym zaoeniem, a
zatem trjki wypisane na pocztku stanowi jedyne rozwizanie danego
ukadu.
5. Niech a
1
, a
2
, ..., a
n
bd rnymi liczbami naturalnymi. Dowie, e
zachodzi nierwno
a
7
1
+a
7
2
+... +a
7
n
+a
5
1
+a
5
2
+... +a
5
n
, 2(a
3
1
+a
3
2
+... +a
3
n
)
2
Rozwizanie
Bdziemy rozumowa indukcyjnie. Dla n = 1 nasza nierwno to a
7
1
+
a
5
1
, 2a
6
1
i jej prawdziwo wynika wprost z nierwnoci midzy redni
arytmetyczn a geometryczn.
Zamy wic, e nierwno zachodzi dla n = k i udowodnijmy j dla
n = k +1. Ze wzgldu na symetri bez straty oglnoci moemy zaoy,
71
e a
1
< a
2
< . . . < a
k
< a
k+1
. Poniewa a
1
, a
2
, . . . , a
k
, a
k+1
s rnymi
liczbami naturalnymi zachodzi nierwno
a
3
1
+a
3
2
+. . . +a
3
k
1
3
+ 2
3
+. . . + (a
k+1
1)
3
=
a
2
k+1
(a
k+1
1)
2
4
,
z ktrej wprost wynika, e
4a
3
k+1
(a
3
1
+a
3
2
+. . . +a
3
k
) a
5
k+1
(a
k+1
1)
2
= a
7
k+1
+a
5
k+1
2a
6
k+1
,
czyli
a
7
k+1
+a
5
k+1
, 4a
3
k+1
(a
3
1
+a
3
2
+. . . +a
3
k
) + 2a
6
k+1
.
Z zaoenia indukcyjnego wiemy te, e
a
7
1
+a
7
2
+. . . +a
7
k
+a
5
1
+a
5
2
+. . . +a
5
k
, 2(a
3
1
+a
3
2
+. . . +a
3
k
)
2
.
czc te dwie nierwnoci dostajemy
a
7
1
+a
7
2
+. . . +a
7
k+1
+a
5
1
+a
5
2
+. . . +a
5
k+1
, 2(a
3
1
+a
3
2
+. . . +a
3
k
)
2
+4a
3
k+1
(a
3
1
+a
3
2
+. . . +a
3
k
) + 2a
6
k+1
= 2(a
3
1
+a
3
2
+... +a
3
k
+a
3
k+1
)
2
,
co koczy dowd indukcyjny.
6. Niech n bdzie liczb naturaln. Udowodni, e wielomian (x
2
+
x)
2
n
+1 jest nierozkadalny na iloczyn niestaych wielomianw o wspczyn-
nikach cakowitych.
Rozwizanie
Bdziemy korzystali z wasnoci wielomianw nad Z
2
(ciao modulo 2).
Kady taki wielomian posiada wspczynniki, argumenty i wartoci w
zbiorze 0, 1. Mwimy, e dwa wielomiany nad Z
2
s sobie rwne jeli
rwne s odpowiadajce wspczynniki (zakadamy oczywicie, e wsp-
czynnik wiodcy to 1). Wikszo wasnoci jest analogiczna do tych dla
zwykych wielomianw nad Z, w szczeglnoci np. prawdziwa jest jednoz-
naczno rozkadu na iloczyn wielomianw nierozkadalnych. Nieprawd
jest natomiast, e jeli dwa wielomiany przyjmuj te same wartoci to s
sobie rwne, np. wielomian x
2
x w Z
2
przyjmuje te same wartoci co
wielomian 0. Kademu wielomianowi P(x) o wspczynnikach cakow-
itych odpowiada pewien wielomian w Z
2
, ktry powstaje przez zredu-
kowanie wspczynnikw P modulo 2. Dla uproszczenia bdziemy pisali
P(x) =
2
Q(x) jeli P i Q posiadaj ten sam wielomian zredukowany.
72
Zauwamy na pocztek, e jeli 0 < k < 2
n
, to
_
2
n
k
_
jest liczb
parzyst. Mamy bowiem
_
2
n
k
_
=
2
n
k

2
n
1
1

2
n
2
2
. . .
2
n
(k 1)
k 1
.
Liczby 1, 2, . . . , k nie s podzielne przez 2
k
, a zatem najwysza potga
liczby 2 dzielca 2
i
i jest taka sama jak liczby i. Oznacza to, e uamek
2
i
i
i
zapisany w postaci nieskracalnej ma licznik i mianownik niepodzielny
przez 2. A z kolei licznik uamka
2
n
k
zapisanego w postaci nieskracalnej
jest podzielny przez 2, co wiadczy o tym, e 2[
_
2
n
k
_
.
Niech F(x) = (x
2
+ x)
2
n
+ 1 i zamy, e F(x) = G(x) H(x) dla
pewnych niestaych wielomianw G(x), H(x) o wspczynnikach cakow-
itych. Ze wzoru dwumianowego Newtona i powyej udowodnionego faktu
wynika, e (x
2
+ x)
2
n
po zredukowaniu modulo 2 odpowiada wielomi-
anowi x
2
n
+x
2
n
, a wic F(x) po zredukowaniu odpowiada wielomianowi
x
2
n
+x
2
n
+1. Ale z tych samych powodw ten sam wielomian odpowiada
wielomianowi (x
2
+ x + 1)
2
n
. Czyli F(x) =
2
(x
2
+ x + 1)
2
n
. Zauwamy
jednak, e wielomian x
2
+ x + 1 nie ma pierwiastka w Z
2
a wic jest
nierozkadalny w Z
2
, gdy jest stopnia 2. A skoro
F(x) = G(x) H(x) =
2
(x
2
+x + 1)
2
n
,
to G(x) =
2
(x
2
+ x + 1)
k
i H(x) =
2
(x
2
+ x + 1)
2
n
k
dla pewnego
0 k 2
n
. Zwrmy uwag, e musi by nawet 0 < k < 2
n
, gdy
inaczej wielomian H lub odpowiednio wielomian G byby co najmniej
stopnia 2
n+1
, czyli tego stopnia co F, a to nie jest moliwe bo G, H nie
s stae. Istniej wic takie wielomiany U(x), V (x) o wspczynnikach
cakowitych, e G(x) = (x
2
+ x + 1)
k
+ 2U(x) oraz H(x) = (x
2
+ x +
1)
2
n
k
+ 2V (x). Mamy zatem
(x
2
+x)
2
n
+ 1 = ((x
2
+x + 1)
k
+ 2U(x))((x
2
+x + 1)
2
n
k
+ 2V (x)).
Niech oznacza liczb zespolon bdc pierwiastkiem wielomianu
x
2
+x + 1 (czyli jest to pierwiastek trzeciego stopnia z jednoci). atwo
zauway, e wwczas U() jest liczb zespolon postaci u
1
+u
2
gdzie
u
1
, u
2
Z. Wynika to z faktu, e
2
= 1, a wic jeli wystpuje w
wikszej potdze ni 1 to moemy j zredukowa. Analogicznie V () =
v
1
+v
2
dla v
1
, v
2
Z. Podstawiajc wic x = do powyszej rwnoci
dostajemy
73
(1)
2
n
+ 1 = 4(u
1
+u
2
)(v
1
+v
2
),
czyli, po przyrwnaniu czci rzeczywistych i urojonych, u
1
v
1
=
1
2
. A
to nie jest moliwe, co dowodzi, e nie istnieje rozkad F(x) na iloczyn
niestaych wielomianw o wspczynnikach cakowitych.
7. ABCD jest czworoktem wypukym, w ktrym prosta AC jest
dwusieczn kta BAD. Punkt E ley na odcinku CD, a F jest przeci-
ciem BE i AC. Odcinek DF przeduamy do przecicia z bokiem BC w
punkcie G. Wykaza, e GAC = EAC.
Rozwizanie
Bez straty oglnoci przyjmijmy AB AD. Niech punkty D

, E

bd
obrazami punktw D, E w symetrii wzgldem prostej AC. Wiadomo, e
punkty A, B, D

s wspliniowe; naley pokaza, e punkty A, G, E

rwnie. Niech x, y, z bd takimi liczbami rzeczywistymi nieujemnymi,


e
x
z
=
D

B
BA
oraz
y
z
=
DE
EC
. Umiemy w punkcie A mas x, w punkcie C
mas y, a w punktach D i D

mas z. Wida, e rodkiem masy punktw


A i D

jest punkt B, a punktw C i D - punkt E. Wobec tego rodek masy


czwrki punktw ley na prostej BE. Ponadto rodki mas par punktw
A, C oraz D, D

le na prostej AC, wic ley tam te rodek masy caej


czwrki. Wobec tego rodkiem masy czwrki A, C, D, D

jest punkt F.
W takim razie rodek masy punktw A, C, D

ley na prostej DF. Musi


te lee na prostej BC, bo B jest rodkiem masy A i D

. W takim razie
rodkiem masy trjki A, C, D

jest punkt G. Ale rodkiem masy C i D

jest E

, wic punkty A, G, E

s wspliniowe.
8. Dany jest nierozwartoktny trjkt ABC. Punkt D jest spodkiem
wysokoci z A, natomiast I
1
oraz I
2
s odpowiednio rodkami okrgw
wpisanych w trjkty ABD i ACD. Prosta I
1
I
2
przecina AB i AC
odpowiednio w punktach P i Q. Udowodni, e AP = AQ wtedy i tylko
wtedy, gdy AB = AC lub A = 90

.
Rozwizanie
Jasne jest, e jeli AB = AC, to AP = AQ. Zamy wic najpierw, e
A = 90

.
Niech P

i Q

bd takimi punktami odpowiednio na bokach AB i AC,


e AP

= AQ

= AD. Wwczas AP

= AQ

= 45

. Niech I

1
i I

2
oznaczaj odpowiednio przecicia dwusiecznych ktw BAD i CAD
74
z prost P

. Wtedy z cechy bok-kt-bok mamy przystawanie trjktw


AP

1
z ADI

1
oraz AQ

2
z ADI

2
. A zatemADI

1
= ADI

2
= 45

, czyli
punkt I

1
ley na dwusiecznej kta BDA, a I

2
ley na dwusiecznej kta
CDA. Wynika std, e punkty I

1
i I

2
s rodkami okrgw wpisanych
odpowiednio w trjkty ABD i ACD, a zatem I

1
= I
1
oraz I

2
= I
2
. A
to oznacza, e P

= P oraz Q

= Q, czyli AP = AQ.
Zamy teraz, e AP = AQ. Niech D bdzie takim punktem pprostej
AD, e AD

= AP = AQ. Przyjmijmy, e D ,= D

. Trjkt API
1
jest
przystajcy do trjkta AD

I
1
a trjkt AQI
2
jest przystajcy do AD

I
2
,
czyli w szczeglnoci
AD

I
1
= API
1
= AQI
2
= AD

I
2
.
A zatem mamy I
1
D

D = I
2
D

D i poniewa wiemy rwnie, e I


1
DD

=
I
2
DD

(oba te kty maj miar 45

jeli D

ley na odcinku AD oraz


135

jeli ley poza) otrzymujemy przystawanie trjktw I


1
D

D i I
2
D

D.
A wic przystajce s te trjkty AD

I
1
oraz AD

I
2
czyli mamy
BAD = 2I
1
AD

= 2I
2
AD

= CAD,
skd wnioskujemy, e AB = AC.
Przyjmijmy teraz, e D = D

. Wwczas
API
1
= ADI
1
= ADI
2
= AQI
2
= 45

,
czyli A = 90

, co koczy dowd.
9. Dany jest trjkt ABC. Okrg o jest styczny do odcinkw AB i
AC odpowiednio w punktach D i E, rnych od B i C. Ten sam okrg
przecina bok BC w punktach K i L. Odcinki AL i DE przecinaj si w
punkcie P, a przektne czworokta BCED przecinaj si w punkcie Q.
Dowie, e punkty P, Q, K s wspliniowe.
Rozwizanie
Z twierdzenia o potdze punktu wzgldem okrgu wynikaj rwnoci
BD
2
= BK BL oraz CE
2
= CK CL. Zamy najpierw, e AB ,= AC.
Niech T bdzie punktem przecicia prostych BC i DE. Z twierdzenia
Menelaosa dla trjkta ABC i prostej DE mamy
TB
TC
=
BD
CE
. Niech K

bdzie punktem wsplnym prostych BC i PQ. Poniewa rzut z punktu


zachowuje dwustosunek, wic mamy
TB LC
TC LB
=
TD PE
TE PD
=
TC K

B
TB K

C
,
75
czyli
K

B
K

C
=
TB
2
LC
TC
2
LB
=
BD
2
LB

LC
CE
2
=
KB
KC
.
W takim razie K = K

.
Jeeli natomiast AB = AC, to BC[[DE, BK = CL, oraz BL = CK. Z
twierdzenia Talesa otrzymujemy:
K

B
K

C
=
PE
PD
=
LC
LB
=
KB
KC
,
czyli znowu K = K

.
10. Wyznaczy najmniejsz liczb naturaln n > 1, dla ktrej rednia
kwadratowa liczb 1, 2, ..., n jest liczb cakowit.
Rozwizanie
Jako, e 1
2
+ 2
2
+ . . . + n
2
=
n(n+1)(2n+1)
6
nasze zadanie sprowadza si
do znalezienia najmniejszej liczby naturalnej n > 1, dla ktrej istnieje
m N speniajce rwnanie
(n + 1)(2n + 1)
6
= m
2
.
Po pomnoeniu obu stron rwnania przez 48 i zwiniciu w kwadraty
powysze rwnanie przeksztaca si do postaci
(4n + 3)
2
3(4m)
2
= 1.
Szukamy wic najmniejszego rozwizania rwnania Pella x
2
3y
2
= 1,
dla ktrego x > 7, x 3 (mod 4) oraz 4[y. Rozwizaniem minimalnym
tego rwnania jest para (x
0
, y
0
) = (2, 1), pozostae rozwizania otrzymu-
jemy z rekurencji x
k+1
= 2x
k
+ 3y
k
, y
k+1
= x
k
+ 2y
k
. Obliczamy rcznie
kolejne pary: (7, 4), (26, 15), (97, 56), (362, 209), (1351, 780). Ostatnia z
wypisanych par spenia dane warunki, a wic n = 337 jest rozwiza-
niem zadania.
11. Dla danego h = 2
r
, gdzie r , 0, wyznaczy wszystkie liczby
naturalne k, dla ktrych istnieje m > 1 nieparzyste oraz liczba naturalna
n taka, e k[m
h
1 oraz m[n
m
h
1
k
+ 1.
76
Rozwizanie
Udowodnimy, e rozwizaniem zadania s wszystkie liczby k, dla ktrych
2
r+1
[k. Pokaemy najpierw, e jest to warunek konieczny.
Zamy wic, e liczba 2 nie dzieli k w wyszej potdze ni r i e
istnieje m > 1 nieparzyste oraz n, ktre speniaj warunki dane w treci
zadania. Niech t oznacza najwysz potg liczby 2 dzielc m 1.
Zauwamy, e m
h
1 = m
2
r
1 = (m
2
r1
1
)(m
2
r1
+ 1) = . . . =
(m 1)(m + 1)(m
2
+ 1) . . . (m
2
r1
+ 1), a poniewa m jest nieparzyste
kada z liczb m+1, m
2
+1, . . . , m
2
r1
+1 dzieli si przez 2. Wynika std,
e 2
t+r
[m
h
1. A zatem mamy rwnie 2
t
[
m
h
1
k
. Spord wszystkich dziel-
nikw pierwszych liczby m wybierzmy ten dzielnik p, dla ktrego potga
liczby 2 dzielca p 1 jest najmniejsza i oznaczmy j przez s. Jasne jest,
e t , s. Zachodzi te kongruencja
n
m
h
1
k
1 (mod p).
Zauwamy jednak, e
m
h
1
2
s
k
jest liczb cakowit, a wic po podniesieniu
kongruencji do potgi
p1
2
s
i skorzystaniu z Maego Twierdzenia Fermata
dostajemy
(n
m
h
1
k
)
p1
2
s
= (n
m
h
1
2
s
k
)
p1
1 (1)
p1
2
s
= 1 (mod p),
a to jest niemoliwe. Zatem rzeczywicie podzielno 2
r+1
[k jest warunk-
iem koniecznym. Wykaemy teraz, e jest to warunek wystarczajcy.
Niech k = 2
a
b, gdzie b jest nieparzyste oraz a , r + 1. W pierwszej
kolejnoci rozpatrzymy przypadek a > r + 1.
Zamy wic, e a , r +2 i przyjmijmy m = 2
ar
b +1. Podobnie jak
wczeniej, rozkadamy wyraenie m
2
r
1 z rnicy kwadratw: m
2
r
1 =
(m1)(m+1)(m
2
+1) . . . (m
2
r1
+1). Poniewa m 1 (mod 4) kady z
nawiasw, poczwszy od drugiego, zawiera liczb podzieln przez 2, ale
nie przez 4. Zatem 2 dzieli liczb m
h
1 dokadnie w a r + r = a-tej
potdze. Jasne jest rwnie, e b[m
h
1, gdy b[m1. Wynika std, e
m
h
1
k
jest liczb cakowit nieparzyst. Wystarczy wic przyj n = m1,
gdy wtedy
(m1)
m
h
1
k
+ 1 (1)
m
h
1
k
+ 1 0 (mod m).
Niech teraz a = r+1. Zauwamy, e nie moemy powtrzy rozumowa-
nia z poprzedniego przypadku, gdy jeli przyjlibymy m = 2b + 1, to
77
liczba m+1 dzielia by si przez 4. Tym razem wemy wic m = 4b
2
+1.
Wwczas tak jak poprzednio moemy sprawdzi, e liczba 2 dzieli m
h
1
w dokadnie r + 2-tej potdze. Jasne jest te, e b[m 1. A zatem
m
h
1
k
= 2c, gdzie c jest liczb cakowit nieparzyst. Przyjmujc n = 2b
dostajemy
n
m
h1
k
+ 1 = (2b)
2c
+ 1 = (4b
2
)
c
+ 1 (1)
c
+ 1 0 (mod m),
a wic tak dobrane n spenia podzielno m[n
m
h
1
k
+ 1 i rozwizanie
zadania jest zakoczone.
12. Rozstrzygn dla jakich liczb naturalnych a istnieje nieskoczenie
wiele liczb bezkwadratowych n takich, e n[a
n
1.
Rozwizanie
Udowodnimy, e jeli tylko a ,= 2, 3 to speniony jest warunek dany w
zadaniu.
W tym celu wykaemy najpierw, e a = 2 i a = 3 nie s rozwizaniami
zadania, gdy jasne jest e a = 1 spenia dany warunek.
Zamy, e n[2
n
1 i n > 1. Wemy najmniejszy dzielnik pierwszy
p liczby n. Wwczas p[2
n
1 i niech k oznacza rzd liczby 2 modulo p,
tzn. najmniejsz liczb cakowit dodatni, dla ktrej 2
k
1 (mod p).
Wtedy k[n oraz k[p 1, gdy z Maego Twierdzenia Fermata p[2
p1
1.
Ale skoro k[p1, to k < p1 i jeeli k ,= 1, to n posiada dzielnik pierwszy
mniejszy ni p, co stoi w sprzecznoci z zaoeniem o minimalnoci p. A
jeli k = 1 to p[2
1
1 = 1, czyli rwnie sprzeczno. A wic n = 1 jest
jedyn liczb naturaln, dla ktrej n[2
n
1 i jasne jest, e a = 2 nie
spenia danego warunku.
Zamy, e dla pewnej liczby bezkwadratowej n > 2 zachodzi n[3
n
1.
Wybierzmy najmniejszy dzielnik pierwszy p liczby n, ktry jest wikszy
ni 2 (taki dzielnik pierwszy istnieje, bo n > 2 i n jest niepodzielne przez
4). Wwczas p[3
n
1 i podobnie jak poprzednio niech k bdzie rzdem
liczby 3 modulo p. Wtedy k[n oraz k[p 1, czyli k < p i musi by k = 1
lub k = 2, gdy z zaoenia o minimalnoci p wynika, e k nie posiada
dzielnikw pierwszych wikszych od 2 i mniejszych od p, a jednoczenie
k nie jest podzielne przez 4 (bo n nie jest). W pierwszym przypadku
mamy p[3
1
1 = 2, a w drugim p[3
2
1 = 8, czyli w obu przypadkach
p = 2, sprzeczno. Skd wynika, e a = 3 rwnie nie jest rozwizaniem
zadania.
78
Zamy teraz, e a , 4 i skonstruujmy indukcyjnie cig p
1
, p
2
, p
3
, . . .
rnych liczb pierwszych takich, e p
1
p
2
. . . p
i
[a
p
1
p
2
...p
i
1 dla i N.
Przyjmijmy p
1
= p, gdzie p jest dowolnym dzielnikiem pierwszym liczby
a 1 > 1 i zamy, e mamy k liczb pierwszych p
1
, p
2
, . . . , p
k
, ktre
speniaj napisan wczeniej podzielno dla i = 1, 2, . . . , k. Wystar-
czy pokaza, e liczba a
p
1
p
2
...p
k
1 posiada dzielnik pierwszy p rny od
p
1
, p
2
, . . . p
k
. Wwczas bdzie wystarczyo przyj p
k+1
= p i wspomniana
podzielno zachodzi bdzie rwnie dla i = k + 1, a wic indukcyjna
konstrukcja naszego cigu zostanie zakoczona.
Przyjmijmy A = a
p
1
p
2
...p
k1
(gdy k = 1 przyjmujemy A = a) oraz
niech M =
A
p
k
1
A1
= A
p
k
1
+ A
p
k
2
+ . . . + A + 1. Jasne jest, e M jest
liczb naturaln wiksz od 1, ktra dzieli a
p
1
p
2
...p
k
1. Zauwamy, e
jeli l = 1, 2, . . . , k 1, to p
l
nie dzieli M. Rzeczywicie, wiadomo, e
p
l
[a
p
1
p
2
...p
l
1, a wic rwnie p
l
[A 1. Czyli
M = A
p
k
1
+A
p
k
2
+. . . +A + 1 1 + 1 +. . . + 1 p
k
(mod p
l
),
co dowodzi, e p
l
,[M.
Rozpatrzymy teraz przypadek p
k
= 2. Jeli k = 1, to liczba a
2

1 posiada pewien dzielnik pierwszy nieparzysty p, gdy jak nietrudno


sprawdzi, a
2
1 = (a 1)(a + 1) jest potg liczby 2 wtedy i tylko
wtedy, gdy a = 1 lub a = 3. W tym wypadku wystarczy wic przyj
p
k+1
= p
2
= p. Zamy wic, e k > 1. Moemy teraz rozoy
M = A+1 = a
p
1
...p
k1
+1 = (a+1)(a
p
1
...p
k1
1
a
p
1
...p
k1
2
+. . . a+1).
i poniewa k > 1 nietrudno sprawdzi, e liczba w drugim nawiasie
jest wiksza ni 1. Ponadto, liczba w drugim nawiasie jest niepodzielna
przez 2. Jeli 2[a to sprawa jest oczywista, a w przeciwnym razie w
tym nawiasie wystpuje suma p
1
p
2
. . . p
k1
skadnikw nieparzystych, a
poniewa liczby p
1
, p
2
, . . . , p
k
s rne i p
k
= 2, to 2 nie dzieli p
1
p
2
. . . p
k1
.
A zatem M posiada pewien nieparzysty dzielnik pierwszy p, ktry na
mocy powyszych rozwaa jest rny od p
1
, p
2
, . . . , p
k1
. Podobnie jak
poprzednio moemy zatem przyj p
k+1
= p.
Zamy wic, e p
k
,= 2. Udowodnimy, e w takim wypadku p
2
k
nie
dzieli M.
A A
p
k
a
p
1
p
2
...p
k
1 (mod p
k
),
a zatem p
k
[A1 i moemy napisa A = tp
k
+1, gdzie t Z. Wykorzys-
tamy teraz wzr dwumianowy Newtona. Zauwamy, e
M =
A
p
k
1
A 1
=
(tp
k
+ 1)
p
k
1
tp
k
79
=
(tp
k
)
p
k
+. . . +
_
p
k
2
_
(tp
k
)
2
+
_
p
k
1
_
tp
k
+ 1 1
tp
k
= (tp
k
)
p
k
1
+. . . +
p
k
1
2
t(p
k
)
2
+p
k
p
k
(mod p
2
k
),
gdy p
k
> 2, czyli
p
k
1
2
jest liczb cakowit.
Pokazalimy wic, e liczba M nie dzieli si przez adn z liczb p
1
, . . . ,
p
k1
oraz przez liczb p
k
wycznie w pierwszej potdze. Jednoczenie
zauwamy, e M , a
p
k
1
> p
k
, gdzie drug nierwno atwo pokaza in-
dukcyjnie dla a , 4 i p
k
> 2. Wynika std, e M posiada pewien dzielnik
pierwszy p nie bdcy adn z liczb p
1
, p
2
, . . . , p
k
. Jako, e M[a
p
1
p
2
...p
k
1
wystarczy, e przyjmiemy p
k+1
= p i rozwizanie zadania jest zakoczone.
Zadania nieco trudniejsze:
1. Wrd n osb niektre trjki byy razem na imprezie. Dla kadych
dwch rnych osb A i B istnieje dokadnie jedna osoba C taka, e
A, B i C byli razem na imprezie. Co wicej, jeli dla szeciu rnych
osb A, B, C, X, Y, Z trjki A, B, X, B, C, Y oraz C, A, Z byy razem na
imprezie, to rwnie X, Y, Z byli razem na imprezie. Znale wszystkie
n, dla ktrych taka sytuacja jest moliwa.
Rozwizanie
Wykaemy, e n = 2
k
1 dla pewnego k Z
+
. Najpierw pokaemy, e dla
n = 2
k
1 sytuacja opisana w zadaniu jest faktycznie moliwa. Zdeniu-
jmy operacj xor (bitowy) oznaczan przez

dla liczb od 0 do 2
k
1. Jeli
a = c
k
, c
k1
. . . , c
0
)
2
oraz b = d
k
, d
k1
, . . . , d
0
)
2
to przedstawienia a i b
w systemie dwjkowym, to a

b = e
k
, e
k1
. . . , e
0
)
2
, gdzie e
i
= (c
i
+ d
i
)
(mod 2). Zauwamy teraz, e jeli osoby x
1
, . . . , x
2
k
1
spotykay si tak,
e na jednej imprezie byli razem x
a
, x
b
oraz x
a

b
dla dowolnych a i b, to
warunki zadania s spenione. Po pierwsze

jest przemienny oraz czny,
czyli a

(a

b) = b, wic faktycznie jeli x


a
i x
b
byli z x
a

b
, to x
a
i x
a

b
byli
z x
b
oraz x
b
i x
ab
byli z x
a
. Drugi warunek jest rwnie speniony, gdy
jeli x = a

b, y = b

c, z = c

a, to x

y = a

c = a

c = z.
Naley teraz jeszcze wykaza, e dla n ,= 2
k
1 opisana sytuacja nie
jest moliwa. Zauwamy, e sytuacj z zadania moemy zinterpretowa
jako dziaanie na zbiorze osb. Niech X to zbir moliwych osb. Zdeni-
ujemy dziaanie : X X X, tak, e dla a, b X, a ,= b okrelamy
80
przez a b osob, z ktr a oraz b spotkay si na imprezie. Poniewa
istnieje dokadnie jedna taka osoba, to jest to dobrze zdeniowane dzi-
aanie. Z treci zadania wynika, e dla dowolnych a, b, c X, parami
rnych spenia:
a b = b a
(a b) a = b
jeli a b = x, b c = y oraz c a = z, to x y = z
Zauwamy, e trzeci warunek daje czno , gdy dla dowolnych a, c, x
mona dobra takie b, y, z, eby zachodzi punkt trzeci (deniujemy b =
a x, z = c a, y = (a x) c). Wwczas spenione jest:
x (a c) = x z = y = b c = (x a) c.
Zatem dowolne parami rne elementy z X s przemienne i czne.
Dziaanie bdziemy nazywa mnoeniem. Problemem jest to, e nie
mona wykona mnoenia na dwch takich samych elementach. Aby om-
in problem dodajmy dodatkowy element, zwany 0; niech dla dowolnego
a X 0 zachodzi 0 = a a oraz 0 a = a 0 = a. Bez trudu mona
wykaza, e wwczas dla dowolnych elementw a, b, c X0 zachodz
warunki:
a (b c) = (a b) c
a 0 = 0 a = a
dla dowolnego a istnieje b (element odwrotny do a), takie e a b =
b a = 0 (w naszym wypadku a = b)
Zbir z takim dziaaniem nazywany jest w matematyce grup. W naszej
grupie dowolny element pomnoony przez siebie daje 0, czyli aa = 0, w
jzyku grup mwimy, e dowolny element ma rzd rwny 2. W oglnoci
rzd elementu x jest to minimalne n takie, e x pomnoony n razy przez
siebie daje element neutralny, czyli 0.
Do wykazania, e w naszym wypadku n = 2
k
1 wystarczy pokaza,
e rozwaana grupa musi mie 2
k
elementw (bo dodalimy 0), liczb
elementw grupy nazywamy jej rzdem. Pokaemy, e gdyby rzd naszej
grupy nie by rwny 2
k
, to istniaby w niej element o rzdzie rnym od
2. Spraw t zaatwi lemat.
81
Lemat Jeeli grupa ma rzd kp, gdzie k Z, a p jest liczb pierwsz, to
istnieje w tej grupie element rzdu p.
Dowd (lematu) Rozwamy wszystkie p-tki (x
1
, . . . , x
p
) takie, e x
1

. . . x
p
= 0. Jest ich (kp)
p1
, bo pierwsze p 1 elementw wybieramy
dowolnie, a ostatni jest elementem przeciwnym do (x
1
. . . x
p1
).
Ponadto liczba takich p-tek, e x
1
= . . . = x
p
jest rwnie podzielna
przez p, gdy liczba pozostaych jest podzielna przez p (moemy je wwczas
przesuwa cyklicznie, wic podzielimy je na grupy takie, e kada p-tka
moe powsta z kadej innej; grupy te s wielkoci p).
Jednak 0 . . . 0 = 0, czyli liczba pozostaych p-tek o rwnych skad-
nikach jest niepodzielna przez p, a co za tym idzie rna od 0. Zatem
istnieje takie a, e a . . . a = 0, innymi sowy istnieje element a rzdu
p.

2. Niech punkty D, B, C, E le na jednej prostej w tej wanie
kolejnoci i niech punkt A spenia rwnoci AB = DB oraz AC = EC.
Poprowadmy dwusieczne ktw ABC oraz ACB i ich przecicia z
okrgiem opisanym na trjkcie ABC oznaczmy odpowiednio przez K i
L, za ich przecicia z przeciwlegymi bokami trjkta ABC odpowiednio
przez P i Q. Niech O
1
bdzie rodkiem okrgu opisanego na trjkcie
DBL, za O
2
rodkiem okrgu opisanego na trjkcie ECK. Przez S
oznaczmy punkt przecicia CO
1
i BO
2
. Udowodni, e AS PQ.
Rozwizanie
Oznaczmy przez R zoenie inwersji o rodku B i kwadracie promienia
BA BC z symetri wzgldem prostej BK. Z twierdzenia o dwusiecznej
R(E) = P. Oczywicie R(C) = A. Ponadto BAP = BAC = BKC
oraz ABP = KBC, czyli trjkty ABP i KBC s podobne, a std
R(K) = Q. W takim razie okrg opisany na trjkcie ECK przechodzi
na okrg opisany na trjkcie PAQ. Srd wniosek, e proste BO
2
i BO
s izogonalne wzgldem kta ABC, gdzie O jest rodkiem okrgu
opisanego na trjkcie APQ (bo rodek okrgu, rodek jego obrazu w
inwersji i rodek tej inwersji lea na jednej prostej). Analogicznie CO
1
i
CO s izogonalne wzgldem kta ACB, a wic punkty S i O s izog-
onalne wzgldem trjkta ABC, czyli proste AS i AO s izogonalne
wzgldem kta BAC. Wobec tego SAP = OAQ = 90

APQ i
mamy tez.
82
3. Dane jest n liczb rzeczywistych dodatnich x
1
, x
2
, . . . , x
n
o iloczynie
rwnym 1. Wykaza, e

1i<jn
(x
i
x
j
)
2
,
n

i=1
x
2
i
n.
Rozwizanie
Dan nierwno moemy przeksztaci do rwnowanej postaci
n , (n 1)
_
n

k=1
x
2
k
_
+
_
n

k=1
x
k
_
2
.
Dowodzimy powysz nierwno indukcyjnie. Dla n = 2 obie strony
nierwnoci s rwne. Zamy wic, e nierwno jest prawdziwa dla
n 1 i wykaemy j dla n. Niech
f(x
1
, x
2
, . . . , x
n
) = (n 1)
_
n

k=1
x
2
k
_
+
_
n

k=1
x
k
_
2
n.
Chcemy udowodni, e f(x
1
, x
2
, . . . , x
n
) 0.
Bez straty oglnoci moemy zaoy, e x
1
jest minimalna spord
liczb x
1
, x
2
, . . . , x
n
. Przyjmijmy wwczas G =
n1

x
2
x
3
. . . x
n
. Udowod-
nimy nierwno f(x
1
, x
2
, . . . , x
n
) f(x
1
, G, G, . . . , G). atwo zauway,
e jest ona rwnowana nierwnoci
(n 1)
n

k=2
x
2
k

_
n

k=2
x
k
_
2
, 2x
1
_
n

k=2
x
k
(n 1)G
_
.
Poniewa x
1
jest minimalne wiadomo, i x
1
G. Z nierwnoci midzy
redni arytmetyczn a geometryczn wiadomo rwnie, e

n
k=2
x
k
,
(n 1)G. Wystarczyoby wic wykaza, e
(n 1)
n

k=2
x
2
k

_
n

k=2
x
k
_
2
, 2G
_
n

k=2
x
k
(n 1)G
_
,
gdy x
1
zastpilimy niemniejsz liczb G, a wyraenie w nawiasie jest
nieujemne.
Niech y
k
=
x
k
G
dla k = 2, 3, . . . , n. Wwczas, iloczyn liczb y
k
jest rwny
1, a nierwno po podzieleniu obu stron przez G
2
przepisuje si jako
83
(n 1)
n

k=2
y
2
k

_
n

k=2
y
k
_
2
, 2
_
n

k=2
y
k
(n 1)
_
.
Z zaoenia indukcyjnego wiadomo, e
(n 2)
n

k=2
y
2
k
+ (n 1) ,
_
n

k=2
y
k
_
2
.
Wystarczy wic pokaza, e zachodzi
n

k=2
y
2
k
+n 1 ,
n

k=2
2y
k
,
czyli
n

k=2
(y
k
1)
2
, 0,
a to jest jasne.
Wykazalimy wic, e f(x
1
, x
2
, . . . , x
n
) f(x
1
, G, G, . . . , G). Aby do-
koczy krok indukcyjny wystarczy pokaza, e f(x
1
, G, G, . . . , G) 0.
Ale poniewa x
1
=
1
G
n1
ostatnia nierwno sprowadza si do
(n 1)
_
(n 1)G
2
+
1
G
2(n1)
_
+n ,
_
(n 1)G+
1
G
n1
_
2
lub po prostu
n 2
G
2n2
+n ,
2n 2
G
n2
,
co jest natychmiastowym wnioskiem z nierwnoci midzy redni aryt-
metyczn a geometryczn dla liczb 2n 2 liczb
1
G
,
1
G
, . . . ,
1
G
, 1, 1, . . . , 1.
Dowd kroku indukcyjnego zosta zakoczony, a wic zadanie jest rozwizane.
4. W kadym punkcie kratowym paszczyzny, ktry ma niedodatni
wsprzdn x pooono jeden pionek. Dozwolone s ruchy polegajce na
wybraniu pewnej pary pionkw stojcych na ssiadujcych w pionie lub
poziomie punktach A i B i zbicie jednym z nich drugiego, tj. zdjcie
pionka z pola A i przestawienie pionka z pola B na taki punkt C, e A
jest rodkiem odcinka BC (w punkcie C, na ktry przestawiamy pionek z
punktu B, nie mg dotychczas sta aden pionek). Znale najwiksze x,
dla ktrego istnieje skoczona sekwencja dozwolonych ruchw, po ktrej
pewien pionek znajduje si w punkcie (x, y) (dla pewnego y).
84
Rozwizanie
Pokaemy, e istnieje sekwencja ruchw doprowadzajca pionek na pole
(4, 0) oraz, e dla dowolnego x nie istnieje sekwencja ruchw doprowadza-
jca pionek na pole (5, x).
Do zapisania szukanej sekwencji uyjemy notacji (x
1
, y
1
)(x
2
, y
2
)
(x
3
, y
3
) mwicej, e pionek z pola o wsprzdnych (x
1
, y
1
) zbija pionek
z pola (x
2
, y
2
) i zajmuje pole (x
3
, y
3
). Sekwencja doprowadzajca do pi-
onka na polu (4, 0) to:
(1, 1), (0, 1) (1, 1)
(1, 2), (0, 2) (1, 2)
(1, 2), (1, 1) (1, 0)
(0, 0), (1, 0) (2, 0)
(0, 2), (0, 1) (0, 0)
(1, 0), (0, 0) (1, 0)
(1, 0), (2, 0) (3, 0)
(3, 1), (2, 1) (1, 1)
(3, 2), (2, 2) (1, 2)
(1, 2), (1, 1) (1, 0)
(2, 0), (1, 0) (0, 0)
(1, 2), (1, 1) (1, 0)
(1, 0), (0, 0) (1, 0)
(4, 0), (3, 0) (2, 0)
(3, 2), (3, 1) (3, 0)
(3, 0), (2, 0) (1, 0)
(2, 2), (2, 1) (2, 0)
(2, 0), (1, 0) (0, 0)
(0, 0), (1, 0) (2, 0)
(2, 0), (3, 0) (4, 0)
Naley teraz wykaza, e do pola postaci (5, x) nie da si doj. Udowod-
nimy to nie wprost. Przypumy przeciwnie, bez straty oglnoci moemy
zaoy, e jest to pole (5, 0). Przypiszmy kademu polu na szachownicy
liczb, polu (x, y) przypiszemy liczb
d(x,y)
, gdzie =

51
2
, d(x, y) za
jest rwna odlegoci (x, y) w metryce miejskiej od (5, 0), czyli d(x, y) =
[x 5[ + [y[. Liczba pojawia si tu, gdy jest rozwizaniem rwnania
x
2
+ x = 1. Udowodnimy zaraz, e na pocztku suma liczb na polach,
na ktrych stoj pionki wynosi 1. Zauwamy, e po ruchu suma liczb
na polach opionkowanych nie zwikszy si, gdy jeeli pionek zbijajcy
85
i pionek zbijany stay na polach o liczbach
k
oraz
k+1
, to nowy pi-
onek stoi na polu o liczbie maksymalnie
k1
(jego odlego od pola
(5, 0) wynosi co najmniej k 1). Mamy wwczas
k+1
+
k
=
k1
, wic
suma liczb na polach opionkowanych si zachowuje. Moe si oczywicie
zdarzy, e suma liczb na tych polach si zmniejszy, gdy ruch bdzie nie
w kierunku (5, 0). Czyli reasumujc suma liczb na polach opionkowanych
jest pniezmiennikiem. Zauwamy, e gdy na polu (5, 0) stoi pionek, to
dodaje on do sumy liczb na opionkowanych polach liczb
0
= 1, czyli po
wykazaniu, e na starcie suma liczb na opionkowanych polach wynosi 1
pokaemy, e wszystkie musz by wykorzystane do uzyskania pionka na
(5, 0), zatem w skoczonej iloci ruchw taka sytuacja jest niemoliwa. Do
zakoczenia dowodu wystarczy pokaza, e suma liczb na polach (x, y),
gdzie y 0 wynosi 1.
Obliczmy najpierw sum liczb w kolumnie o drugiej wsprzdnej k.
Oznaczmy j przez S
k
.
S
k
=

i=5k

i
+

i=6k

i
=
5k
1
1
+
6k
1
1
.
Przez S oznaczmy szukan sum. Wwczas
S =
0

k=
S
k
=

k=0

5+k
1
1
+

k=0

6+k
1
1
=
_
1
1
_
2
(
5
+
6
)
=
4
(3

5)
2

5 + 1
2
_

5 1
2
_
5
= . . . = 1,
gdzie ostatnia rwno wynika z szeregu elementarnych przeksztace
uamkw.
5. Niech P bdzie wielomianem o wspczynnikach cakowitych, a n
liczb naturaln. Dla kadej liczby cakowitej dodatniej m liczba P(2
m
)
jest n-t potg pewnej liczby naturalnej. Udowodni, e wwczas P(x) =
(Q(x))
n
dla pewnego wielomianu Q o wspczynnikach cakowitych.
Rozwizanie
W rozwizaniu wykorzystamy 4 lematy. Zaczniemy od wypowiedzenia
znanego Lematu Gaussa.
Lemat (Gauss). Jeli F(x) jest wielomianem o wspczynnikach cakow-
itych, G(x) i H(x) s wielomianami o wspczynnikach wymiernych oraz
86
F(x) = G(x) H(x), to istnieje taka liczba wymierna q, e qG(x) i
1
q
H(x)
s wielomianami o wspczynnikach cakowitych (w szczeglnoci F jest
rozkadalny na iloczyn wielomianw o wspczynnikach cakowitych).
Udowodnimy teraz
Lemat (1). Jeli F(x) jest takim wielomianem o wspczynnikach ze-
spolonych, e jego wspczynnik wiodcy jest liczb cakowit (czyli te
rzeczywist) oraz wielomian (F(x))
k
posiada wspczynniki wymierne (czyli
te rzeczywiste) dla pewnego k Z
+
, to wspczynniki F s liczbami
wymiernymi (czyli te rzeczywistymi). Co wicej, jeli dla pewnego k
Z
+
wspczynniki wielomianu (F(x))
k
s cakowite, to rwnie wspczyn-
niki F s liczbami cakowitymi.
Dowd. Na pocztku udowodnimy pierwsz cz lematu, czyli pokaemy,
e wielomian F posiada wspczynniki rzeczywiste wymierne przy zaoe-
niu, e (F(x))
k
Q[x] dla pewnego k. Niech
F(x) = a
n
x
n
+a
n1
x
n1
+. . . +a
1
x +a
0
oraz
(F(x))
k
= b
kn
x
kn
+b
kn1
x
kn1
+. . . +b
1
x +b
0
.
Z zaoenia a
n
Q. Przyjmijmy wic, e wymierne (czyli te rzeczy-
wiste) s wspczynniki a
n
, a
n1
, . . . , a
nl
i pokamy, e o ile tylko l < n
to rwnie a
nl1
Q. Zobaczmy jak wyglda wspczynnik b
knl1
w
zalenoci od wspczynnikw wielomianu F. Po rcznym wymnoeniu
wszystkich nawiasw nietrudno zauway, e b
knl1
= a
nl1
a
k1
n
+S
gdzie S to suma pewnych iloczynw, w ktrych wystpuj wycznie
potgi liczb a
n
, a
n1
, . . . , a
nl
. Jednak na mocy zaoenia te liczby s
wymierne, a wic S rwnie jest wymierne. Poniewa wymierne s take
liczby a
n
oraz b
knl1
stwierdzamy, e wymierna jest te liczba a
nl1
co
pokazuje, e rzeczywicie F(x) Q[x].
Zaomy teraz, e (F(x))
k
ma wspczynniki cakowite i pokamy,
e F rwnie ma wspczynniki cakowite. Z dowodu poprzedniej czci
lematu wiemy ju, e F ma wspczynniki wymierne. Jeli k = 1, to
nie ma czego dowodzi. Zamy wic, e k > 1 i zapiszmy (F(x))
k
=
F(x) (F(x))
k1
. Wielomian (F(x))
k
o wspczynnikach cakowitych za-
pisalimy jako iloczyn wielomianw o wspczynnikach wymiernych, a
wic z Lematu Gaussa istnieje taka liczba wymierna q, e qF(x) Z[x]
oraz
1
q
(F(x))
k1
Z[x]. Jeli q =
a
b
, gdzie a, b Z oraz (a, b) = 1, to
w szczeglnoci aF(x) Z[x] oraz b(F(x))
k1
Z[x]. Ale wtedy te
87
a
k1
(F(x))
k1
Z[x], a poniewa (a, b) = 1, to rwnie (a
k1
, b) = 1.
Istniej wic takie s, t Z, e a
k1
s +bt = 1. Mamy wwczas
sa
k1
(F(x))
k1
+tb(F(x))
k1
= (F(x))
k1
Z[x].
Powtarzajc to rozumowanie otrzymujemy kolejno, e (F(x))
k2
,
(F(x))
k3
, . . . , F(x) Z[x], co koczy dowd lematu.
Lemat (2). Jeli F(x) Z[x] jest stopnia nk (gdzie k N), o wspczyn-
niku wiodcym a
n
gdzie a Z oraz F(t) jest n-t potg liczby cakowitej
dla nieskoczenie wielu t N, to F(x) = (G(x))
n
dla pewnego G(x) o
wspczynnikach cakowitych.
Dowd. Jasne jest, e jeli 2[n, to wspczynnik wiodcy wielomianu
F jest liczb dodatni. Jeli 2 nie dzieli n i wspczynnik wiodcy wielo-
mianu F jest ujemny, to moemy rozwaa wielomian F(x). W kadym
bd razie moemy przyj, e wspczynnik wiodcy jest dodatni i co
za tym idzie F(x) > 0 dla odpowiednio duych x.
Zauwamy na pocztek, e moemy skonstruowa taki wielomian G(x)
Q[x] o stopniu k, e k + 1 najwyszych wspczynnikw wielomianu
(G(x))
n
pokrywa si z odpowiadajcymi wspczynnikami F(x). Istot-
nie, przyjmujemy, e a jest wspczynnikiem wiodcym wielomianu G i
kolejne wspczynniki wyliczamy za pomoc metody, ktr zastosowal-
imy w pierwszym kroku dowodu lematu (1). Kady wspczynnik G
jest wymierny, gdy otrzymujemy go z rwnania liniowego o wymiernych
wspczynnikach.
Pokaemy teraz, e F(x) = (G(x))
n
. Dla dowodu nie wprost zamy,
e te wielomiany nie s sobie rwne. Wwczas istnieje skoczenie wiele x,
dla ktrych F(x) = (G(x))
n
, a jednoczenie istnieje nieskoczenie wiele
t N takich, e
n
_
F(t) Z. W szczeglnoci istnieje wic nieskoczenie
wiele r N, dla ktrych G(r) ,=
n
_
F(r) Z. Jeli M oznacza najmniejsz
wspln wielokrotno mianownikw wspczynnikw G, to MG(r) Z,
a zatem MG(r) oraz M
n
_
F(r) rni si o co najmniej 1, gdy s to
dwie rne liczby cakowite. Otrzymujemy wic [G(r)
n
_
F(r)[ ,
1
M
.
Dostaniemy sprzeczno z t nierwnoci dobierajc odpowiednio due
r.
Poniewa k +1 najwyszych wspczynnikw wielomianu (G(x))
n
po-
krywa si z odpowiadajcymi wspczynnikami F(x) wielomian (G(x))
n

F(x) ma stopie co najwyej nk k 1. A zatem istnieje taka staa c


1
,
88
e [G(r))
n
F(r)[ < c
1
r
nkk1
dla odpowiednio duego r. Jednoczenie
dla odpowiednio duych r zachodzi F(r) > 0, a wic rwnie G(r) > 0.
Dla odpowiednio duego r mamy wic
[(G(r))
n
F(r)[ = [G(r)
n
_
F(r)[ [(G(r))
n1
+. . . + (
n
_
F(r))
n1
[
,
1
M
[G(r)[
n1
,
a poniewa G(x) jest stopnia k istnieje taka staa c
2
> 0, e dla odpowied-
nio duych r mamy [G(r)[
n1
> c
2
r
knk
. A wic mamy rwnie r <
Mc
1
c
2
, co jednak nie jest prawd dla odpowiednio duych r. Otrzymana
sprzeczno pokazuje, e w istocie F(x) = (G(x))
n
. Z lematu (1) otrzy-
mujemy natychmiast, e G(x) ma wspczynniki cakowite i dowd jest
zakoczony.
Lemat (3). Jeli F(x) jest wielomianem o wspczynnikach cakowitych
o wspczynniku wiodcym a i 1 < a
1
< a
2
< . . . < a
n1
s liczbami
cakowitymi takimi, e
F(x)F(a
1
x)F(a
2
x) . . . F(a
n1
x) = (G(x))
n
,
dla pewnego G(x) Z[x], to wwczas F(x) = ax
m
(H(x))
n
dla pewnej
nieujemnej liczby cakowitej m i pewnego wielomianu H(x) Q[x].
Dowd. Wykaemy, e kady niezerowy pierwiastek zespolony wielo-
mianu F wystpuje w krotnoci podzielnej przez n. Zamy wic, e tak
nie jest i spord wszystkich pierwiastkw niespeniajcych tego warunku
wybierzmy ten o najmniejszym module, niech bdzie to z. Wwczas
z
a
n1
jest pierwiastkiem F(a
n1
x) i jego krotno rwnie jest niepodzielna
przez n. Zauwamy te, e jeli
z
a
n1
jest pierwiastkiemF(a
i
x) dla pewnego
1 i < n 1, to wwczas wystpuje w krotnoci podzielnej przez n.
Bowiem gdyby tak nie byo to
a
i
z
a
n1
byoby niezerowym pierwiastkiem F
o krotnoci niepodzielnej przez n. Ale skoro a
i
< a
n1
, to [
a
i
z
a
n1
[ < [z[,
sprzeczno z minimalnoci z. Z tych samych powodw, jeli
z
a
n1
jest
pierwiastkiem F(x) to w krotnoci podzielnej przez n.
Pokazalimy zatem, e
z
a
n1
jest pierwiastkiem wielomianu
F(x)F(a
1
x)F(a
2
x) . . . F(a
n1
x) o krotnoci niepodzielnej przez n. Ale
jest to natychmiastowa sprzeczno z faktem, e jest to rwnie pier-
wiastek wielomianu (G(x))
n
.
89
Oznaczmy przez m krotno 0 w F(x). Poniewa wszystkie pierwiastki
F oprcz zera wystpuj w krotnoci podzielnej przez n moemy za-
pisa F(x) = ax
m
(H(x))
n
dla pewnego unormowanego wielomianu H
o wspczynnikach zespolonych. Wielomian
F(x)
ax
m
posiada wspczynniki
wymierne i jest rwny (H(x))
n
, a poniewa H ma wspczynnik wiodcy
1 na mocy lematu (1) stwierdzamy, e H Q[x] i lemat (3) jest dowiedziony.
Przejdmy do waciwej czci rozwizania. Rozwamy wielomian
Q(x) = P(x)P(2
n
x)P(2
2n
x) . . . P(2
(n1)n
x).
Niech k bdzie stopniem wielomianu P, natomiast a jego wspczyn-
nikiem wiodcym. Wwczas wspczynnikiem wiodcym wielomianu Q
jest (a2
k+2k+...+(n1)k
)
n
, a jego stopie to nk. Jasne te jest, e dla kadego
m Z
+
liczba Q(2
m
) jest n-t potg liczby cakowitej, gdy jest iloczynem
n-tych potg liczb cakowitych. A zatem, na mocy lematu (2) istnieje taki
wielomian G Z[x], e Q(x) = (G(x))
n
. Z kolei z lematu (3) otrzymu-
jemy, e wwczas P(x) = ax
m
(H(x))
n
dla pewnego H o wspczynnikach
wymiernych.
Jeli P(x) 0 to teza jest oczywicie speniona. Jeli P(x) , 0, to
moemy wybra takie l, e P(2
ln
) ,= 0. Wstawiajc wic x = 2
nl
do
powyszej rwnoci dostajemy, e a jest n-t potg liczby wymiernej,
a skoro a Z, to a jest n-t potg liczby cakowitej. Zapiszmy wic
a = b
n
. Moemy wybra takie l, e P(2
ln+1
) ,= 0. Dostajemy wwczas,
e b
n
2
m(ln+1)
(H(2
ln+1
))
n
jest n-t potg liczby cakowitej. Czyli 2
m(ln+1)
jest n-t potg liczby wymiernej, ale tak jak poprzednio stwierdzamy,
e musi to te by n-ta potga liczby cakowitej a wic n[m, czyli m = nt
dla t N. Aby zakoczy dowd wystarczy zauway, e wielomian P(x)
jest n-t potg wielomianu bx
t
H(x) o wspczynnikach wymiernych. Ale
skoro P ma wspczynniki cakowite to na mocy lematu (1) wielomian
bx
t
H(x) posiada wspczynniki cakowite i dowd jest zakoczony.
6. Niech S
1
, S
2
bd okrgami przecinajcymi si w dwch rnych
punktach A i B. Prosta przechodzca przez punkt A przecina okrg S
1
w punkcie C, a okrg S
2
w punkcie D. Punkty M, N, K le odpowied-
nio na odcinkach CD, BC, BD oraz prosta MN jest rwnolega do
BD, a prosta MK jest rwnolega do BC. uki BC okrgu S
1
oraz BD
okrgu S
2
zawieraj odpowiednio punkty E i F, przy czym prosta EN
jest prostopada do BC, a prosta FK jest prostopada do BD. Dowie,
e kt EMF jest prosty.
90
Rozwizanie
Oznaczmy przez O
1
, O
2
rodki okrgw S
1
, S
2
. Zachodz rwnoci
BO
1
C = 2BAC = 360

2DAB = 360

DO
2
B.
Std wynika, e istnieje podobiestwo o orientacji ujemnej, ktre przepro-
wadza S
1
w S
2
, B na D, a C na B. Z twierdzenia Talesa
BK
KD
=
CM
MD
=
CN
NB
,
wic punkt N przechodzi w tym podobiestwie na punkt K. Niech E

bdzie obrazem punktu E w tyme podobiestwie. Wtedy oczywicie


punkty E

, K, F s wspliniowe i z twierdzenia o potdze punktu wzgl-


dem okrgu mamy
FK
KB
=
DK
KE

=
BN
NE
,
a std
FK
KM
=
FK
BN
=
BK
NE
=
MN
NE
.
Ponadto KFBK[[MN oraz KM[[BNNE, wic trjkty FKM i
MNE s podobne przez zoenie jednokadnoci z obrotem o kt prosty.
Musi by rwnie EMMF i mamy tez.
7. Udowodni, e istnieje liczba postaci 333333
333333
n
, gdzie n jest
liczb naturaln, zakoczona 333333
333333
trjkami w zapisie dziesitnym.
Rozwizanie
Dla danej liczby cakowitej dodatniej a niech v
5
(a) oznacza liczb cakow-
it nieujemn n, dla ktrej 5
n
[[a (tzn. tak liczb n, e 5
n
[a ale 5
n+1
nie
dzieli n).
Lemat (1). Jeli v
5
(a 1) > 0, to v
5
(a
n
1) = v
5
(a 1) + v
5
(n) gdzie
a, n Z
+
i a > 1.
Dowd. Przeprowadzimy dowd indukcyjny ze wzgldu na v
5
(n). Jeli
v
5
(n) = 0, to zauwamy, e a
n
1 = (a1)(a
n1
+a
n2
+. . . +a+1) i w
drugim nawiasie wystpuje dokadnie n czynnikw. Jednak skoro a 1
(mod 5), a n jest niepodzielne przez 5 to jasne jest, e liczba w drugim
nawiasie nie dzieli si przez 5, a wic w tym wypadku v
5
(a
n
1) =
v
5
(a 1).
Zamy wic, e teza zachodzi dla v
5
(n) = k , 0 i wykaemy j
dla v
5
(n) = k + 1. Zapiszmy n = 5
k+1
b, gdzie b jest liczb cakowit
niepodzieln przez 5. Na mocy zaoenia indukcyjnego wiemy, e a
5
k
b
=
91
5
k+l
c + 1 gdzie c Z jest liczb niepodzieln przez 5 i dla uproszczenia
przyjlimy l = v
5
(a 1). Podnoszc jednak t rwno do potgi 5
a
5
k+1
b
= (5
k+l
c + 1)
5
= 5
5k+5l
c
5
+ 5
4k+4l+1
c
4
+ 2 5
3k+3l+1
c
3
+2 5
2k+2l+1
c
2
+ 5
k+l+1
c + 1.
Na mocy zaoenia l > 0, a wic atwo zauway, e pierwsze 4 skadniki
powyszej sumy s podzielne przez 5
k+l+2
. A zatem
a
5
k+1
b
1 5
k+l+1
c (mod 5
k+l+2
),
czyli v
5
(a
5
k+1
b
1) = k + l + 1, gdy c jest niepodzielne przez 5. A
dokadnie to chcielimy wykaza.
Niech A = 333333.
Lemat (2). Niech k , 4 bdzie liczb naturaln. Wwczas 2
k
[[A
510
k2
1
oraz 5
k
[[A
510
k2
1.
Dowd. Niech B = A
5
k1
. Poniewa A 5 (mod 8) mamy te B 5
(mod 8) oraz
A
510
k2
1 = B
2
k2
1 = (B 1)(B + 1)(B
2
+ 1) . . . (B
2
k3
+ 1).
Pierwszy nawias dzieli si przez 4 ale nie przez 8, a pozostae k 2
nawiasw dzieli si przez 2 ale nie przez 4. A zatem cay iloczyn dzieli
si dokadnie przez 2
k
.
Zauwamy teraz, e A
510
k2
1 = (A
4
)
2
k4
5
k1
1. Nietrudno sprawdzi,
e v
5
(A
4
1) = 1. Poniewa k , 4 i 2
k4
Z moemy wykorzysta
udowodniony wczeniej lemat by otrzyma
v
5
_
(A
4
)
2
k4
5
k1
1
_
= v
5
(A
4
1) +v
5
(2
k4
5
k1
) = 1 +k 1 = k,
co chcielimy wykaza.
Lemat (3). Niech k , 4 bdzie liczb naturaln. Wwczas A
m
A
n
(mod 10
k
) wtedy i tylko wtedy, gdy m n (mod 5 10
k2
).
Dowd. Kongruencja A
m
A
n
(mod 10
k
) rwnowana jest kongru-
encji A
mn
1 (mod 10
k
). Jeli 5 10
k2
[mn, to ze wzgldu na lemat
(2) jasne jest, e owa kongruencja jest speniona. Z drugiej jednak jeli
ta podzielno nie zachodzi to niech 2
a
[[mn oraz 5
b
[[mn. Wwczas
92
a < k 2 lub b < k 1. W pierwszym przypadku jeli zapiszemy
m n = 2
a
c, gdzie 2 nie dzieli c i rozoymy wyraenie A
mn
1
za pomoc rnicy kwadratw, tak jak w dowodzie lematu 2 to do-
jdziemy do wniosku, e 2
a+2
[[A
mn
1. Czyli 2
k
nie dzieli A
mn
1,
bo a + 2 < k 2 + 2 = k. Jeli a , k 2 , 2 oraz b < k 1, to
zapiszmy m n = 4 5
b
d gdzie d jest niepodzielne przez 5. Wwczas
A
mn
1 = (A
4
)
d5
b
1. Poniewa v
5
(A
4
1) = 1 na mocy lematu (1)
mamy
v
5
_
(A
4
)
d5
b
1
_
= v
5
(A 1) +v
5
(d 5
b
) = 1 +b < 1 +k 1 = k.
A wic 5
k
nie dzieli A
mn
1. W obu przypadkach otrzymalimy, e 10
k
nie dzieli A
mn
1, a zatem teza lematu zostaa dowiedziona.
Lemat (4). Niech k, m, R bd liczbami naturalnymi takimi, e k , 4,
0 < R < 10
k
oraz A
m
R (mod 10
k
). Wwczas dla dowolnego 0 c <
10 istnieje takie 0 t < 10 e A
t510
k2
+m
c 10
k
+R (mod 10
k+1
).
Dowd. Rozwamy liczby A
t
= A
t510
k2
+m
dla t = 0, 1, 2, . . . 9. Na
mocy lematu (3) A
t
R (mod 10
k
) dla t = 0, 1, 2, . . . , 9, gdy wszystkie
wykadniki daj t sam reszt modulo 5 10
k2
. Z lematu (3) wynika
rwnie, e reszty z dzielenia przez 10
k+1
tych liczb s parami rne, gdy
rne s reszty z dzielenia wykadnikw przez 510
k1
. Jako, e tych liczb
jest 10 jasne jest, e musz to by wanie reszty 0 10
k
+ R, 1 10
k
+
R, . . . , 9 10
k
+R.
Przystpujemy do waciwej czci rozwizania. Oznaczmy A = 333333.
Wykaemy indukcyjnie, e dla kadej liczby naturalnej k istnieje takie n,
e liczba A
A
n
posiada co najmniej k trjek kocowych.
Dla k = 6 sprawa jest jasna, gdy wystarczy przyj n = 0. Zamy
wic, i dla pewnego n liczba A
A
n
koczy si co najmniej k trjkami. Z
lematu (4) wiadomo, e dla pewnego 0 l < 10 mamy
A
l510
k2
+A
n
3 10
k
+
k
..
33 . . . 3
k+1
..
33 . . . 3 (mod 10
k+1
),
czyli liczba A
l510
k2
+A
n
koczy si przynajmniej k + 1 trjkami. Na
mocy lematu (3) wystarczy wic znale takie m, e A
m
l 510
k2
+A
n
(mod 510
k1
). Wwczas bowiem bdzie A
A
m
A
l510
k2
+A
n
(mod 10
k+1
)
i liczba A
A
m
bdzie koczya si co najmniej k +1 trjkami. Znajdziemy
takie m, e nawet A
m
l 5 10
k2
+A
n
(mod 10
k
).
93
Niech 0 < R
1
< 10
k2
bdzie reszt z dzielenia liczby A
n
przez 10
k2
,
a 0 < R
2
< 10
k1
reszt z dzielenia liczby l 5 10
k2
+ A
n
przez 10
k1
.
Na mocy lematu (4) istnieje m
1
takie, e
A
m
1
c
1
10
k2
+R
1
(mod 10
k1
),
gdzie c
1
to reszta z dzielenia liczby 5l +
A
n
R
1
10
k2
przez 10 (jest to liczba
cakowita ze wzgldu na okrelenie R
1
).
Wwczas
A
m
1

_
10s + 5l +
A
n
R
1
10
k2
_
10
k2
+R
1
l 5 10
k2
+A
n
R
1
+R
1
R
2
(mod 10
k1
).
Ponownie z lematu (4) wynika istnienie takiego m
2
, e
A
m
2
c
2
10
k1
+R
2
(mod 10
k
),
gdzie c
2
jest reszt z dzielenia liczby
l510
k2
+A
n
R
2
10
k1
przez 10. Analog-
iczny rachunek jak przedtem pokazuje, e wtedy A
m
2
l 5 10
k2
+A
n
(mod 10
k
). Czyli zadanie jest rozwizane.
8. Wielomiany W i V nazywamy wzgldnie pierwszymi jeli nie istnieje
wielomian U stopnia dodatniego taki, e U[W i U[V . Niech P, Q i R bd
wzgldnie pierwszymi wielomianami stopnia dodatniego. Udowodni, e
jeeli zachodzi
_
P(x)
_
n
+
_
Q(x)
_
n
=
_
R(x)
_
n
,
to n 2.
Rozwizanie
Lemat
Dane s wzgldnie pierwsze wielomiany stopnia dodatniego A, B, C
takie, e
A(t) = a
k

i=1
(t x
i
)

i
, B(t) = b
l

i=1
(t y
i
)

i
, C(t) = c
m

i=1
(t z
i
)

i
,
gdzie a, b, c, x
1
, . . . x
k
, y
1
, . . . , y
l
, z
1
, . . . z
m
s dowolnymi liczbami zespolo-
nymi, a w wykadnikach wystpuj liczby cakowite dodatnie (zasadnicze
twierdzenie algebry mwi, e kady wielomian jest takiej wanie postaci)
oraz A +B = C.
94
Wwczas k +l +m > maxdeg(A), deg(B), deg(C).
Dowd
Rwno A +B = C rniczkujemy stronami:
A(t)
k

i=1

i
t x
i
+B(t)
l

i=1

i
t y
i
= C(t)
m

i=1

i
t z
i
A(t)
_
k

i=1

i
t x
i

i=1

i
t z
i
_
= B(t)
_
m

i=1

i
t z
i

i=1

i
t y
i
_
,= 0.
Niech D(t) =

k
i=1
(t x
i
)

l
i=1
(t y
i
)

m
i=1
(t z
i
). Mamy teraz rwno
wielomianw:
A(t)D(t)
_
k

i=1

i
t x
i

i=1

i
t z
i
_
= B(t)D(t)
_
m

i=1

i
t z
i

i=1

i
t y
i
_
.
Ale A i B s wzgldnie pierwsze, czyli A(t)[D(t)
_

m
i=1

i
tz
i

l
i=1

i
ty
i
_
.
W szczeglnoci deg(A) < deg(D). Analogicznie deg(B) < deg(D) i
deg(C) < deg(D). Wystarczy teraz zauway, e deg(D) = k +l +m.

Podstawmy teraz A = P
n
, B = Q
n
i C = R
n
. Mamy:
3 maxdeg(P), deg(Q), deg(R) , deg(P) + deg(Q) + deg(R) ,
, k +l +m > n maxdeg(P), deg(Q), deg(R),
czyli n < 3.
9. S
1
, S
2
, ..., S
2008
s podzbiorami zbioru 1, 2, . . . , 2008, z ktrych
kady posiada parzyst liczb elementw. Dowie, e dla pewnych 1
i < j 2008 zbir S
i
S
j
rwnie posiada parzyst liczb elementw.
Rozwizanie
Dla potrzeb tego rozwizania przyjmijmy, e wszystkie rwnoci rozwaamy
(mod 2). Dla A, B 1, 2, . . . , 2008 wprowadmy oznaczenie:
A, B) := [A B[.
Zauwamy, e spenione s rwnoci: A, B + C) = A, B) + A, C)
oraz A, B) = B, A) dla wszystkich A, B, C 1, 2, . . . , 2008, gdzie
B + C = (B C) (B C). atwo te zobaczy, e [S
i
+ S
j
[ = 0, dla
95
1 i, j 2008, skd przez indukcje wynika, e [S
i
1
+ . . . + S
i
k
[ = 0 dla
1 i
1
, . . . , i
k
2008. Wszystkich zbiorw postaci S
i
1
+ . . . + S
i
k
jest
2
2008
, jednak kady z nich ma parzycie wiele elementw, a , takich pod-
zbiorw zbir 1, 2, . . . , 2008 ma 2
2007
, skd wniosek, e S
i
1
+. . . +S
i
k
=
S
i
k+1
+. . .+S
i
l
dla pewnych i
1
, . . . , i
l
, bez straty oglnoci parami rnych.
W takim razie S
i
1
+. . . +S
i
l
= . Rozpatrzmy przypadki:
Przypadek 1
l jest parzyste. Wtedy:
0 = S
i
1
, ) = S
i
1
,
l

j=1
S
i
j
) = S
i
1
, S
i
1
) +
l

j=2
S
i
1
, S
i
j
) =
l

j=2
S
i
1
, S
i
j
).
Poniewa po prawej stronie jest nieparzycie wiele skadnikw, wic co
najmniej jeden z nich jest zerem, co daje tez.
Przypadek 2
l jest nieparzyste. Niech i 1, 2, . . . , 2008 i
1
, . . . , i
l
,= , bo l jest
nieparzyste. Wtedy:
0 = S
i
, ) = S
i
,
l

j=1
S
i
j
) =
l

j=1
S
i
, S
i
j
).
Poniewa po prawej stronie jest nieparzycie wiele skadnikw, wic co
najmniej jeden z nich jest zerem, co daje tez.
10. W nierwnoramiennym trjkcie ABC punkty M
a
, M
b
, M
c
oz-
naczaj odpowiednio rodki bokw BC, CA, AB. Punkt I jest rodkiem
okrgu wpisanego w ten trjkt, a punkty A

, B

, C

s punktami sty-
cznoci tego okrgu odpowiednio z bokami BC, CA, AB. Prosta k jest
prost symetryczn do prostej BC wzgldem prostej AI, a prosta l jest
prost prostopada do prostej IM
a
i przechodzc przez punkt A

. X
a
jest
punktem przecicia prostych k i l, a punkty X
b
i X
c
deniujemy analog-
icznie. Wykaza, e punkty X
a
, X
b
, X
c
le na jednej prostej, ktra jest
styczna do okrgu wpisanego w trjkt ABC.
Rozwizanie
Wykaemy, e kady z punktw X
a
, X
b
, X
c
ley na wsplnej stycznej
okrgu wpisanego w trjkt ABC i okrgu opisanego na trjkcie M
a
M
b
M
c
(okrg Feuerbacha). Wystarczy to pokaza dla X
a
; dowd dla pozostaych
96
punktw bdzie analogiczny. Bdziemy rozwaa biegunowe punktw wzgl-
dem okrgu wpisanego. Z warunkw zadania wynika, e prosta l jest
biegunow punktu M
a
, a prosta k jest biegunow swojego punktu sty-
cznoci do okrgu wpisanego. Aby dosta tez, wystarczy pokaza, e
prosta przechodzca przez te dwa punkty przechodzi rwnie przez punkt
stycznoci okrgw wpisanego i Feuerbacha. Rozwamy inwersj o rodku
M
a
i promieniu M
a
A

. Potga punktu M
a
wzgldem okrgu wpisanego
jest rwna kwadratowi promienia inwersji, a wic okrg ten przechodzi na
siebie. Okrg Feuerbacha przechodzi na pewn prost. Poniewa spodek
H
a
wysokoci opuszczonej z punktu A ley za okrgu Feuerbacha, wic
punkt I
a
przecicia prostych BC i AI ley na jego obrazie, gdy:
M
a
H
a
= [M
a
C H
a
C[ = [
a
2

a
2
+b
2
c
2
2a
[ = [
b
2
c
2
2a
[,
M
a
I
a
= [M
a
C I
a
C[ = [
a
2

ab
b+c
[ = [
a(bc)
2(b+c)
[,
M
a
A

= [M
a
C A

C[ = [
a
2

a+bc
2
[ = [
bc
2
[,
czyli M
a
H
a
M
a
I
a
= M
a
A
2
. Ponadto kierunek prostej bdcej obrazem
powinien by taki, aby bya ona antyrwnolega do prostej M
b
M
c
wzgl-
dem kta M
b
M
a
M
c
(czyli rwnolega do obrazu w symetrii wzgldem
dwusiecznej). Aby to zobaczy wystarczy rozway obrazy punktw M
b
i M
c
. Poniewa teraz mwimy tylko o kierunkach, wic moemy zamieni
proste na proste do nich rwnolege i powiedzie, e szukana prosta
powinna by antyrwnolega do prostej BC wzgldem kta BAC, czyli
rwnolega do prostej k. Ale te dwie proste maj punkt wsplny I
a
, wic
si pokrywaj. Prosta k jest styczna do okrgu wpisanego; w takim razie
styczny jest rwnie okrg Feuerbacha i punkty stycznoci okrgu Feuer-
bacha i prostej k do okrgu wpisanego lea na jednej prostej z punktem
M
a
, co koczy dowd.
11. Udowodni, e dla dodatnich liczb a, b, c zachodzi nierwno

2a
a +b
+

2b
b +c
+

2c
c +a
3.
Rozwizanie
Jak atwo zauway dan nierwno moemy przeksztaci rwnowanie
c +a
2(a +b +c)

2a
(a +b)(c +a)
2
+
a +b
2(a +b +c)

2b
(b +c)(a +b)
2
97
+
b +c
2(a +b +c)

2c
(c +a)(b +c)
2

3
2(a +b +c)
.
Z nierwnoci Jensena dla wklsej funkcji

x oraz wag
c+a
2(a+b+c)
,
a+b
2(a+b+c)
,
b+c
2(a+b+c)
otrzymujemy
c +a
2(a +b +c)

2a
(a +b)(c +a)
2
+
a +b
2(a +b +c)

2b
(b +c)(a +b)
2
+
b +c
2(a +b +c)

2c
(c +a)(b +c)
2

_
1
a +b +c
_
a
(a +b)(c +a)
+
b
(b +c)(a +b)
+
c
(c +a)(b +c)
_
.
Pozostaje wic wykaza, e otrzymane wyraenie jest niewiksze ni
3
2(a+b+c)
. Po podniesieniu do kwadratu i pomnoeniu przez 4(a + b + c)
2
widzimy, e jest to rwnowane nierwnoci
4(a +b +c)
_
a
(a +b)(c +a)
+
b
(b +c)(a +b)
+
c
(c +a)(b +c)
_
9,
a po pomnoeniu obu stron nierwnoci przez (a + b)(b + c)(c + a) i
otworzeniu wszystkich nawiasw
8(a
2
b +ab
2
+b
2
c +bc
2
+c
2
a +ca
2
) + 24abc
9(a
2
b +ab
2
+b
2
c +bc
2
+c
2
a +ca
2
) + 18abc
i po zredukowaniu wyrazw podobnych
6abc a
2
b +ab
2
+b
2
c +bc
2
+c
2
a +ca
2
,
co jest natychmiastowym wnioskiem z nierwnoci midzy redni aryt-
metyczn a geometryczn. Koczy to dowd.
12. Na paszczynie dany jest n-kt wypuky P. Trjkt utworzony
z trzech rnych wierzchokw P nazywamy dobrym jeli wszystkie jego
boki maj dugo 1. Dowie, e jest nie wicej ni
2
3
n dobrych trjktw.
Rozwizanie
Rozwamy wszystkie dobre trjkty zawierajce ustalony wierzchoek A.
98
Pozostae dwa wierzchoki takiego trjkta musz lee na okrgu
A
o
rodku w A i promieniu rwnym 1. Poniewa wielokt P jest wypuky,
to wierzchoki te le na uku o kcie mniejszym ni 180

. Niech L
A
, R
A
bdzie najkrtszym takim ukiem, przy czym L
A
jest wczeniej ni R
A
w
porzdku zgodnym ze wskazwkami zegara. Zarwno AL
A
, jak i AR
A
s
bokami pewnych dobrych trjktw. Powiemy, e dobry trjkt z bokiem
AL
A
jest przypisany przeciwnie do wskazwek zegara do A, dobry trjkt
z bokiem AR
A
jest za przypisany zgodnie ze wskazwkami zegara do A.
Jeeli jest to ten sam trjkt, to mwimy, e jest on dwukrotnie przyp-
isany do A. Zatem dla kadego wierzchoka s dokadnie dwa przypisania,
czyli oglnie przypisa jest 2n.
Rozwamy pewien dobry trjkt ABC, gdzie wierzchoki A,B oraz C
le w kolejnoci zgodnej ze wskazwkami zegara. Udowodnimy, e ABC
jest przypisany do swoich wierzchokw co najmniej 3 razy. Wwczas, jeli
oznaczymy liczb dobrych trjktw przez t, a ogln liczb przypisa
przez K, to otrzymamy 2n = K , 3t, czyli t
2n
3
, co daje tez zadania.
Udowodnimy konkretnie, e ABC jest przypisany zgodnie ze wska-
zwkami zegara do B lub przypisany przeciwnie do wskazwek zegara do
C. Jeeli to wykaemy, to analogicznie dostaniemy, e ABC jest przyp-
isany zgodnie ze wskazwkami zegara do C lub przeciwnie do A oraz
zgodnie do A lub przeciwnie do B, czyli w oglnoci jest minimalnie 3
razy gdziekolwiek przypisany, co daje tez.
Przypumy przeciwnie, e ABC ani nie jest przypisany zgodnie ze
wskazwkami zegara do B ani przeciwnie do wskazwek zegara do C,
czyli R
B
,= A oraz L
C
,= A. Oznaczmy przez A

, B

oraz C

przecicia
okrgw
A
,
B
oraz
C
rne od A, B i C. Niech CL
C
L

C
bdzie dobrym
trjktem zawierajcym bok CL
C
. Zauwamy, e kt L
C
CA ma miar
mniejsz ni 120

(bo w przeciwnym razie kt L


C
CB byby wikszy ni
180

), czyli jeden z punktw L


C
oraz L

C
naley do uku B

A okrgu
C
,
oznaczmy ten punkt przez X. Jeeli L
C
= B

oraz L

C
= A, to niech
X = B

.
Analogicznie zauwamy, e jeden z punktw R
B
oraz R

B
ley na
uku C

A, oznaczmy ten punkt przez Y , Y ,= A. Wwczas kty XAY ,


Y AB, BAC oraz CAX s mniejsze ni 180

, co znaczy, e punkt A ley


wewntrz czworokta BCXY , co jest sprzeczne z wypukoci P, gdy A,
B, C, X oraz Y s picioma wierzchokami P. Tym samym otrzymujemy
tez.
13. Cig (e
n
) deniujemy nastpujco. e
1
= 1, e
2
= 2, a e
n+1
jest na-
99
jmniejsz liczb, ktra dotychczas nie wystpia w cigu i NWD(e
n
, e
n+1
) >
1. Udowodni, e dla kadej odpowiednio duej liczby pierwszej p pier-
wszym wyrazem cigu (e
n
) podzielnym przez p jest wyraz 2p.
Rozwizanie
Niech e
k
p
bdzie pierwszym wyrazem cigu (e
n
) podzielnym przez p.
Niech A(p) bdzie liczb takich liczb i < k
p
, e e
i
< 2p i e
i+1
, 2p. Dla
kadej takiej pary istnieje liczba pierwsza q
i
taka, e q
i
[NWD(e
i
, e
i+1
).
Oznacza to w szczeglnoci, e wszystkie liczby podzielne przez q
i
mniejsze
od 2p ju wystpiy w cigu, czyli e
i
jest ostatnim wyrazem cigu (e
n
)
podzielnym przez q
i
mniejszym od 2p. W takim razie A(p) nie przekracza
liczby liczb pierwszych mniejszych od 2p: A(p) (2p). Niech teraz B(p)
bdzie liczb takich liczb i < k
p
takich, e e
i
, 2p i e
i+1
< 2p. Przy-
pumy, e e
k
p
= kp > 2p. Rozwamy liczby postaci 2kl, gdzie
p
k
< l <
p
2
Po pierwsze, wszystkie musiay wystpi w cigu z indeksami mniejszymi
od k
p
. W przeciwnym razie z tego, e p e
k
p
1
i NWD(e
k
p
1
, e
k
p
) > 1
mamy NWD(e
k
p
1
, k) > 1 i wyraz postaci 2kl powinien pojawi si za-
miast kp jako mniejszy. Poniewa do tej pory 2p nie wystpio w cigu,
wic po wyrazie postaci 2kl musi pojawi si wyraz mniejszy od 2p. Z
tych rozwaa wynika, e B(p) ,
p1
2

p
k
,
p
6

1
2
>
p
2008
dla p > 3. Z
drugiej strony oczywicie A(p) B(p) , 0. Mamy w takim razie:
0
A(p) B(p)
2p
ln(2p)

(2p)
2p
ln(2p)

ln(2p)
1004
.
Przy p prawa strona dy do , bo
(2p)
2p
ln(2p)
1. Std wniosek, e
dla dostatecznie duych p nierwno nie zachodzi i e
k
p
= 2p.
100
Regulamin Meczu Matematycznego
Ustalenia wstpne
1. W meczu bior udzia dwie druyny. Kada z druyn wybiera ze
swojego grona Kapitana.
2. W pierwszej fazie Meczu obie druyny rozwizuj 12 zada dostar-
czonych przez Jury i przygotowuj si do zreferowania rozwiza
przy tablicy.
3. Drug faz Meczu jest rozgrywka.
Rozgrywka
4. Ekipy na przemian wywouj druyn przeciwn do zreferowania
rozwizania jednego z niewybranych dotd zada. Wywoywanie
rozpoczyna druyna wylosowana tu przed rozgrywk. Numer zada-
nia jest wybierany przez druyn wywoujc.
5. Druyna wywoywana do rozwizania zadania deklaruje, czy przyj-
muje zadanie. Dalszy przebieg rozgrywki zaley od decyzji druyny
wywoanej.
Jeli druyna wywoana przyjmuje zadanie...
6. Druyna wywoana staje si druyn referujc.
7. Zawodnika druyny referujcej, ktry przedstawia rozwizanie przy
tablicy, wyznacza Kapitan druyny przeciwnej.
8. Zawodnik moe by wyznaczony jedynie wtedy, gdy kady zawodnik
z jego druyny zakoczy referowanie zadania nie mniej razy ni
on. Nie mona wyznaczy zawodnika po raz drugi do tego samego
zadania. Jeeli do referowania wyznaczono Kapitana, wskazuje on
na czas pobytu pod tablica swego zastpc.
9. Osoba referujca nie moe korzysta z notatek, ani konsultowa si
ze swoja druyn. Druyna przeciwna nie moe przeszkadza lub
przerywa referujcemu.
10. Kapitan druyny referujcej moe odwoywa osoby referujce dowolna
liczb razy. Take osoba referujca moe zrezygnowa z referowa-
nia.Wwczas Kapitan druyny przeciwnej wskazuje kolejna osob
101
druyny referujcej do kontynuowania rozwizania przy tablicy na
zasadach opisanych w punktach 7 i 8. Druyna zmieniajca referu-
jcego traci N punktw przy swojej N-tej zmianie w czasie Meczu.
11. czny czas na zreferowanie rozwizania przez druyn referujca
wynosi 10 minut. Po upywie tego czasu Jury moe przerwa refer-
owanie, poprosi o streszczenie dalszej czci rozwizania lub poz-
woli na dalsze referowanie, w zalenoci od tego, czy rozwizanie
zdaniem Jury rokuje nadzieje na poprawno i zblia si do koca.
12. Po oznajmieniu przez referujcego, ze referowanie rozwizania zostao
zakoczone, druyna przeciwna moe zgosi zastrzeenia co do po-
prawnoci rozwizania, a nastpnie referujcy odpowiada na te
zastrzeenia.
13. Jeeli podczas dyskusji druyna wywoujca zwrcia uwag na bdy
lub luki dyskwalikujce rozwizanie, ma ona prawo do zreferowania
brakujcych czci rozwizania na zasadach okrelonych w punktach
611.
14. Ostatecznie Jury ocenia zaprezentowane referaty oraz dyskusje i
przyznaje obu druynom nieujemne liczby punktw o sumie nie
przekraczajcej 10 punktw. Druyna, ktra przedstawia poprawne
rozwizanie, otrzymuje co najmniej 7 punktw.
Jeli druyna wywoana nie przyjmuje zadania...
15. Druyna wywoujca staje si druyna referujca i prezentuje roz-
wizanie zgodnie z zasadami okrelonymi w punktach 611.
16. Ostatecznie Jury przyznaje druynie referujcej od 7 do 10 punk-
tw, jeeli zaprezentowane rozwizanie jest poprawne, albo -10 (mi-
nus dziesi) punktw w przeciwnym przypadku. Jury moe rwnie
przydzieli druynie przeciwnej punkty za wskazanie luk lub bdw
w przedstawionym rozwizaniu.
Ustalenia kocowe
17. Rozgrywka koczy si po wywoaniu 8 zada. W przypadku remisu
wywouje si dodatkowo 2 zadania.
18. Przewodniczcy Jury moe naoy kare punktowa na druyn za
niezgodne z niniejszym regulaminem zachowania jej zawodnikw.
102
19. Mecz wygrywa druyna, ktra zdobdzie wicej punktw.
20. Interpretacja niniejszego regulaminu naley do przewodniczcego
Jury.
103

You might also like